You are on page 1of 512

*1

MCQs In
Oral Surgery
By
Babu S.Parmar

Done By: Moh A.S. 2023


Quotes: - Wherever the art of medicine is loved,
there is also a love of humanity
- Cure sometimes, treat often, comfort always
“Hippocrates”

2
MCQs
Oral Surgery

Babu S.Parmar
MDS, FAOMSI

Professor and Head


Department of Oral and Maxillofacial Surgery
Government Dental College and Hospital
New Civil Hospital Campus, Asarwa
Ahmedabad - 380016
Member: Dental Council of India, New Delhi
Hon. Dental Surgeon to HE The Governor of Gujarat

Foreword
Neelima A Malik
JAYPEE BROTHERS
MEDICAL PUBLISHERS (P) LTD
New Delhi

3
Prof. Dr. (Mrs.) Neelima A. Malik
M.D.S. (Bom.), F.I.A.O.S.
ORAL & MAXILLOFACIAL SURGEON
HEAD OF THE DEPT. OF.
ORAL & MAXILLOFACIAL SURGERY
NAIR HOSPITAL DENTAL COLLEGE
DR. A. L. NAIR ROAD, MUMBAI-400 008. INDIA.
Hosp. Tel.: 308 2714
FOREWORD
Since the advent of entrance examinations for various graduate courses
in our country, the criterion for admission is solely based on their
performances in the MCQ based examinations. The Postgraduate
entrance examinations for MDS course, though, began a bit later,
however; have gained a momentum presently all over India for most of
the institutions. Preparation for these examinations is totally different
from that preparing for the other undergraduate examinations. Lot of
hard work with logical learning is required for high marks. In such a
competitive arena, there is a long felt need for good standard MCQ
books, which are more specific and comprehensive with regards to a
particular subject. I personally feel that such books are not only meant
for the entrance examination, but also useful in enrichment of the
knowledge. Professor Babu S. Parmar wishes to publish a book on
MCQs in Oral Surgery. I have seen the manuscript on the laptop and I
am pleased to endorse the work of Dr. Babu S.Parmar, whom I know
from his student days. He has risen in the field of Oral and Maxillofacial
Surgery through his hard and sincere work as a dynamic individual. He
is a dedicated teacher and academician. His vast experience as a DCI
member and an Inspector for various dental colleges has aided in the up
liftmen of the academic standards of the dental students across the
4
country. This book is a collection of MCQs, which have stratified into
the important topics pertaining to Oral and Maxillofacial Surgery. These
topics have thoroughly researched upon and a very concise, yet easy to
comprehend text has laid forth. I being an author of a Textbook of Oral
and Maxillofacial Surgery, know that writing a book is a tedious and
time-consuming job and hence the efforts of Dr.Parmar are commend-
able. His book has almost covered the entire syllabus in the subject,
providing organized, reliable information for the post-graduate entrance
examinations. I am sure that the students, who aspire for success in the
entrance examination, will get high yielding results.
I congratulate and compliment Dr. Babu S.Parmar for his well thought
student friendly excellent work and wish him all the best in his future
endeavors. I also take this opportunity to congratulate the publishers,
who are also providing a platform for all deserving authors through their
constant encouragement, and motivation. Once again, congratulations to
all involved and best wishes.

5
Preface
Life transforming ideas have always come to me through books.
— Bell Hooks
The field of Oral and Maxillofacial Surgery has been ever changing
since past 50 years. The newer advances in the field of reconstruction,
repair, anesthesia and pain have made it a branch of interest not only for
the people belonging to the fraternity but even other specialties.
In this gruesome world of tough competition, I felt under moral
obligation to do something related to my field for those struggling young
minds who aspire to specialize and super-specialize but are defeated by
the system of entrance examinations.
This is a comprehensive book comprising of about 1500 Multiple
Choice Questions assorted from all the topics of Oral Surgery. I have
tried to include all possible questions, which in one or other form can be
asked in the forthcoming entrance examinations or will be of aid to
undergraduate students in their syllabus.
The book also tends to needs of Postgraduate students in giving them a
bird sight view of the subject and can aid them in viva-voce. It is a
matter of pride that the book has been under-taken for publishing by
Jaypee Brothers Medical
Publishers (P) Ltd., New Delhi, whose name in Publishing parallels
authenticity.
Undertaking of such a magnitude cannot completed without the
cooperation and support of some individuals. I sincerely thank to my
postgraduate students for their timely help and support during the
writing of the book.

6
I am also indebted for life to my family for their belief in me and their
unconditional support when I could not spare even minimal time for
them.
I extend a token of gratitude to my Department, Dept. of Oral and
Maxillofacial Surgery, Government Dental College and Hospital,
Ahmedabad for their cooperation. Last but not the least I thank almighty
for his blessings.
Babu S.Parmar

7
Contents

1- Surgical Anatomy ………………………………….. 1


2- LA/GA/Pain Control ……………………………….. 17
3- Sterilization and Instruments ………………………. 78
4- Exodontia / Impaction …………………………...…. 85
5- Minor Surgical Procedure …………………………... 130
6- Cyst Tumor/Surgical Pathology …………………….. 146
7- Facial Space Infection/Osteomyelitis ……………….. 174
8- TMJ and Maxillary Sinus …………………………… 191
9- Maxillofacial Injury …………………………………. 205
10- Reconstructive and Orthognathic Surgery …………... 243
11- Medical Emergency …………………………………. 253
12- Implant ………………………………………………. 297
13- Miscellaneous ……………………………………….. 304

8
Chapter 1: Surgical Anatomy
1. The major blood supply of the Dura mater is via which of the
following artery:
A. Internal carotid
B. Middle cerebral
C. Accessory meningeal
D. Middle meningeal

2. The osteum of the maxillary sinus normally communicates with the


nasal cavity via:
A. The ethmoid infundibulum
B. The infratemporal fossa
C. The nasolacrimal duct
D. Below the level of inferior turbinate

3. When attempting to ligate the lingual artery low in the submandibular


triangle, which of the following nerve is most susceptible to damage?
A. Facial
B. Lingual
C. Accessory
D. Hypoglossal

1- D 2- A 3- D
9
4. The ramus mandibularis branch of the facial nerve has a constant
relationship with which of the following veins in the region of the
mandibular angle.
A. Internal jugular
B. External jugular
C. Deep lingual
D. Retromandibular

5. Which of the following artery is most commonly involved in


extradural hemorrhage?
A. Meningeal branch of the ascending pharyngeal
B. Anterior cerebral
C. Middle meningeal
D. Accessory meningeal

6. When attempting veni puncture in the region of the anti cubital fossa
which of the following artery is most likely to be encountered?
A. Deep brachial
B. Brachial
C. Recurrent radial
D. Radial

4- D 5- C 6- B

10
7. In a patient presenting with a dilating pupil should suspect
involvement of the:
A. Ophthalmic division of the trigeminal nerve
B. Superior cervical ganglion
C. Ciliary ganglion
D. Maxillary division of the trigeminal nerve

8. Aphasia involves the lesion of which of the following areas


A. Cerebral cortex
B. Medulla oblongata
C. Spinal cord
D. Cerebellum

9. In removal of the sub maxillary gland which of the following nerves


are most likely to be damaged?
A. Lingual and glossopharyngeal
B. Lingual and hypoglossal
C. Facial and glossopharyngeal
D. Facial and accessory

7- C 8- A 9- B
11
10. One of the earlier signs of cavernous sinus thrombosis is due to
deficit in the function of:
A. Trochlear nerve
B. Abducent nerve
C. Oculomotor nerve
D. Ophthalmic division of trigeminal nerve

11. Which of the following best illustrates the symptoms of Horner's


syndrome?
A. Myosis and ptosis of the upper eyelid
B. Mydriasis and ptosis
C. Myosis and exopthalmos
D. Ptosis and hydrosis

12. Ataxia involves the lesion of the:


A. Spinal cord
B. Cerebral cortex
C. Medulla oblongata
D. Cerebellum

10- B 11- A 12- D


12
13. The mandibular branch of the trigeminal passes through the:
A. Foramen ovale
B. Foramen rotundum
C. Foramen lacerum
D. Foramen spinosum

14. Roof of pterygomandibular space is formed by:


A. Cranial base
B. Temporal muscle
C. Medial pterygoid
D. Lateral pterygoid

15. The source of motor innervation of the larynx:


A. Glossopharyneal
B. Facial
C. Hypoglossal
D. Vagus

13- A 14- D 15- D

13
16. Which of the following nerves are involved in the gag reflex?
A. Hypoglossal
B. Hypoglossal and glossopharyngeal
C. Glossopharyngeal and vagus
D. Accessory and vagus

17. Which are the following cranial nerves does not make the cranial
component of the parasympathetic system?
A. Occulomotor
B. Accessory
C. Facial
D. vagus

18. Which of the following innervates the buccal gingiva of maxillary


second premolar?
A. Buccal nerve
B. Anterior palatine nerve
C. Posterior palatine nerve
D. Middle superior alveolar nerve

16- C 17- B 18- D


14
19. Which is the only medial branch of the external carotid artery?
A. Ascending pharyngeal
B. Superior thyroid
C. Lingual
D. Internal maxillary

20. Which is not one of the main sensory branches of the mandibular
division of the trigeminal nerve?
A. Lingual nerve
B. Inferior alveolar nerve
C. Nerve to the mylohyoid
D. Auriculo temporal nerve

21. Which duct provides drainage to the parotid gland?


A. Bartholin's
B. Santorinis
C. Warthon's
D. Stenson's

19- A 20- C 21- D

15
22. In acute airway obstruction should it be necessary to perform a
coniotomy the entry should be made at:
A. Cricoid sartilage
B. Thyroid notch
C. Cricothyroid ligament
D. Thyroid membrane

23. In the pterygomandibular space the inferior alveolar nerve passes:


A. Medial to the pterygomandibular ligament
B. Medial to the medial pterygoid muscle
C. Lateral to the sphenomandibular ligament
D. Anterior to the deep tendon of temporalis muscle

24. During removal of the torus palatinus the mid portion of the palatine
process of the maxilla is inadvertently removed. One would expect to
see:
A. An opening in to the nasal cavity
B. A vertical fracture of the maxilla
C. An opening in to maxillary antrum
D. A horizontal fracture of maxilla

22- C 23- C 24- A

16
25. Blood vessels and nerves are generally scarce in which component
of TMJ
A. Anterior portion of disc
B. Posterior portion of disc
C. Central portion of disc
D. Articular capsules

26. The facial nerve exits from skull from which foramen?
A. Ovale
B. Rotundum
C. Stylomastoid
D. Spinosum

27. Motor innervation of the muscle of facial expression is via:


A. Abducent nerve
B. Facial nerve
C. 1st division of the trigeminal nerve
D. 2nd division of the trigeminal nerve

25- C 26- C 27- B

17
28. The middle meningeal artery is associated with which foramen?
A. Ovale
B. Rotundum
C. Stylomastoid
D. Spinosum

29. A lesion of the lingual branches of the glossopharyngeal nerve


would result in the loss of taste from which portion of the tongue?
A. Anterior third
B. Anterior two-third
C. Posterior third
D. Posterior two-third

30. Langer's line in the face usually are:


A. Parallel to the natural creases of the face
B. At right angle to the natural creases of the face
C. Due to subcutaneous interstitial fluid
D. Due to subcutaneous fat of the face

28- D 29- C 30- A

18
31. Which of the following is not a structural part of the
temporomandibular joint?
A. Sigmoid notch
B. Condylar process
C. Articular disc
D. Capsular ligament

32. Which is not an anterior triangle of neck?


A. Diagastric
B. Muscular
C. Parotid
D. Subclavian

33. Which autonomic ganglion is not associated with the function of the
major salivary gland?
A. Otic
B. Ciliary
C. Submaxillary
D. None of the above

31- A 32- D 33- B


19
34. The hypoglossal nerve provides:
A. Motor innervation to all of the muscles of the tongue both intrinsic
and extrinsic
B. Sensory innervation to the anterior two-third of the tongue
C. Motor innervation to the styloglossus and hypoglossus muscles only
D. Sensory innervation to the posterior third of the
Tongue

35. In phonation, the true vocal cords deal primarily with:


A. Expiration
B. Inspiration
C. Both of the above
D. None of the above

36. In the inferior alveolar nerve block, the needle is inserted adjacent
and lateral to a raphe formed by the buccinators and the:
A. Temporalis
B. Internal pterygoid
C. Superior constrictor of the pharynx
D. Middle constrictor of the pharynx

34- A 35- B 36- C


20
37. Classic Bell's palsy results from a lesion involving which of the
following nerves?
A. Trigeminal
B. Hypoglossal
C. Glossopharyngeal
D. Facial

38. The lingual artery is a branch of which of the following artery?


A. External carotid
B. Internal carotid
C. Internal maxillary
D. External maxillary

39. Which one of the following veins does not drain into the internal
jugular?
A. Vein
B. Posterior facial (retromandibular)
C. Anterior facial
D. Posterior auricular

37- D 38- A 39- A


21
40. The mesenteric artery arises from which of the following artery:
A. Middle meningeal
B. Internal maxillary
C. External maxillary
D. Inferior alveolar

41. The triangle of the lingual artery is:


A. Lesser's triangle
B. Carotid triangle
C. Submental triangle
D. Muscular triangle

42. Severe trismus resulting from acute dental pathology indicates


swelling in which one of the following spaces:
A. Sublingual
B. Carotid
C. Retropharyngeal
D. Masticator

40- B 41- A 42- D

22
43. Which of the following muscle is an intrinsic muscle of the tongue?
A. Chondroglossus
B. Glossopalatinus
C. Transverses lingual
D. Hypoglossus

44. Which of the following is not a branch of the internal maxillary


artery?
A. Posterior auricular
B. Infraorbital
C. Posterior superior alveolar
D. Inferior alveolar

45. Which of the following is innervated by vagus nerve?


A. Mylohyoid muscle
B. Tensor veli palatine
C. Levator veli palatine
D. Posterior belly of diagastric

43- C 44- A 45- C

23
46. Which muscle does not function in opening of the mandible?
A. External pterygoid
B. Diagastric
C. Hypoglossus
D. Mylohyoid

47. The carotid triangle is separated from the submaxillary triangle by:
A. The superior belly of omohyoid muscle
B. The hyoid bone
C. Anterior belly of diagastric
D. Posterior belly of diagastric

48. The hypoglossal nerve is:


A. Purely sensory nerve
B. Mixed nerve
C. Autonomic nerve
D. Purely motor nerve

46- C 47- A 48- D

24
49. Which are components of the posterior triangle of neck?
A. Occipital triangle
B. Carotid triangle
C. Subclavian triangle & occipital triangle
D. Muscular triangle

50. The trigeminal nerve is:


A. Mixed nerve
B. Purely motor nerve
C. Purely sensory nerve
D. Mixed and autonomic nerve

51. Which one of the following bones does not form the part of the
orbit?
A. Palatine
B. Maxillary
C. Frontal
D. Nasal

49- C 50- A 51- D

25
52. The no of branches of the internal carotid artery in the neck:
A. None
B. 1
C. 2
D. 3

53. Which one of the following nerves has a constant relation to the
occipital artery as it arises from the external carotid artery?
A. Vagus
C. Accessory
B. Glossopharyngeal
D. Hypoglossal

54. The semilunar/gasserian ganglion is found in a space known as:


A. Scarpa's space
B, Meckel's cavity
C. Antrum of Highmore
D. Rathke's pouch

52- A 53-D 54- B

26
55. The course of the lingual nerve in relation to the submaxillary duct
as it passes forward is sequentially:
A. Below, medial, superior, lateral
B. Above, medial, inferior, lateral
C. Above, lateral, inferior, medial
D. Below, lateral, superior, lateral

56. The intrinsic muscles of the larynx deals with the true and false
vocal cords. Which cranial nerve innervates these muscles?
A. Vagus
B. Spinal accessory
C. Hypoglossal
D. Glossopharyngeal

57. Which structure passes between two roots of the auriculotemporal


nerve?
A. Inferior alveolar artery
B. Internal maxillary artery
C. Middle meningeal artery
D. Sphenomandibular ligament

55- C 56- A 57- C

27
58. The Ansa hypoglossal is composed of the:
A. Ascending cervical and ascending hypoglossal nerve
B. Ascending cervical and descending hypoglossal nerve
C. Descending cervical and ascending hypoglossal nerve
D. Descending cervical and descending hypoglossal nerve

59. Which syndrome consists of flushing, warmness, perspiration over


the cheek and pinna of the ear on one side following ingestion of highly
seasoned foods?
A. Cushing's syndrome
B. Horner's syndrome
C. Auriculotemporal syndrome
D. Fanconi's syndrome

60. The common carotid artery usually divides into the external and
internal carotid arteries at the level of:
A. Hyoid bone
B. Above the level of hyoid bone
C. Superior border of thyroid cartilage
D. Inferior border of thyroid cartilage

58- D 59- C 60- C

28
61. The maxillary branch of trigeminal nerve passes through the:
A. Foramen ovale
B. Foramen rotundum
C. Superior orbital fissure
D. Foramen lacerum

62. Which is not a branch of facial nerve?


A. Zygomatic
B. Temporal
C. Buccal
D. Greater auricular

63. Which artery is not a part of circle of Willis?


A. Internal carotid
B. Posterior communicating
C. Anterior cerebral
D. Vertebral

61- B 62- D 63- D

29
64. The schneiderian membrane lines the:
A. Maxillary sinus
B. Oral cavity
C. Heart
D. Inner ear

65. The motor component of the trigeminal nerve supplies all of the
following except:
A. Muscles of mastication
B. Anterior belly of digastric
C. Tensor tympani
D. Platysma muscle

66. The maxillary sinus is also known as:


A. Meckels cave
B. Aantrum of Highmore
C. Burns space
D. Rathke's pouch

64- A 65- D 66- B

30
67. Which nerve supplies sensory innervation for taste to the anterior
two-third of the tongue?
A. Hypoglossal
B. Lingual
C. Chorda tympani
D. vagus

68. Which nerve is not a branch of the trigeminal?


A. Digastric
B. Mandibular
C. Maxillary
D. Lingual

69. Waldeyer's ring contains what type of tissue:


A. Muscle
B. Nerve
C. Lymphoid
D. None of the above

67- C 68- A 69- C

31
70. Motor innervation to the sternocleidomastoid muscle is supplied by:
A. Hypoglossal
B. Spinal accessory
C. Glossopharyngeal
D. vagus

71. The facial nerve supplies all of the following except:


A. Anterior belly of digastric
B. Posterior belly of digastric
C. Platysma
D. scalp

72. The left common carotid artery usually arises from:


A. Left auricle of the heart
B. Costocervical trunk
C. Arc of aorta
D. Brachiocephalic artery

70- B 71- A 72- C

32
73. Greater palatine foramen is situated:
A. Between first and second maxillary molars
B. Between second and third maxillary molars
C. Between two central incisors
D. Between first and second maxillary premolars

74. Of the following which artery is most commonly involved in stroke?


A. Lenticulostriate artery
B. Cerebral artery
C. Ophthalmic artery
D. None of the above

75. Any laceration of the scalp causes less bleeding:


A. Because of less blood supply
B. Because the vessels are bound loosely in the connective tissue and
retract readily
C. All of the above
D. Above statement is wrong

73- B 74- A 75- D

33
76. Mental foramen opening is directed in:
A. Forward and medial direction
B. Backward and medial direction
C. Forward and lateral direction
D. Backward and lateral direction

77. In pterygomandibular space the inferior alveolar nerve passes:


A. Medial to stylomandibular ligament
B. Anterior to the deep tendon of the temporal muscle
C. Lateral to the spheno mandibular ligament
D. Superior to lateral pterygoid muscle

78. Of the following which tissue has the least regenerating capacity
after injury?
A. Bone
B. Liver
C. Peripheral nerve
D. Tendon

76- D 77- C 78- D

34
79. Sensory fibers of lingual nerve supplies to:
A. Tongue
B. Lingual surface of mandible
C. Floor of the mouth
D. All of the above

80. All of the following bones contain air sinuses except:


A. Frontal
B. Nasal
C. Sphenoid
D. Ethmoid

79- D 80- B

35
Chapter 2: LA\GA\Pain Control
1. Epinephrine is added to local anesthetics because
A. Decreases the rate of absorption of the local anesthetic at the injection
site
B. Prevents the rapid deterioration of the local anesthetic solution
C. Increases the rate of destruction of the local anesthetic
D. Potentiates the action of all local anesthetics

2. The most likely cause of trismus after block anesthesia for surgery in
the mandibular molar area is:
A. Excessive edema
B. Damage to the medical pterygoid muscle on injection
C. Stretching of the lateral pterygoid muscle
D. Submandibular cellulitis

3. The patient has received an injection of 1.8 ml of local anesthetic


containing 2 percent lidocaine with epinephrine. Thirty seconds later he
goes into syncope. The most probable cause is:
A. Bradycardia
B. Tachycardia
C. Cerebral hypoxia
D, A toxic reaction to the lidocaine

1. A 2. B 3. C

36
4. Nitrous oxide alone isn’t used as a general anesthetic agent because of
the:
A. Difficulty in maintaining an adequate oxygen concentration
B. Expense of the agent and its explosive hazard
C. Adverse effects on the liver
D. Poor analgesic properties

5. When attempting to achieve an intraoral palatal second division block


anesthesia, the needle should enter the:
A. Greater palatine foramen
B. Stylomastoid foramen
C. Nasopalatine foramen
D. Foramen rotundum

6. A deep level of general anesthesia is enhanced by:


A. High alveolar concentration of anesthetic agent
B. Loose-fitting mask
C. Nonirritating drug
D. Muscle relaxant

4. A 5. A 6. A
37
7. A decrease in interstitial fluid pH will:
A. Decrease the effectiveness of a local anesthetic block
B. Increase the effectiveness of a local anesthetic block
C. Have no effect on the effectiveness of a local anesthetic block
D. Decrease, then increase the effectiveness of a local anesthetic block

8. Which of the following deep bony landmarks is important in


performing a block of the second and third divisions of the trigeminal
nerve from the lateral approach?
A. Mastoid bone
B. Styloid process
C. Temporal surface of the sphenoid
D. Lateral plate of the pterygoid process

9. A history of medication with which of the following drugs requires


special consideration prior to general anesthesia?
A. Estrogen
B. Cortisone
C. Meperidine
D. Phenacetin

7. A 8. D 9. B

38
10. When administered in concentrations necessary to produce sedation,
nitrous oxide:
A. Does not produce nausea
B. Combines with hemoglobin
C. Involves no risk for the patient
D. Will depress the bone marrow and peripheral white cell counts in
man after prolonged use

11. Which of the following areas are anesthetized due to the anterior
(greater) palatine nerve block?
A. Incisors and canine on the injected side
B. Upper lip, nose and lower eyelid
C. Posterior portion of the hard palate and overlying structures up to the
first premolar on the injected side
D. Incisor to premolar on the injected side

12. Of the following which group of drugs eliminate all sensations?


A. Anesthetics
B. Analgesics
C. Narcotics
D. Sedatives

10. D 11. C 12. A

39
13. Of the following which causes anesthesia of the lower lip?
A. Metastatic malignancy
B. CNS tumor
C. Fracture mandible body region
D. All of the above

14. For what purposes adrenaline is added to procaine?


A. Reduction of hemorrhage in the field of operation
B. Prevention of toxic effects from too rapid absorption
C. Prolongation of anesthesia
D. All of the above

15. Following area of the brain is the "last" one depressed by GA agents:
A. Medulla.
B. Cerebellum
C. Pons
D. None of the above

13. D 14. D 15. A

40
16. The presser effect of both epinephrine and norepinephrine is
increased by which of the following:
A. Procaine
B. Dibucaine
C. Cocaine
D. Lidocaine

17. A nerve is absolutely refractory during:


A. Depolarization
B. After depolarization
C. Hyperpolarization
D. Firing level only

18. A nerve can be stimulated during relative refractory period by:


A. Stronger than normal stimuli
B. Sustained normal stimuli
C. Sub threshold stimuli
D. None of the above

16. C 17. A 18. A

41
19. Local anesthetic agents act by:
A. Increasing the rate of depolarization
B. Shortening the rate of repolarization
C. Decreasing the threshold potential
D. Increasing the threshold potential

20. The local anesthetic agent acts on:


A. Nerve membrane
B. Axoplasm
C. Epineurium
D. Perineurium

21. The most acceptable theory which explains the actions of LA:
A. Surface charge theory
B. Calcium displacement theory
C. Membrane expansion theory
D. Receptor binding theory

19. D 20. A 21. D

42
22. The ultimate action of binding the receptor by agent is brought about
by its:
A. Hydrophilic component
B. Lipophilic component
C. Intermediary chain
D. RN of amide agents

23. Amide group of local anesthetic agents are dispensed as salts of


strong acids because:
A. They are not lipid soluble but stable in air
B. They are not water soluble but stable in air
C. They are not water soluble and unstable in air
D. They are lipid soluble but stable in air

24. In acidic medium (during pyogenic infections) local anesthetics are


less effective because:
A. More uncharged particles are released
B. Less uncharged particles are released
C. Less charged particles are released
D. None of the above

22. A 23. C 24. B

43
25. Local anesthetic agents with higher pka would have:
A. Shorter onset of action
B. Longer onset of action
C. No effect on onset of action
D. None of the above

26. Which characteristic of a LA agent is responsible for its penetration


into the nerve?
A. Lipid solubility
B. Water solubility
C. Its ionization
D. None of the above

27. Addition of a vasoconstrictor to LA agents:


A. Increases alkalinity of the solution
B. Increases acidity of the solution
C. Has no effect on the pH
D. None of the above

25. B 26. A 27. B

44
28. Sodium bisulfite has the following effect on the action of LA
solution:
A. Slows down its onset of action
B. Decreases its duration of action
C. Increases its pH
D. Has no affect

29. Increasing the concentration of LA from 2 to 5% would have:


A. Rapid onset and prolonged action
B. Onset would not be affected but action would be prolonged
C. No change on action
D. Rapid onset and duration not affected

30. The efficacy of benzocaine in inflamed area would be:


A. Decreased
B. Increased
C. Not altered
D. Prolonged

28. A 29. D 30. C

45
31. The main barrier for diffusion of LA into the nerve is:
A. Epineurium
B. Perineurium
C. Endoneurium
D. Neural membrane

32. Which fibers of the nerve are anaesthetized first:


A. Mantle fibers
B. Core fibers
C. Both are anaesthetized at the same time
D. None of the above

33. The inadequate pulpal anesthesia in presence of adequate soft tissue


anesthesia can be due to:
A. Faulty technique
B. Insufficient penetration of core fibers
C. Insufficient penetration of fasciculi
D. Insufficient penetration of mantle fibers

31. C 32. C 33. B

46
34. Prolonged duration of action of drugs like etidocaine and
bupivacaine can be attributed to their:
A. Increased lipid solubility
B. Increased water solubility
C. Increased protein binding
D. Vasoconstrictor action

35. Tachyphylaxis occurs due to:


A. Increased dose of LA
B. Increased dose of vasoconstrictor
C. Repeated use of LA
D. Allergy to sodium metabisulfite

36. Which of the following belongs to ester group of local anaesthetics?


A. Bupivacaine
B. Benzocaine
C. Etidocaine
D. Mepivacaine

34. C 35. C 36. B

47
37. The only local anesthetic with vasoconstrictor
A. Cocaine
B. Procaine
C. Benzocaine
D. Lidocaine

38. Which of the following is not an ester local anesthesia


A. Propoxycaine
B. Procaine
C. Prilocaine
D. Piperacaine

39. Which of the following local anesthetics crosses the blood-brain


barrier:
A. Lignocaine
B. Etidocaine
C. Bupivacaine
D. All of the above

37. A 38. C 39. D

48
40. All of the local anesthetics cross the placenta:
A. Except mepivacaine
B. Statement is true
C. Statement is false
D. Except lignocaine

41. Ester type local anesthetics are metabolized in the:


A. Liver only
B. Kidney
C. Plasma
D. Lungs

42. A patient who had a history of prolonged apnoea during


administration of muscle relaxant (succinylcholine) should not be given?
A. Ester local anesthetics
B. Amide local anesthetics
C. Vasoconstrictors
D. Sodium bisulfite

40. B 41. C 42. A

49
43. A patient complains of history of hepatitis one month ago should be
preferably given which local anesthetic agent?
A. Lignocaine
B. Bupivacaine
C. Procaine
D. Procainamide

44. One of the complications of prilocaine LA is:


A. Agranulocytosis
B. Hepatic dysfunction
C. Methemoglobinemia
D. None of the above

45. Local anesthetics are excreted mainly by:


A. Lungs
B. Fecal route
C. Kidneys
D. Uterus

43. C 44. C 45. C

50
46. Blood level of > 7 ug\ml of LA produces:
A. Anticonvulsive activity
B. Tonic clonic seizure
C. CVS stimulation
D. None of the above

47. The anticonvulsant activity of local anesthetic agents occurs at:


A. < 4 ug/ml
B. 4-7 ug/ml
C. 7-10 ug/ml
D. 10-12 ug/ml

48. The differentiating factor between LA toxicity and developing


syncope would be:
A. CNS stimulation
B. CNS depression
C. Pallor of skin
D. Light reflex

46. B 47. A 48. A

51
49. The level of 2% lidocaine which reaches blood after use of one or
two cartridges is:
A. 0.5-2 ug/ml
B. 1.5-5 ug/ml
C. 5-10 ug/ml
D. No LA reaches blood if given safely after aspiration

50. A patient with known history of hyperthermia should be given:


A. Lignocaine
B. Procaine
C. Bupivacaine
D. Mepivacaine

51. With overdose of local anesthetic agent one would observe:


A. Hypertension
B. Hypotension
C. No change in BP
D. Cardiac arrhythmias

49. A 50. B 51. B

52
52. Epinephrine (Adrenalin) which is used in dental cartridge of LA acts
on:
A. a receptors only
B. P receptors only
C. a and P receptors but p predominantly
D. a and P receptors but a predominantly

53. When local anesthetic agent with adrenalin is injected, the


termination of activity of the vasoconstrictor is brought by:
A. Adrenergic nerve endings
B. Blood enzymes COMT and MAO
C. Excretion in urine (80%)
D. A and B

54. Use of norepinephrine in dental practice is not recommended


because it causes:
A. Bradycardia
B. Intense peripheral vasoconstriction
C. Hypertension
D. Sensitization of myocardium

52. C 53. D 54. B

53
55. When one has to use the weakest vasoconstrictor (e.g., in patient
with history of angina) one should consider:
A. Epinephrine
B. Norepinephrine
C. Phenylephline
D. Levonordefrin

56. Rebound phenomenon is most commonly seen with use of:


A. Epinephrine
B. Norepinephrine
C. Phenylephrine
D. Levonordefrin

57. The absolute contraindication for use of adrenalin in LA is:


A. Myocardial infarction, 3-6 months ago
B. Angina pectoris
C. Hyperthyroidism
D. Pregnancy

55. C 56. A 57. C

54
58. Adrenalin should not be used when halothane is used during GA
because halothane:
A. Sensitizes the myocardium to adrenalin
B. Increases the heart rate
C. Increases the blood pressure
D. Interferes with AV conduction

59. A cartridge of LA contains adrenalin, it indicates that there is:


A. 0.005 mg/ml of adrenalin
B. 0.065 mg/ml of adrenalin
C. 0.0125 mg/ml of adrenalin
D. 0.02 mg/ml of adrenalin

60. Maximum dose of adrenalin which can be given to a patient with


history of cardiovascular disease is:
A. 0.2 mg/ml
B. 0.2 mg
C. 0.04 mg
D. 0.005 mg/ml

58. A 59. A 60. C


55
61. To a patient of 50 kg weight how many cartridges of LA with 1:
200,000 adrenalins can be given (considering patient is normal, healthy
and lignocaine toxicity is not considered):
A. 10.5
B. 22
C. 32
D. 40

62. Lidocaine was first prepared by:


A. Nils Lofgren
B. Ekenstam
C. Einhorn
D. None of the above

63. The duration and depth of pulpal anesthesia with lignocaine (2%)
added to epinephrine as compared to 1: 100,000 epinephrine would be:
A. Longer and profound
B. Duration would be two times longer but depth would be same
C. No much difference
D. Duration would be 4 times

61. B 62. A 63. C

56
64. Three percent lignocaine indicates that there is:
A. 25 mg/ml of lignocaine
B. 30 mg/ml of lignocaine
C. 54 mg/ml of lignocaine
D. 27 mg/ml of lignocaine

65. How many cartridges of 2% lignocaine can be given to a 50 kg man


(with adrenalin)?
A. 6
B. 12
C. 17
D. 25

66. When vasoconstrictor is contraindicated, the ideal local anaesthetic


would be:
A. Lignocaine
B. Mepivacaine
C. Cocaine
D. Bupivacaine

64. B 65. A 66. B


57
67. A patient with respiratory disease presents for treatment, which drug
should not be used:
A. Lignocaine
B. Adrenaline
C. Prilocaine
D. Mepivacaine

68. Which of the following is least toxic LA?


A. Lignocaine
B. Mepivacaine
C. Propoxycaine
D. Bupivacaine

69. When injecting into relatively highly vascular area as in posterior


superior alveolar nerve block, one should use needle with:
A. Smaller gauge
B. Larger gauge
C. Gauge does not matter
D. None of the above

67. C 68. D 69. B


58
70. One should use a needle which has:
A. Greatest angle of bevel
B. Minimum angle of bevel with tip lying in the center of the lumen
C. No bevel at all
D. None of the above

71. Which of the following is a long acting LA agent?


A. Mepivacaine
B. Bupivacaine
C. Prilocaine
D. Propoxycaine

72. The safest local anesthetic agent:


A. Cocaine
B. Procaine
C. Chloroprocaine
D. Propoxycaine

70. B 71. B 72. C

59
73. Which local anesthetic agent when used topically interferes with
sulphonamide action:
A. Lidocaine base
B. Lidocaine
C. Benzocaine
D. Propoxycaine

74. Self-aspirating syringes, provide aspiration by:


A. Pulling the thumb ring
B. Negative pressure created due to elasticity of rubber diaphragm
C. Pressure release on thumb disc
D. All of the above

75. The jet injectors are used to obtain:


A. Pulpal anesthesia
B. Topical anesthesia
C. Regional block
D. Nerve block also

73. D 74. D 75. B

60
76. The gauge of needle used in dental syringes refers to:
A. Internal diameter of the lumen
B. External diameter of needle
C. Diameter of bevel only
D. Diameter of hub

77. Sodium bisulphite used in dental LA cartridge acts as:


A. Antioxidant for adrenalin
B. Antioxidant for lignocaine
C. Antibacterial for lignocaine
D. Not used any more

78. The allergic reactions commonly seen following use of cartridge of


LA is due to:
A. Lignocaine
B. Vasoconstrictor
C. Methyl paraben
D. Sodium metabisulphite

76. A 77. A 78. C

61
79. Glass LA cartridge should be sterilized by:
A. Autoclaving
B. Dry heat
C. Cold sterilization
D. None of the above

80. If the diaphragm of cartridge is soaked in isopropyl alcohol for


purpose of antisepsis, it may result in:
A. Reduced anesthesia
B. No anesthesia
C. Long-term paresthesia
D. None of the above

81. Local infiltration should be:


A. Para periosteal
B. Subperiosteal
C. Transeptal
D. None of the above

79. D 80. C 81. A

62
82. Infiltration is not successful for anaesthetizing buccal roots of:
A. Maxillary 1st permanent molar
B. Maxillary 1st deciduous molar
C. Mandibular 1st permanent molar
D. Deciduous maxillary 1st molar

83. For posterior superior alveolar nerve one should use:


A. Long needle (40 mm)
B. Short needle (25 mm)
C. Length not a criterion
D. Only bevel should be considered

84. Greater palatine foramen is present:


A. Between 1st and 2nd maxillary molars
B. Between 2nd and 3rd maxillary molars
C. Distal to 3rd maxillary molar
D. Mesial to 1st maxillary molar

82. C 83. B 84. B

63
85. In 80% of patient’s infraorbital nerve block is effective for buccal
aspect of:
A. Central incisors and canines
B. Central incisors to 1st premolars
C. Central incisors to mesiobuccal root of 1st maxillary molar
D. Central incisors only

86. In greater palatine nerve block the needle should be:


A. Parallel to mucosa
B. Perpendicular to mucosa
C. 450 inclined to mucosa
D. Parallel to roots of molars

87. The two techniques used for maxillary block are:


A, Greater and lesser palatine approach
B. Greater palatine and high tuberosity approach
C. Greater tuberosity and retromolar approach
D. Gow gates and Akinosi technique

85. C 86. B 87. B

64
88. Inferior alveolar nerve block anaesthetizes all except:
A. Body of mandible lower part
B. Mandibular teeth
C. Mucous membrane anterior of first mandibular molar
D. Mucous membrane distal to 1st mandibular molar

89. The needle while giving inferior alveolar nerve block passes
through:
A. Buccinator muscle
B. Pterygomandibular raphe
C. Buccal fat
D. Stylomandibular raphe

90. While giving inferior alveolar nerve block the needle is lateral to:
A. Lingual nerve
B. Sphenomandibular ligament
C. Medial pterygoid muscle
D. All of the above

88. D 89. A 90. D

65
91. If bone is not contacted before injecting local anesthetic in inferior
alveolar nerve block, there are chances of:
A. Lingual nerve anesthesia
B. Transient facial palsy
C. Transient maxillary anesthesia
D. None of the above

92. Inferior alveolar nerve block at times is not very effective because
which of the nerves is not anaesthetized:
A. Mental nerve
B. Lingual nerve
C. Mylohyoid nerve
D. Incisal nerve

93. Gow gates technique is for:


A. Mandibular nerve block
B. Inferior alveolar nerve block
C. Trigeminal ganglion block
D. V2, V3, block

91. B 92. C 93. A


66
94. The target of Gow Gates technique is:
A. Coronoid notch
B. Sigmoid notch
C. Foramen ovale
D. Neck of condyle

95. In patients with reduced mouth opening which technique of


mandibular anesthesia should be used?
A. Gow gates
B. Akinosis
C. Labyrinths
D. Williams

96. For extra oral maxillary nerve block the target area is:
A. Posterior to lateral pterygoid plate
B. Anterior to lateral pterygoid plate
C. Pterygomandibular fissure
D. Pterygomandibular fossa

94. D 95. B 96. B

67
97. For extra oral mandibular nerve block the needle should be inserted
from:
A. Above the zygomatic arch
B. Below the zygomatic arch
C. Coronoid notch
D. None of the above

98. If needle breaks during injecting LA and radio graphically it appears


to be deep in tissues, the advised management would be:
A. Removal of needle under LA
B. Removal of needle under GA
C. Leaving the needle in the tissue
D. None of the above

99. Use of which of the solutions relatively can have more burning
sensation?
A. Plain lignocaine
B. Isotonic solution
C. Lignocaine and adrenalin
D. Hypotonic solution

97. B 98. C 99. C

68
100. Persistent anesthesia can result most often in which of the nerves,
after LA injection?
A. Inferior alveolar
B. Lingual nerve
C. Infraorbital nerve
D. Mental nerve

101. Aspiration should be carried out at least in:


A. One plane
B. Two planes
C. Three planes
D. Four planes

102. ECG changes can first be observed when level of lignocaine is


more than:
A. 5-6 ug/ml
B. 10-12 ug/ml
C. 2-4 ug/ml
D. 1-2 ug/ml

100. B 101. B 102. A


69
103. To control tonic clonic seizures following lignocaine toxicity the
drug of choice would be:
A. Pentobarbital
B. Diazepam
C. Succinylcholine
D. Antihistaminic

104. Succinylcholine can be used for control of tonic clonic seizures but
along with this:
A. Pentobarbitone should be used
B. Artificial respiration is must
C. Atropine should be given
D. Neostigmine should be given to terminate its action

105. Postictal phase, which follows CNS toxicity with Lignocaine


should be managed by:
A. CNS stimulants
B. Analeptics
C. Supportive therapy only
D. Diazepam

103. B 104. B 105. C

70
106. During CVS depression in lignocaine toxicity, one should
administer:
A. Vasoconstrictors
B. Atropine
C. Crystalloids
D. All of the above

107. Gingival retraction cords contain adrenalin in concentration of:


A. 1:200,000 / inch
B. 0.3 ug / inch - 1.0 ug / inch
C. 300 ug/inch-1000 ug/inch
D. 1000 ug/inch-2000 Pig/ inch

108. Extraction of deciduous teeth in dental clinic can be carried out in


which stage of general anesthesia:
A. Stage I
B. stage II
C. stage III plane II
D. stage III plane III

106. D 107. C 108. A


71
109. Surgical plane for major surgery during general anesthesia is
during:
A. stage III plane I
B. stage III plane II
C. stage III plane III
D. stage HI plane IV

110. Succinylcholine is administered during GA for:


A. Better control
B. Intubation
C. Prevention of apnoea
D. Decreasing respiratory rate and thus decreasing GA toxicity

111. Nowadays induction phase of GA has been reduced because of use


of:
A. Halothane
B. Ether, halothane combination
C. Thiopentone sodium
D. Ether, N2O, halothane combination

109. C 110. B 111. C


72
112. If long-acting muscle relaxants are used during GA, their action is
terminated by use of:
A. Neostigmine
B. Atropine
C. Ketamine
D. Succinylcholine

113. The N2O gas cylinder used in GA is:


A. White and black colored
B. Blue colored
C. Red and yellow colored
D. White and blue colored

114. For maxillofacial injuries one should always use which


endotracheal tube for GA
A. Non inflatable
B. Inflatable cuffed
C. Catheterized
D. None of the above

112. A 113. B 114. B


73
115. The endotracheal tube should be placed for GA:
A. In right bronchus
B. In left bronchus
C. Above cricoid
D. In laryngopharynx

116. Rotameter on Boyle's trolley for GA is used to measure:


A. Pressure of gas in the cylinders
B. Pressure of halothane
C. Flow of gases in the tubes
D. None of the above

117. Goldman's vapourizer is used for:


A. N2O
B. 50% N2O + 20% O2 mixture
C. 50% N2O + 50% O2 mixture
D. 50% either + 20% O2 mixture

115. C 116. C 117. B

74
118. In TMJ ankylosis patient, GA cannot be administered
A. Oral intubation
B. Blind nasal intubation
C. Fiberoptic assisted intubation
D. B and C

119. In TMJ ankylosis patient, GA can be administered


A. Oral intubation
B. Blind nasal intubation
C. Fiberoptic assisted intubation
D. B and C

120. A patient who is being operated under halothane should not be


given:
A. Lignocaine
B. Lignocaine + adrenalin
C. Propoxycaine
D. Mepivacaine

118. A 119. D 120. B

75
121. Glycopyrrolate is usually used during GA to:
A. Reduce heart rate
B. Increase BP
C. Reduce secretions
D. Control bleeding

122. In most surgical procedures, GA with N2O is given


A. 70% N2O + 30% O2
B. 70% N2O + 20-30% O2 + other GA agent
C. 50% N2O + 50% O2 + other GA agent
D. 70% O2 + 20% N2O

123. Which of the following is used as a dissociative the anesthetic


agent:
A. Fentanyl
B. Thiopentone
C. Ketamine
D. Halothane + either mixture

121. C 122. D 123. C

76
124. During GA oxygen concentration of blood should not fall below:
A. 90%
B. 60%
C. 40%
D. 20%

125. Local anesthetics are most effective in tissue that have what pH?
A. Below 7
B. Above 7
C. Below 4
D. Makes no difference what the pH of the tissue is

126. A 40-year-old male develops right facial and head pain which is
intermittent with abrupt onset and cessation. It usually occurs at night
awakening him from sleep. The pain is associated with unilateral
flushing, sweating, rhinorrhea and increased lacrimation. What is the
most likely diagnosis?
A. Tic doulourcux
B. Vidian neuralgia
C. Sphenopalatine ganglion neuralgia
D. Histamine cephalgia

124. A 125. B 126. D

77
127. Which local anesthetic listed below may possibly manifest its
toxicity clinically by initial depression and drowsiness rather than
stimulation and convulsion?
A. Lidocaine
B. Procaine
C. Benzocaine
D. Tetracaine

128. How will a larger than normal functional residual capacity affect?
A. Nitrous oxide sedation
B. Nitrous oxide sedation will happen much quicker
C. Nitrous oxide sedation will take longer
D. Functional residual capacity does not affect nitrous oxide sedation

129. A patient who is allergic to para-amino benzoic acid will also be


allergic to which of the following:
A. Lidocaine
B. Tetracaine
C. Prilocaine
D. Carbocaine

127. A 128. B 129. B

78
130. Nitrous oxide works on which system listed below?
A. Peripheral nervous system (PNS)
B. Central nervous system (CNS)
C. Autonomic nervous system (ANS)
D. None of the above

131. During an inferior alveolar block injection, the needle passes


through the mucous membrane and the buccinator muscle and lies lateral
to which muscle listed below?
A. Masseter
B. Lateral pterygoid
C. Hypoglossus
D. Medial pterygoid

132. After receiving an injection of a local anesthetic containing 2


percent lidocaine with epinephrine, the patient loses consciousness.
Which of the following is the most probable cause?
A. Acute toxicity
B. Allergic response
C. Syncope
D. Hyperventilation syndrome

130. B 131. D 132. C

79
133. How many milligrams of epinephrine are in each cartridge (1.8 cc)
of 2% lidocaine with epinephrine?
A. 0.018 mg
B. 18 mg
C. 0.036 mg
D. 36 mg

134. Local anesthetics act directly on the nerve membrane in what way?
A. Decrease the membrane's permeability to sodium
B. Increase K + flux
C. Increase the membrane's permeability to sodium
D. Increase membrane excitability

135. Phlebitis of a vein after administration of IV valium is usually


attributed to the presence of which of the following in the mixture?
A. Hydroquinone
B. Water
C. Alcohol
D. Propylene glycol

133. A 134. A 135. D


80
136. Which drug listed below is most commonly used to attain general
anesthesia?
A. Valium
B. Chloral Hydrate
C. Phenargan
D. Methohexital

137. Which of the following is a peculiar thermal alteration that occurs


during surgery in susceptible persons?
A. Malignant hypothermia
B. Heat stroke
C. Malignant hyperthermia
D. None of the above

138. The maximum allowable dose for 2% lidocaine with epinephrine is


what?
A. 2.0 mg lidocaine/per pound
B. 3.5 mg lidocaine/per pound
C. 5.0 mg lidocaine/per pound
D. 7.5 mg lidocaine/per pound

136. D 137. A 138. B

81
139. Which vein listed below is the optimum site for IV sedation for an
outpatient?
A. Median basilica
B. Median cephalic
C. Median antebrachial
D. Angular

140. What is usually the first clinical sign of mild lidocaine toxicity?
A. Itching
B. Nervousness
C. Vomiting
D. Sleepiness

141. The CNS and respiratory depressant effects of barbiturates can be


enhanced by concomitant use of:
A. Phenothiazines
B. Alcohol
C. Both of the above
D. None of the above

139. B 140. B 141. C


82
142. Productive cough, often with wheezing, is a universal factor of
which disease:
A. Emphysema
B. Congestive heart failure
C. Chronic bronchitis
D. Hepatitis

143. The amount of vasoconstrictor in 1ml or 2% lignocaine solution


with adrenaline is:
A. 0.5 mg
B. 0.05 mg
C. 0.005 mg
D. 0.0005 mg

144. The latest drug of choice in the management of trigeminal neuralgia


is:
A. Valproic acid
B. Carbamazepine
C. Diphenhydantoin
D. Carbazepine

142. C 143. C 144. D

83
145. Medical oxygen is stored in steel cylinders painted:
A. Black
B. Blue
C. White
D. Green

146. How much lignocaine is present in 2.0 ml of 2% lignocaine


solution?
A. 42.6 mg
B. 21.3 mg
C. 32.1 mg
D. 02.31 mg

147. Following a right inferior alveolar nerve block, the patient


experiences difficulty in closing the eyelid of the right side. The problem
cause is:
A. Anesthesia of the motor branch of the mandibular nerve
B. Anesthesia of the facial nerve within the parotid gland
C. Anesthesia of an aberrant branch of the cervical colli nerve
D. Anesthesia of the infra-orbital nerve because the injection was too
high

145. A 146. A 147. B

84
148. Local anesthetics produce anesthesia by:
A. Depolarizing a nerve
B. Inhibiting the influx of sodium ions through the never membrane
C. Creating negative after potentials
D. Increasing the outflow of Potassium ions through the nerve
membrane

149. When blocking a nerve containing both motor and sensory fibers,
the last functional property lost is?
A. Temperature
B. Pain
C. Proprioception
D. Touch

150. Sodium meta-bisulfite is added in local anesthetic solutions to:


A. Act as a preservative
B. Maintain the pH of the solution
C. Act as an anti-oxidant for adrenaline
D. Prevent the spontaneous dissociation of lignocaine in solution

148. B 149. C 150. C


85
151. In tracheostomy, the entry into the trachea is through the:
A. Cricoid cartilage
B. First tracheal ring
C. Second and third tracheal ring
D. Fourth and fifth tracheal ring

152. The following is true for percutaneous tracheostomy, expect:


A. No peritracheal dissection is required
B. Can be carried out at the bedside
C. A transverse incision is used
D. Treatment of choice in intubated patient

153. Trismus after an inferior alveolar nerve block most likely results
from:
A. Damage to the inferior alveolar nerve
B. Damage to the Medial pterygoid muscle
C. Myositis of the buccinator muscle
D. Accidental blockade of a major branch of the trigeminal nerve motor

151. C 152. C 153. B

86
154. The maximum safe dose of adrenaline that can be given in a local
anesthetic solution to a healthy dental out patient is:
A. 0.002 mg
B. 0.2 mg
C. 2.0 mg
D. 0.02 mg

155. The ideal rate if injection of a local anesthetic solution is:


A. 1.00 ml/ min
B. 1.50 ml/ min
C. 1.80 ml/ min
D. 2.00 ml/ min

156. The aims of pre-anesthetic medication include the following,


except:
A. Allaying apprehension
B. Decreasing oral and gastric secretions
C. Increasing reflex irritability
D. Increasing the pain threshold

154. B 155. C 156. C


87
157. Gustatory swelling is the chief symptom in:
A. Sjogren's syndrome
B. Frey's syndrome
C. Bell's palsy
D. Sialolithiasis

158. The purpose of taping the eyes shut before a patient is draped for
surgery under GA is to:
A. Prevent lacrimal secretions from contaminating the field
B. Prevent corneal abrasions
C. Limit ocular motility
D. All of the above

159. During an oral surgical procedure under general anesthesia, the


anesthetic technique best suited to avoid aspiration of blood or other
debris is:
A. Open drop
B. Nasopharyngeal intubation
C. Endotracheal intubation with cuffed tube
D. Oropharyngeal intubation

157. B 158. B 159. C

88
160. Allergic reactions in patients who receive amide type local
anesthetic solutions for tooth extractions are most likely caused by a
reaction to:
A. Lignocaine hydrochloride
B. Adrenaline bitartrate
C. Methylparaben
D. Sodium meta-bisuphite

161. Extra-oral injection for obtaining the anesthesia of the entire second
division of the fifth cranial nerve is given in\around the:
A. Infraorbital foramen
B. Foramen ovale
C. Pterygopalatine fissure
D. Foramen spinosum

162. After receiving a posterior superior alveolar block the patient


experiences:
A. Numbness of the molar teeth
B. Numbness of the upper lip
C. Numbness of the side of the face
D. Few subjective signs of anesthesia

160. C 161. C 162. D

89
163. Which of the following drug is used to manage an acute urticarial
reaction after administration of a local anesthetic?
A. Intravenous injection of aminophylline
B. Inhalation of aromatic spirits of ammonia
C. Intravenous injection if hydrocortisone methyl acetate
D. Intravenous injection of diphenhydramine

164. The most common postoperative complication of outpatient general


anesthesia is:
A. Syncope
B. Nausea
C. Atelectasis
D. Shivering

165. The piercing of the infra orbital nerve with a 26-gauge needle while
giving a block in the canine fossa produces:
A. Lancinating pain in the lower eyelid
B. A mild, temporary paresthesia of the upper lip
C. Destruction of the nerve
D. Hematoma formation in the region

163. D 164. B 165. B

90
166. Vascular compression of the gasserin ganglion by the following
blood vessel is thought to be responsible for tic:
A. Superior cerebellar artery
B. Superior cerebral artery
C. Middle cerebral artery
D. Antero-inferior cerebellar artery

167. A patient with upper motor lesions of the facial nerve presents with
the following clinical appearance except:
A. Sagging of the corner of the mouth
B. Flattening of the nasolabial furrow
C. Loss of eyebrow and forehead wrinkles
D. Incompetent lip seal

168. Endotracheal intubation is only possible in the following stage of


general anesthesia:
A. stage I
B. stage III, Plane 1
C. stage III, Plane 2
D. stage III, Plane 3

166. A 167. C 168. C

91
169. The important sign a patient exhibits when the correct level of
sedation is reached while using diazepam sedation is?
A. Blurring of vision
B. Slurring of speech
C. Loss of gag reflex
D. 50% ptosis of the eyelids

170. In general anesthesia the loss of eyelash reflex


A. stage I
B. stage II
C. stage III, Plane 1
D. stage III, Plane 4

171. Guedel's criteria for classification of the depth of general anesthesia


are based on:
A. Respiration, eyeball movement and presence and absence of various
reflexes
B. Response to skin incision
C. Pharyngeal and laryngeal reflexes
D. Respiration, eyelid reflex and secretion of tears

169. D 170. C 171. A

92
172. According to the gate-control theory of pain, impulses traveling in
the A fibers tend to
A. Inhibit the substansia geletinosa cells thereby blocking the pain
stimuli transmitted by the C fibers
B. Excite the substansia gelatinosa cells thereby blocking the pain
stimuli transmitted by the nonmyelinated fibers
C. Be associated with pain of inflammatory origin
D. Bypass the central control process

173. The "burning" felt at the site of venipuncture after administration of


diazepam can be attributed to:
A. Propylene glycol in the mixture
B. Benzoic acid in the mixture
C. Use of a blunt or large diameter needle
D. Poor venipuncture technique

174. Nitrous oxide-oxygen inhalation sedation is contraindicated in


patients with:
A. Nasal obstruction
B. Emotional instability
C. Emphysema
D. All of the above

172. B 173. A 174. D


93
175. Dissociative analgesia is produced using following drug
combination:
A. Ketamine and scopolamine
B. Fentanyl and droperidol
C. Pentobarbital and meperidine
D. Diazepam and atropine

176. The minimum distance a nerve should be covered by the anesthetic


solution to ensure thorough blockade of the nerve impulse, is:
A. 3-5 mm
B. 5-8 mm
C. 8-10 mm
D. 10-14 mm

177. The absolute contraindication to the use of a local anesthetic


solution with epinephrine is in a patient with:
A. Uncontrolled hypertension
B. Untreated hyperthyroidism
C. Diabetes mellitus
D. Ventricular fibrillation

175. A 176. C 177. B

94
178. The pH of the tissues does not affect the effective- ness of this
anesthetic agent:
A. Bupivacaine
B. Procaine
C. Benzocaine
D. Mepivacaine

179. The most potent vasodilator local anesthetic agent


A. Cocaine
B. Lignocaine
C. Tetracaine
D. Procaine

180. The following incision is used to gain access to the medial surface
of the mandibular ramus, for inferior alveolar neurectomy:
A. Risdon's incision
B. Ginwall's incision
C. Bayonet incision
D. Crow's foot incision

178. C 179. D 180. B


95
181. Of the following which is most suitable anesthesia for the
extraction of a deciduous molar?
A. Inferior alveolar nerve block
B. Surface anesthesia
C. Local infiltration
D. None of the above

182. Most reactions following injection of LA are caused


A. Allergy
B. Anxiety
C. Toxicity
D. Over dosage

183. Hematoma formation after the posterior superior alveolar nerve


block is due to the damage to:
A. Internal maxillary artery
B. Small capillaries and arterioles in the region
C. Pterygoid venous plexus
D. Facial artery

181. A 182. B 183. C


96
184. Two minutes following cessation of a two-hour treatment session
using 80% N20 and 20% 02 and local anesthesia. The patient becomes
cyanotic and tachycardiac most probable cause is:
A. Diffusion hypoxia
B. Over oxygenation
C. Under oxygenation
D. Malignant hyperthermia

185. Level of analgesia is best monitored by:


A. Eye movements and absence of corneal reflex
B. Eye movements and absence of conjunctival reflex
C. Respiration, rate, rhythm and type of respiratory movements
D. Verbal response

186. Among the following which is the optimum site for intravenous
sedation in an outpatient?
A. Jugular vein
B. Median cephalic vein
C. Dorsal vein back of the hand
D. Popliteal vein

184. A 185. D 186. B

97
187. Which respiratory conditions is most alarming during patient
sedation in dental hospital?
A. Tachypnoea
B. Dyspnoea
C. Apnoea
D. Hyperpnoea

188. The common anesthetic complication occurring within first day


after surgery under GA is:
A. Cardiac failure
B. Renal failure
C. Atelectasis
D. Hypotension

189. According to Gate control theory of pain, large fiber impulses tend
to:
A. By pass the central control process
B. Inhibit the effect of painful stimuli transmitted by small fibers
C. Transmit severe .chronic pain only
D. Potentiate the effect of painful stimuli

187. C 188. C 189. B

98
190. In case of mild lidocaine toxicity, the first clinical sign is:
A. Nervousness
B. Lethargy
C. Convulsions
D. Tachycardia

191. Which of the following is the cause of allergies to local anesthetics?


A. Slow elimination
B. Improper technique
C. Slow detoxification
D. Antigen and antibody reactions

192. The following drugs potentiates the action of sedative drugs:


A. Phenothiazines
B. Propanol
C. Methyldopa
D. Digitalis

190. A 191. D 192. A

99
193. Each cartridge contains how many ml. of anesthetic solution:
A. 0.9 ml
B. 1.2 ml
C. 1.8 ml
D. 2.2 ml

194. How many mg of 2% lidocaine present in the cartridge?


A. 36 mg
B. 18 mg
C. 180 mg
D. 360 mg

195. How many mg of adrenaline are in each cartridge of 2% lignocaine


with adrenaline:
A. 0.36 mg
B. 0.036 mg
C. 0.018 mg
D. 0.15 mg

193. C 194. A 195. C

100
196. Among the following in which patient GA is contraindicated?
A. An adult male over 65 years of age
B. Patient with acute respiratory infection
C. Child under 6 years of age
D. Patient with wheel compensated heart disease

197. The following two laboratory tests should be considered before


deciding whether to use general anesthesia or not?
A. Total WBC count and prothrombin time
B. Total WBC count and urine analysis
C. Bleeding time and clotting time
D. Complete blood count and urine analysis

198. Nerve membrane stabilization action of local anesthetic agents is


due to:
A. Increasing conductance of potassium ions
B. Creating negative after potentials
C. Preventing conductance of Na ions from the exterior of the nerve to
the interior
D. Preventing conductance of Na ions from the interior to the exterior of
the nerve

196. B 197. D 198. C

101
199. Thirty seconds after the injection of 1.8 ml of local anesthetic
containing 2% lidocaine with adrenaline patient loses consciousness.
The most probable diagnosis is:
A. Toxic reaction to the adrenaline
B. Cerebral hypoxia
C. Tachycardia
D. Toxic reaction to the lidocaine

200. In which conditions N O - O sedation in contraindicated?


A. Emphysema
B. Upper respiratory obstruction
C. Emotional instability
D. All of the above

201. Members of the sterile professional team include


A. Surgeon, assistants and scrub nurse
B. Surgeon, scrub nurse, and circulating nurse
C. Surgeon, scrub nurse, circulating nurse and anesthesiologist
D. A and B

199. B 200. D 201. A


102
202. One wants to anesthetize the inferior alveolar nerve at the point at
which it exists from the mandibular canal, so the injection site most
often located in
A. Between first and second molars inferior to the root apices
B. Between first and second molar superior to root apices
C. Between first and second premolars inferior to root apices
D. Between first and second premolars superior to root apices

203. Accidental intravenous injection of LA which contains a


vasoconstrictor may cause:
A. Convulsions and palpitations
B. Unconsciousness and depressed respiration
C. Increased respiration
D. Any of the above may occur

204. Compared to inhalation, IV sedation:


A. Produces more; predictable amnesia
B. May produce more serious complications
C. May result in deeper sedation
D. All of the above are correct

202. C 203. D 204. D

103
205. Among the following which is least likely to block reflexes for
surgery completely when used alone?
A. Nitrous oxide
B. Halothane
C. Ethane
D. Thiopentone sodium

206. Likely cause of trismus after inferior alveolar block anesthesia is:
A. Damage to medial pterygoid muscle during injection
B. Myositis of the external pterygoid muscle
C. Submandibular cellulitis
D. Stretching of the pterygomandibular raphe

207. Most common cause of death occurring under GA is due to:


A. Reflex cardiac standstill
B. Over dosage of anesthetic agent
C. Excessive carotid sinus stimulation
D. Airway obstruction with improper ventilation

205. A 206. A 207. D

104
208. Three common symptoms indicating the correct level of sedation
after diazepam administration is:
A. Blurring of vision, slurring of speed and loss of GAG reflex
B. 50% ptosis eyelids, blurring of vision, slurring of speech
C. Paresthesia of lips, tongue and fingers
D. Sweating over the face, itching of the nose and loss of gag reflex

209. Preferable patient position in recovery room after ambulatory


general anesthesia is:
A. Supine
B. Sitting
C. Prone
D. Lateral

210. Among the following which nerve innervates the buccal gingiva of
the maxillary second premolar?
A. Anterior palatine nerve
B. Buccal nerve
C. Middle superior alveolar nerve
D. Posterior palatine nerve

208. B 209. D 210. C

105
211. Paresthesia during a mandibular nerve block is due to:
A. Sodium conductance through the nerve
B. Acid pH of the anesthetic solution
C. Contact of the needle with the nerve trunk
D. Poor technique

212. Of the following in which condition local anesthesia is ineffective?


A. Edema
B. Localized infection
C. Hematoma
D. Anemia

213. The following anatomical land marks are useful in the closed
mouth approach of mandibular nerve block except:
A. Occlusal plane of occluding teeth
B. Anterior border of ramus
C. Mucogingival junction of the mandibular teeth
D. A and B

211. C 212. B 213. C

106
214. A contraindication to local infiltration technique:
A. Hypertension
B, Diabetic patient in whose urine sugar is present
C. Infection in the arch
D. Kidney problems

215. The purpose of gas filled bubbles in anesthetic cartridge is:


A. Indicate the level of anesthetic agent
B. Prevent deterioration of the vasoconstrictor
C. Formed during the manufacturing process
D. A and C

216. Anterior palatine nerve is anaesthetized by anterior palatine nerve


block. The subjective symptoms include:
A. Feeling of numbness in posterior palate
B. Feeling of numbness in the anterior part of the palate near the incisors
C. Feeling of numbness of whole palate
D. Little or no subjective symptoms

214. C 215. B 216. A

107
217. The primary site of local anesthetic action is:
A. Medulla oblongata
B. Substantia gelatinous
C. Nerve membrane
D. Nodes of Ranvier

218. Which is best monitor of the level of analgesia?


A. Eye movements
B. Respiratory movements
C. Muscle tone
D. Verbal response

219. When comparing to the nonmyelinated nerve, myelinated nerve


requires:
A. Higher concentration of anesthetic agent
B. Lower concentration of anesthetic agent
C. Same concentration of anesthetic agent
D. None of the above because no correlation

217. C 218. D 219. A

108
220. Local anesthetic cartridges contain small bubbles.
These bubbles contain:
A. Oxygen gas
B. Carbon dioxide gas
C. Nitrogen gas
D. Carbon monoxide gas

221. Which of the following statements is true?


A. More concentration of H+ ions in an area potentiate anesthetic agent
action
B. Less concentration of H+ ions in an area potentiate anesthetic agent
action
C. A little is known about the relation between H+ ion concentration and
anesthetic activity
D. No correlation between H+ ion concentration and anesthetic activity

222. A surgical procedure required bilateral block. A bi- lateral inferior


alveolar nerve block is:
A. Not contraindicated
B. Should rarely be performed
C. May cause space infection
D. Dangerous because patient may swallow the tongue

220. C 221. B 222. A

109
223. Risk of death in more in some patients with the use of local
anesthesia with adrenaline. Which of the following group of patients are
called as walking time bombs?
A. Cocaine abusers
B. Hypertensive
C. Hyperthyroidism patients
D. Patients under MAOI (MAO inhibitors) treatment

224. Subjective symptom of buccal nerve block is:


A. Numbness and tingling sensation of the buccal mucosa
B. Numbness of the lingual mucosa near molars
C. No subjective symptoms
D. A and B

225. Ester group of local anesthetics mainly inactivated


A. Hydroxylation
B. Oxidation
C. Hydrolysis
D. None of the above

223. A 224. C 225. C


110
226. Among the following, which condition is a contraindication to the
use of local anesthetic agent?
A. Parkinson's disease
B. Liver damage
C. Pregnancy (3rd trimester)
D. Hypersensitivity to the drug

227. Amide type of local anesthetic agents undergo biotransformation


primarily in the:
A. Kidney
B. Liver
C. Plasma
D. No biotransformation occurs excreted in all unaltered form

228. Ester type local anesthetic agents undergo biotransformation in the:


A. Kidney
B. Liver
C. Plasma
D. B and C

226. D 227. B 228. D


111
229. Of the following in which condition general anesthesia is
contraindicated?
A. Hemoglobinopathy
B. Allergy
C. Diabetes mellitus
D. All of the above

230. The following is least toxic of all local anesthetics:


A. Tetracaine
B. Mepivacaine
C. Prilocaine
D. 2-chloro procaine

234. Cocaine is:


A. Vasodilator in action
B. Vasoconstrictor in action
C. Synthetic
D. Synthetic and vasodilator in action

229. A 230. D 231. B

112
232. Adrenaline is added to local anesthetic agents because it:
A. Potentiate the action of all local anesthetic agents
B. Increases the rate of destruction of the local anesthetic agent
C. Decreases the rate of absorption of the local anesthetic at the injection
site
D. Prevents deterioration of the anesthetic solution

233. Sedation by which of the following routes can be reversed rapidly?


A. Intravenous
B. oral
C. Inhalation
D. Intramuscular

234. Only nitrogen oxide alone is not used as a general anesthetic’s


agent because of the:
A. Poor analgesic properties
B. Expense of the agent and its explosive hazard
C. Difficulty in maintaining an adequate oxygen concentration
D. Hepatotoxic nature

232. C 233. C 234. C


113
235. After the general anesthesia, vomiting may occur.
The immediate postoperative complication following aspiration of liquid
vomitus into trachea and bronchi is:
A. Lung abscess
B. Bronchitis
C. Pleurisy
D. Atelectasis

236. Which of the following side effects is seen commonly with the
administration of nitrous oxide and oxygen?
A. Hallucinations and dreams
B. Tachycardia
C. Tremors
D. Nausea

237. In which stage of general anesthesia surgery is performed?


A. Stage I
C. stage III
B. stage II
D. stage IV

235. B 236. D 237. C

114
238. The following areas are anaesthetized by the anterior palatine nerve
block:
A. Part of the upper lip, lateral surface of nose and lower eyelid
B. Posterior portion of hard palate and overlying tissues up to the first
premolar
C. Incisors canine and first premolar
D. Both hard and soft palates completely

239. The following deep bony landmarks is important in performing a


block of the II and Ill divisions of the trigeminal nerve from the lateral
approach:
A. Temporal surface of sphenoid
B. Perpendicular plate of the palatine bone
C. Lateral plate of the pterygoid process
D. Styloid process

240. For a therapeutic procedure bilateral mandibular nerve block is


required. Bilateral mandibular nerve block is:
A. Should perform rarely
B. Not contraindicated
C. Dangerous because tongue may fall back and patient
D. May cause severe trismus due to space infection

238. B 239. C 240. B


115
241. Facial paralysis after inferior alveolar nerve block is due to:
A. Injection into the parotid gland
B. Aberrant facial nerve
C. Sympathetic stimulation
D. All of the above

242. Among the following items in the past medical history, which
would most affect the choice of local anesthetic agent?
A. RHD
B. Hyperthyroidism
C. Adrenal insufficiency
D. Asthma

243. The main function of atropine in preoperative medication is to:


A. Depress gastro intestinal mobility and relieve anxiety
B. Reduce the salivary secretion and sedation
C. Reduce secretions and depress vagal reflex
D. Reduce bronchial secretions and enhance vagal reflex

241. D 242. B 243. C

116
244. In the extraoral infraorbital nerve block the following nerve is not
involved:
A. Anterior superior alveolar
B. Middle superior alveolar
C. Sphenopalatine
D. Lateral nasal
245. After giving an inferior alveolar nerve block patient develops
paralysis of the muscles of the forehead, eyelids and upper and lower
lips on the same side of the face. This is probably due to:
A. Anesthesia of the motor branches of the mandibular nerve
B. Diffusion of the anesthetic solution involving the ophthalmic division
of the trigeminal
C. Injection of the solution into the capsule of the parotid gland and
consequent block of the facial nerve
D. Diffusion of the solution through the inferior orbital fissure and
anaesthetized the oculomotor nerve

246. Among the following which of the following are potential


immediate post extraction complications for a surgical patient treated
under GA?
A. Respiratory embarrassment
B. Circulator depression
C. Aspiratory of blood or gastric contents
D. All of the above

244. C 245. C 246. D


117
247. The fate of nitrous oxide when administered for analgesia is:
A. Detoxified in liver
B. Exhaled by lungs
C. Excreted by kidney
D. Chemically changed into NO and CO

248. Of the following in which nitrous oxide-oxygen sedation is


contraindicated?
A. Mental retardation
B. Sickle cell anemia
C. History of congestive heart failure 3 months previously
D. None of the above

249. The sensation of "tissue tearing" during inferior alveolar nerve


block indicates:
A. Passage through buccinator muscle
B. Passage through an area of infection
C. Barb on the needle
D. Large bore needle such as 19 no. needle

247. B 248. D 249. C


118
250. Of the following which is main cause of toxic manifestations
following injection of a vasoconstrictor is?
A. Intramuscular injection
B. Intravascular injection
C. Injection into a nerve trunk
D. Allergic reaction

251. Of the following which is most common postoperative


complication of outpatient of general anesthesia?
A. Pneumonia
B. Lung abscess
C. Nausea
D. Atelectasis

252. In a dental clinic after the administration of local anesthesia


depression of respiration occurred. It is a manifestation of:
A. Trauma to a nerve trunk by the syringe needle
B. Toxic effects of the solution
C. Puncture of a blood vessel
D. Use of an isotonic solution

250. B 251. C 252. B

119
253. Among the following which is most common complication
associated with nitrous oxide sedation?
A. Vomitus aspiration
B. Behavioral problem
C. Respiratory depression
D. Vomiting

254. Among the following which is advantage of oral administration of


drugs to achieve conscious sedation?
A. It is highly reliable in producing sedation
B. It is easiest to administer
C. Over dosage is unlikely
D. None of the above

255. Which of the following is immediate post-operative complication


of aspiration of liquid vomitus into the trachea and bronchi?
A. Atelectasis
B. Pneumothorax
C. Chemical pneumonia
D. Pleurisy

253. B 254. B 255. C

120
256. The following factors play a role and cause variations in
susceptibility of sensory nerve fibers to local anesthesia:
A. Diameter of the nerve fiber
B. Distance between nodes or Ranvier
C. Length of nerve fiber
D. All of the above

257. Which of the following are contraindications to nitrous oxide


oxygen sedation?
A. Emotional instability
B. Upper respiratory tract obstruction
C. Emphysema
D. All of the above

258. Which of the following would be indicated if one expected a long


surgical procedure and a general anesthetic to be administrated?
A. Sodium thiopental
B. Vinyl ether, open drop method
C. Intubation with individually selected anesthetic agent
D. None of the above

256. D 257. D 258. C

121
259. Intraoral injection to block second division of the trigeminal nerve
should be given in the:
A. Foramen spinosum
B. Foramen ovale
C. Pterygopalatine fossa
D. Infraorbital foramen

260. Which is the main route of elimination of nitrous oxide?


A. Kidneys
B. Liver
C. Lungs
D. Skin

261. Frequently which of the following in amide type local anesthesia


causes allergic reaction?
A. Lignocaine hydrochloride
B. Methyl paraben
C. Adrenaline
D. Contaminants in the solution

259. C 260. C 261. B


122
262. Usually hydrolysis of amide type of local anesthetic salts is
facilitated by:
A. Tissue pH above 7.0
B. Tissue pH between 5.0 and 7.0
C. Tissue pH below 5.0
D. There is no relationship between pH and hydrolysis in amide type
local anesthesia

263. Respiration in stage IV general anesthesia is characterized by:


A. Thoracoabdominal in nature
B. Abdominothoracic in nature
C. Deep and irregular
D. Absent

264. A patient is allergic to PABA Most probably he will also be allergic


to:
A. Lignocaine
B. Carbocaine
C. Prilocaine
D. Tetracaine

262. A 263. D 264. D

123
265. Nitrous oxide cylinders are usually in color:
A. Black
B. White
C. Blue
D. Red

266. Which of the following should be included in the preoperative


orders?
A. NPO from mid night
B. Take written informed consent
C. Void on call to the OR
D. All of the above

267. In which stage of anesthesia patient becomes hyperactive?


A. stage I
C. stage III
B. stage II
D. stage IV

265. C 266. D 267. B

124
268. Toxic dose of lignocaine with adrenaline is:
A. 5 mg/kg body weight
B. 10 mg/kg body weight
C. 15 mg/kg body weight
D. 20 mg/kg body weight

269. In gate control therapy of pain. Pain is mainly modulated by:


A. Substantia gelatinosa
B. 'B' fibers
C. 'C' fibers
D. Transmission system

270. Morphine scopalamine premedication produces:


A. Amnesia and decreased salivation
B. Psychic sedation
C. Additive effects with anesthetics
D. All of the above

268. A 269. A 270. D

125
271. The following which produces an unconscious state?
A. Regional analgesia
B. Nitrous oxide and oxygen
C. Neuroleptic analgesia
D. None of the above

272. Maximum recommended dosage of lignocaine with adrenaline


subcutaneously is:
A. 100 mg
B. 30 mg
C. 500 mg
D. 700 mg

273. Among the following which stage of anesthesia describes the level
of conscious sedation?
A. Stage I
B. Stage II
C. stage III plane 1
D. stage III plane 2

271. C 272. C 273. A


126
274. The initial stage of paralysis of facial nerve:
A. Tongue deviate to the same side on protrusion
B. Tongue deviate to the opposite side on protrusion
C. No deviation of the tongue
D. None of the above

275. "Tic douloureux" treatment includes:


A. Dilantin sodium in all cases
B. Carbamezapine in all cases
C. Mandibular nerve block in all cases
D. Trichloroethylene in all cases

276. Risus sardonicus is one the sign of the tetanus.


This involves:
A. Spasm of the glottis causing altered voice
B. Spasm of the facial muscle causing a fixed smile
C. Spasm of the pterygoid muscles causing trismus
D. Spasm of the chest wall causing difficulty in breathing

274. A 275. B 276. B

127
277. Which of the following structure is associated with
Bell's palsy"?
A. Submandibular gland
B. Seventh cranial nerve
C. Temporomandibular joint
D. Glossopharyngeal nerve

278. A decrease in the pH of interstitial fluid will:


A. Increase the effectiveness of a local anesthetic block
B. Have no effect on the effectiveness of a local anesthetic block
C. Decrease the effectiveness of the LA solution
D. Initially increases than decreases the effectiveness of LA solution

279. Maximum recommended dosage of lignocaine and adrenaline


subcutaneously is:
A. 100 mg
B. 300 mg
C. 500 mg
D. 700 mg

277. B 278. C 279. C

128
280. The following clinical disease process affect the
TMJ directly:
A. Ankylosis
B. Arthritis
C. Dislocation
D. All of the above

281. Best treatment for a small opening of a disease free maxillary sinus
is:
A. Not treating socket but advising the patient proper home care
B. Packing tile socket with a hemostatic agent to encourage clotting
C. Leaving the socket undisturbed but prescribing nasal vasoconstrictor
and antibiotics
D. Employ primary closure of the socket using sutures.

282. Among the following which factors influence the pain perception
threshold?
A. Emotional state.
B. Apprehension and fear
C. Age
D. All of the above

280. D 281. A 282. D

129
283. The drug of choice to counteract the central nervous system
stimulation caused by accidental IV injection of local anesthesia:
A. Adrenaline
B. Pentobarbital
C. Caffeine
D. None of the above

284. Under GA, hemolysis resulting from a transfusion reaction will


exhibit which of the following symptoms?
A. Chills, fever, dyspnoea, headache, pain in the back
B. Hypotension, skin flush, tachycardia, abnormal hemorrhage,
hyperpyrexia
C. Urticaria, angioneurotic edema, asthma
D. Utricaria and pruritus

285. Following group of drugs used for premedication produce a


detached serenity without clouding consciousness. Can you identify
them?
A. Antiemetics
B. Ataractics
C. Analeptics
D. Psychodissociates

283. B 284. B 285. B


130
286. Among the following which results in sudden swelling in the area
of a dental injection?
A. Drug incompatibility.
B. Paravascular injection
C. Infection
D. Vascular penetration

287. Among the following which has least hypnotic action?


A. Chloral hydrate
B. Pentobarbital
C. Phenobarbital
D. Codeine

288. Which of the following is not a transmitter substance and it has no


role in impulse transmission?
A. Dopamine
B. Noradrenaline
C. Succinyl choline
D. GABA

286. D 287. D 288. C


131
289. Pre-anesthetic medication commonly used to achieve the following
goals except:
A. Decreasing nervous irritability
B. Decreasing secretions
C. Providing analgesia
D. Desensitizing tile respiratory center to CO2

290. When is the appropriate time to administer an analgesic to control


postoperative pain?
A. Only after the return of sensation
B. When pain becomes moderate to severe
C. Before the anesthetic effect wears off
D. None of the above

291. Sedation by which of the following routes can be reversed most


rapidly?
A. Inhalation
B. Intravenous
C. Intramuscular
D. Oral

289. D 290. C 291. A

132
292. Among the following which drug is drug of choice in management
of the acute allergic reaction involving hypotension?
A. Aminophylline
B. Dexamethasone
C. Diphenydramine
D. Adrenaline

293. One cannot determine the level of N20 per tank with
N20 pressure gauge on a sedation machine because:
A. The gas is explosive
B. The pressure drops in a sporadic fashion
C. The pressure remains constant until the tank
D. It is nearly empty

294. Toxic effects seen after administration of a local anesthetic with


epinephrine are probably due to anesthetic agent if the following sign is
present:
A. Tachypnoea
B. Tachycardia
C. Drowsiness
D. Sweating

292. D 293. C 294. C

133
295. While receiving IV diazepam, patient has upper eyelid ptosis
(verrill's sign) the dentist should immediately
A. Assist respiration
B. Consider the patient is adequately sedated
C. Place the patient in Trcndelenberg position.
D. Consider the anesthesia is not complete and administer one more
dose of diazepam

296. Among the following which way best protect from the toxic aspects
of a local anesthetic?
A. Take a thorough medical history
B. Have oxygen available
C. Use an aspirating technique
D. Use the lowest possible concentration of the local anesthetics

297. Trigeminal neuralgia is characterized by:


A. Paralysis of the one side of the face due to paralysis of facial nerve
B. Dull pain when pressure is applied over the affected area (Trigger
zones)
C. Sharp pain when pressure is applied to the affected area
D. Prolonged episodes of the pain which is constant in nature

295. B 296. C 297. C

134
Chapter 3: Sterilization and Instruments
1. Scrub technique refers a method of scrubbing the:
A. Surgical instruments after a contaminated case
B. Patient body surface before placing surgical incision to decrease
bacterial flora.
C. Operating room.
D. Operator's hands and forearms before donning gown and gloves.

2. The sterilizing units should be monitored at which of the following


intervals?
A. Daily
B. Weekly
C. Bi weekly
D. Monthly

3. Scrub technique refers to the proper method of:


A. Washing hands and arms before gloving for surgery
B. Cleaning the operating room after a contaminated case
C. Washing the patient's body surface prior to incision
D. Washing the instruments prior to sterilization

1. D 2. B 3. A

135
4. Bi-beveled chisels are used primarily to:
A. Remove bone
B. Split teeth
C. Make purchase points
D. All of the above

5. Chromic catgut sutures are packed in alcohol to prevent:


A. Atmospheric contamination
B. Enzymatic degradation
C. Loss of tensile strength
D. All of the above

6. Catgut sutures are packed in this type of alcohol:


A. Ethyl alcohol
B. Methyl alcohol
C. Isopropyl alcohol
D. Absolute alcohol

4. B 5. B 6. C

136
7. The typical "railroad track" scar is the result of:
A. Inadequate approximation of wound edge during suturing
B. Delayed removal of sutures
C. Delayed wound closure
D. Excessive fibrous tissue formation

8. The "Wicking" effect will be more commonly seen when the


following suture material is used:
A. Nylon
B. Plain catgut
C. Silk
D. Polypropylene

9. Steam autoclaving:
A. Kills microorganisms by RNA and DNA breakdown
B. Destroys cotton or cloth products
C. Is non corrosive to carbon steel instruments?
D. Utilizes 249.80F at 5 psi and 15 to 30 minutes

7. C 8. C 9. A

137
10. Glutaraldehyde is activated to achieve its full antimicrobial
capability by adding:
A. Sodium sulphate
B. Sodium hypochlorite
C. Sodium bicarbonate
D. Sodium hydroxide

11. A vertical mattress suture is used in wound Closure:


A. To prevent ischemic necrosis of wound edges in areas with poor
blood supply
B. To close flaps that are one tissue layer thick
C. To Evert the wound margins
D. To provide a water tight closure

12. Single use hypodermic needles have colored hubs for ease of
identification of the needle gauge. 25'G needle hubs are:
A. Blue
B. Red
C. Yellow
D. Green

10. C 11. A 12. A

138
13. To reduce thrombophlebitis in patients on parenteral fluid therapy,
IV catheter made up of the following materials are used:
A. Silicone
B. Stainless steel
C. Teflon
D. Plastic

14. The main objective for efficacy of sterilization practice is destruction


of:
A. Cocci
B. Viruses
C. Bacteria
D. Spores

15. Most postoperative infection results from:


A. Faulty surgical technique
B. Inadequate asepsis and disinfection
C. None of the above
D. Both of the above

13. C 14. D 15. D


139
16. _____________ is a chemical, applied to living tissues, such as skin
or mucous membrane to reduce the number of microorganisms present,
by inhibition of their activity or by destruction.
A. Antiseptic
B. Disinfectant
C. Antiseptic and disinfectant
D. Cleaning agent

17. _____________ is the process of destruction or removal of all


microbial forms:
A. Antisepsis
B. Sterilization
C. Disinfection
D. Cleaning

18. The mechanism by which the disinfectant and anti-sepsis act on


microorganisms?
A. Coagulation of bacterial protein
B. Alteration in properties of bacterial wall
C. Binding of sulphydryl groups
D. All of the above

16. A 17. B 18. D

140
19. Alcohol shows maximum antiseptic activity at:
A. 10%
B. 50%
C. 70%
D. 99%

20. In autoclave at 15 psi pressure and 121 degree C temperature time


required to kill all living organisms is:
A. 3 min
B. 6 min
C. 15 min
D. 60 min

21. Organisms used for test of efficiency for heat sterilization


A. S sanguis
B. T pallidum
C. Beta hemolytic streptococci
D. Bacillus sterothermophyllus

19. C 20. C 21. D

141
22. Basic action of Dry Heat sterilization is
A. Dehydration
B. Oxidation
C. Reduction
D. A and B

23. Which of the following relation of temperature and time is for dry
heat sterilization?
A. 60C - 2hrs
B. 100 – 1, 1/2hrs
C. 170C - 1hr
D. 200C - 15mins

24. Glass bead sterilizer is used for sterilization of:


A. Glass instruments
B. Plastic instruments
C. Endodontic files and burs
D. Long metal instruments

22. D 23. C 24. C

142
25. The operation theatres are disinfected by:
A. Alcohol
B. Heat sterilization
C. Fumigation
D. None of the above

26. Excretions and secretions should be disposed by:


A. Buried in deep pit'
B. Powder and lime
C. Incineration
D. Deep burial

27. Laboratory specimen should be kept in:


A. 10% formalin
B. 10% phenol
C. 100% alcohol
D. 0.2% Chlorhexidine

25. C 26. B 27. A

143
28. Plaque from gingival crevice contains around how many bacteria:
A. 1000
B. 2500
C. 2.5 x 1011
D. 2.5 x 1017

29. At birth oral cavity contains:


A. S. sanguis
B. S. hemolyticus
C. M. tuberculae
D. Oral cavity is sterile

30. Most commonly detected organisms in mouth of new born is:


A. S. sanguis
B. T. pallidum
C. S. salivarius
D. C. albicans

28. C 29. D 30. C

144
31. Shape of treponema is:
A. Spiral
B. Cocci
C. Bacilli
D. Dumb-bell

32. Candida albicans is:


A. Virus
B. Bacteria
C. Fungi
D. Yeast

33. A hot air oven cannot be used for sterilization of:


A. Glassware
B. Talcum powder
C. Cotton
D. Culture media

31. A 32. D 33. D

145
34. A gas compound used in special autoclaves for sterilizing materials
that cannot be taken to higher temperatures is:
A. Water vapor
B. Ethylene oxide
C. Hydrogen
D. Cyanide

35. Criteria that have been used to distinguish pathogenic from non-
pathogenic bacteria are known as:
A. Gram stains
B. Koch's postulates
C. Randolph's rules
D. Pasteur's principles

34. B 35. B
146
Chapter 4: Exodontia \ Impaction
1. A patient develops facial edema one day after removal of a tooth.
Instructions to the patient should be to use:
A. Warm, wet applications outside and inside the mouth
B. Cold applications outside and inside the mouth
C. Intraoral cold applications only
D. Intraoral hot applications only

2. Of the following the most common postoperative complication after


surgical removal of mandibular teeth
A. Myositis
B. Paresthesia
C. Loss of the blood clot
D. Postoperative hemorrhage

3. The most serious complication which may occur following surgery


about the maxillary canine is:
A. Iritis
B. Cellulitis
C. Localization of pus
D. Cavernous sinus thrombosis

1. A 2. C 3. D
147
4. The elevators used in exodontia are functionally:
A. Class I levers only
B. Class I and II levers
C. Class Ill levers only
D. Only wedge-shaped

5. To extract a tooth the following type of grip is most desirable:


A. The whole of the inner surface of the forceps blade should fit the root
surface
B. Only edges of the forceps blade should contact the tooth (Two-point
contact)
C. Only single lever contact between forceps blade and root surface (one
point contact)
D. Grip plays little role in extraction

6. Exodontia elevators are based on principle of:


A. Levers
B. wedge
C. Wheel and axle
D. All of the above

4. B 5. A 6. D

148
7. While removing mandibular 3rd molar which part of bone should be
used as a fulcrum?
A. Lingual cortical bone
B. Mesial inter-radicular bone
C. Buccal alveolar bone
D. Distal surface of adjacent crown

8. Mechanical advantage would be maximum for an elevator when:


A. Effort arm is greater than resistance arm
B, Resistance arm is greater than effort arm
C. When fulcrum is in the center
D. When fulcrum is near the point of effort

9. When maxillary lateral incisor has to be extracted the first movement


of forceps should be:
A. Labially then apically
B. Palatally then apically
C. Labially or palatally
D. Apically then palatally

7. C 8. A 9. D
149
10. Mead forceps are used for extraction of:
A. Maxillary molars
B. Mandibular incisors
C. Maxillary canines
D. Maxillary premolars

11. To extract mandibular premolars the first movement of forceps


should be:
A. Apical and labial
B. Apical and lingual
C. Apical and mesial
D. Labial and no apical movement

12. When maxillary and mandibular teeth are to be extracted


simultaneously the order of extraction should be:
A. Maxillary teeth followed by mandibular teeth
B. Mandibular teeth followed by maxillary teeth
C. It does not matter which teeth are removed first
D. For anterior region mandibular first and for posterior region maxillary
first

10. B 11. B 12. A

150
13. When simultaneous extractions have to be carried out in one
maxillary segment the order of extraction should be:
A. Incisors, canine, premolars, molars
B. 3rd molar, second molar, 2nd premolar, 1st molar,
1st premolar, incisors, canine
C. Molars, premolars, canine, incisors
D. Molars, premolars, incisors, canine.

14. The Stobis method of extraction is indicated when:


A. Two adjacent teeth have to be removed
B. When central incisor is to be extracted
C. When isolated molar is to be extracted and adjacent teeth are absent
D. Upper and lower 3rd molars have to be extracted simultaneously

15. 'White line' described by George Winter is a line


A. Along occlusal surfaces of erupted mandibular molars extending to
impacted third molar region
B. From crest of bone lying distal to third molar to crest of the
interdental septum between 1st and 2nd molar
C. From perpendicular to 'amber line'
D. From perpendicular to 'red line'

13. B 14. A 15. A

151
16. 'Red Line' as described by George Winter is demarcated as a line:
A. Drawn from bone distal to third molar to crest of the interdental
septum between 1st and 2nd molar
B. Which is a perpendicular dropped from 'amber line' to point of
application of elevator
C. Which is a perpendicular dropped from 'white line' to point of
application of elevator
D. None of the above

17. If in a mesioangular mandibular impaction there is apical notch


visible on a radiograph the tooth should:
A. Not be removed
B. Be removed by lingual split technique
C. Be removed by splitting the tooth
D. None of the above

18. Which type of mandibular impactions are most commonly associated


with 'apical notch' formation:
A. Mesioangular
B. Distoangular
C. Vertical
D. A and B

16. B 17. C 18. D


152
19. The disadvantage of lingual split technique is\are:
A. Injury to lingual nerve
B. Opening up of facial spaces on lingual side and floor of mouth
C. Chances of dislodging tooth or root in sub- lingual space
D. All of the above

20. The basic advantage of lingual split technique for extraction of


mandibular impacted teeth is\are:
A. Bone loss is minimal
B. Easy and quick method
C. Tissue trauma is minimal
D. All of the above

21. Lateral trepanation technique of Bowdler Henry is indicated for:


A. Extraction of impacted canines
B. Removal of impacted premolars
C. Removal of partially formed un-erupted third molars
D. Treating dentigerous cysts with enclosed third molars

19. D 20. D 21. C

153
22. The chisel should be used with:
A. Bevel towards the bone which is to be saved
B. Bevel towards the bone which is to be sacrificed
C. Bevel direction is not important
D. Flat surface parallel to direction of grains of bone

23. While making vertical incision for flap for mandibular third molar
impactions one can injure:
A. Buccal pad of fat
B. Branches of lingual nerve
C. Branches of facial nerve
D. Branches of facial artery/ vein

24. Dry socket commonly occurs after:


A. 24 hours
B. 2 days
C. 3-4 days
D. 10-15 days

22. B 23. D 24. C

154
25. The treatment of localized osteitis is:
A. Debridement, curettage and sedative packing
B. Curettage, irrigation and sedative packing
C. Irrigation and sedative packing
D. None of the above

26. In case of extraction of maxillary molars if maxillary tuberosity also


fractures the fractured bone:
A. Should be removed
B. Should be replaced and allowed to heal by secondary intention
C. Should be replaced and retained by primary suturing of soft tissues
D. Should be fixed by transosseous wiring or bone plating

27. The complication of using air rotor at 30,000 RPM for impacted
molars is:
A. Necrosis of cone
B. Dehiscence
C. Tissue laceration
D. Emphysema

25. C 26. C 27. D


155
28. The most important suture while closing Ward's incision for
impacted mandibular third molar is:
A. Suturing of vertical limb
B. Suturing of retromolar limb
C. Suture of area immediately distal to 2nd molar
D. None of the above

29. Dull ache in the jaw and ear three days after a dental extraction may
indicate:
A. Pulpitis of an adjacent tooth
B. Need for a stronger prescription for discomfort
C. Need for an antibiotic
D. Loss of the alveolar clot

30. The scalpel blade most commonly used for oral surgical procedures
is:
A. No. 22
B. No. 12
C. No. 11
D. No. 15

28. C 29. D 30. D

156
31. The "red line" in winter’s classification of impacted teeth represents:
A. The depth at which the impacted tooth is located
B. The angulation of the 2nd molar
C. The summit of the alveolar bone covering the impacted tooth
D. Relation of the 3rd molar to the ramus

32. The most serious complication of an abscess of the maxillary


anterior teeth is:
A. Pain
B. Paresthesia
C. Cavernous sinus thrombosis
D. Orbital cellulitis

33. The beaks of an extraction forceps should be placed:


A. On the anatomical crown
B. At the junction of the clinical and anatomic crowns
C. On the root surface as far apically as possible
D. At the junction of the apical and middle thirds of the root

31. A 32. C 33. C

157
34. A mucoperiosteal flap can be repositioned accurately because the:
A. Periosteum is inelastic
B. Periosteum is very elastic
C. Periosteum contains sharpey's fibers
D. All of the above

35. The root most likely to be pushed into the maxillary sinus during a
tooth extraction is:
A. Palatal root of the maxillary second molar
B. Palatal root of the maxillary first premolar
C. Palatal root of the maxillary first molar
D. Mesiobuccal root of the maxillary first molar

36. A 55-year-old lady has a lone standing carious maxillary first molar
tooth on the right side. The tooth has served as an abutment, and now
has to be extracted. Proper care is to be taken during extraction to
prevent:
A. Alveolar osteitis
B. Pushing the palatal root in the maxillary sinus
C. Removing the floor of the maxillary sinus with the tooth
D. Displacement into the infratemporal fossa

34. A 35. C 36. C

158
37. Which of the following muscles is pierced by the needle while
giving an inferior alveolar nerve block?
A. Medial pterygoid
B. Superior constrictor
C. Temporalis
D. Buccinator

38. Primary healing of a mandibular fracture is seen following fixation


with:
A. Gunning splints
B. Compression plates
C. Trans-osseous wires
D. Champy plates

39. In the maxilla, a compression bone plate can be safely applied along
the:
A. Infraorbital margin
B. Anterolateral wall of the maxillary sinus
C. Fronto-zygomatic suture
D. Zygomaücomaxillary suture

37. D 38. B 39. A

159
40. To provide 'absolute stability' of the fractured ends by a compression
bone plate, the minimum number of screws that have to be placed on
both side of the fracture line is:
A. Six
B. Two
C. Three
D. Four

41. A potential complication following tooth extraction from areas,


having undergone irradiation is:
A. Excessive bleeding
B. Alveolar osteitis
C. Osteoradionecrosis
D. Fracture of the underlying bone

42. A straight elevator is property used to advantage when the?


A. Buccal cortical plate is used as a fulcrum
B. Adjacent tooth is not to be extracted
C. Interdental bone is used as a fulcrum
D. When the bone surrounding the tooth to be extracted is sclerosed

40. D 41. C 42. C

160
43. The ideal order of extraction of teeth is:
A. First molar, canine, lateral incisor, second molar, first premolar and
third molar
B. Third molar, second molar, second premolar, first molar, first
premolar, lateral incisor and canine
C. Lateral incisor, canine, first premolar, second premolar, first molar,
second molar and third molar
D. Canine, first molar, third molar, second molar, 2nd premolar, 1st
premolar and lateral incisor

44. The 'cow-horn' (no. 16) forceps should not be used for the extraction
of mandibular deciduous molars as the:
A. Sharp beaks can damage the un-erupted premolar teeth
B, Beaks cannot engage the furcation area properly
C. Forceps is not intended for extraction of deciduous molars
D. None of the above

45. Prophylactic removal of an erupted third molars in a teenager should


ideally be carried out:
A. When crowding/ rotation occurs in the anterior teeth
B. When the root is one-third formed
C. When the root is two-thirds formed
D. Before the root formation

43. B 44. A 45. C

161
46. The most common contributing factor to pericoronitis of an impacted
mandibular third molar is:
A. Bacterial infection
B. Trauma from the opposing maxillary third molar
C. Trauma from the eruptive forces exerted by the tooth
D. Food impaction

47. A palatal flap for the removal a maxillary impacted canine near the
midline should be:
A. Reflected around the necks of the teeth
B. Started with vertical incisions in the opposite canine region
C. Started with a vertical incision in the midline
D. Semilunar in design

48. Epinephrine packs are to be avoided in controlling bleeding from


extraction sockets as:
A. It interferes with the coagulation process
B. It causes vasoconstriction with resultant alveolar osteitis
C. It causes reflex vasodilatation with severe post- operative bleeding
D. It can be lead to a hypertensive crisis

46. B 47. A 48. C

162
49. In the open beak technique of extraction, the tip of the forceps is
placed at:
A. The cementoenamel junction
B. 2/3rd on the root surface
C. Marginal alveolar bone
D. Two mm apical to the DE junction

50. While removing an impacted mandibular third molar buccal leverage


is applied when?
A. Tooth is inclined mesially
B. Tooth tapers towards the root
C. Tooth is very close to the 2nd molar
D. Space exists between the neck of the tooth and the second molar

51. The extraction of an impacted upper third molar with SA is best


carried out using:
A. A straight elevator
B. Miller's elevator
C. Cryer's elevators
D. Apexo elevators

49. C 50. C 51. B

163
52. Blood loss during full mouth extractions of teeth and alveoplasties,
can be expected to be in the range of:
A. 50-200 ml
B. 200-800 ml
C. 800-1000 ml
D. 1000-1200 ml

53. Facial oedema following surgical extraction of an impacted


mandibular third molar, can be best reduced
A. Postoperative administration of dexamethasone
B. Preoperative administration of diclopfenace sodium
C. Careful retraction and manipulation of the soft tissue flap during
surgery
D. Regular use warm saline mouth rinses after surgery

54. Generally, the treatment of choice for cementoma is:


A. Extraction of all teeth
B. Endodontic treatment of involved teeth
C. No treatment
D. Block resection of involved area

52. B 53. C 54. C

164
55. The healing process that takes place following the non-surgical
extraction of a mandibular 1st molar may be described as:
A. 'Healing by first intention
B. Healing by second intention
C. Granulomatosis
D. A and C

56. Best treatment for pericoronitis associated with impacted mandibular


third molar is:
A. Irrigating under the operculum
B. Antibiotic and analgesic therapy
C. Extraction of impacted third molar
D. Operculectomy

57. A palatal flap is required for removal of a maxillary impacted canine


near the midline. Palatal flap should be:
A. Started with a vertical incision in the midline
B. Reflected from the necks of the teeth
C. Made with a semilunar incision
D. None of the above palatal flap should be avoided

55. B 56. C 57. B

165
58. During the tooth extraction a known cardiac patient experienced
angina. This was most likely precipitated
A. Allergy to the anesthetic agent
B. The stimulus of pain and anticipation
C. Adrenaline in the drug
D. The upright position in the chair

59. Three days after the extraction of maxillary first molar patient
developed a fluctuant palatal abscess. The treatment of choice is:
A. Caldwell-Luc procedure
B. Heat and cold therapy
C. I and D and antibiotic therapy
D. Only antibiotic therapy after culture and sensitivity tests

60. Early sign of syncope is:


A. Epistaxis
B. Pupillary constriction
C. Pallor
D. Rapid pulse and increased BP

58. B 59. C 60. C


166
61. Among the following, the factor of least importance in alveolar flap
design is:
A. Linear incision
B. Closure over the bone
C. Tension free closure of the flaps
D. Base relationship to length

62. The following are indication for suturing following exodontia


except:
A. Blood clot protection
B. Reapproximation of mucoperiosteal flaps
C. Immobilisation of tissues
D. Hemostasis

63. In the extraction of mandibular third molars, the main reason why
the posterior incision should be placed more buccally is:
A. To prevent damage to lingual nerve
B. Incision should be on the sound bone
C. To prevent damage to retromolar artery
D. All of the above

61. A 62. A 63. D

167
64. In multiple extractions, the order of extraction usually:
A. Maxillary before mandibular and posterior before anterior
B. Mandibular teeth before maxillary
C. Maxillary teeth before mandibular
D. Anterior teeth before posterior

65. Limited accessibility is one of the most disadvantage of the


following flap:
A. Envelop flap
B. Semilunar flap
C. Three cornered flap
D. Four cornered flap

66. According to Nitzin's theory, the following microorganisms are


responsible for dry socket:
A. Treponema denticola
B. Leprospira pyrogen
C. Pseudomonas
D. Bacillus subtilus

64. A 65. B 66. A


168
67. Most common site for dry socket:
A. Lower incisor area
B. Upper incisor area
C. Lower molar area
D. Upper molar area

68. Few days after a dental extraction patient came with marked pyrexia
and complaining of impairment of labial sensation and marked
tenderness on extra oral palpation. Most probable diagnosis is:
A. Dry socket
B. Acute osteomyelitis
C. Postoperative hematoma
D. None of the above

69. Advantage of chisel and mallet over bur in removing bone is:
A. Less skill is needed
B. Psychologically more acceptable
C. No coolant is necessary
D. Chisel and mallet are good instruments to remove dense bone

67. C 68. D 69. C


169
70. The following design parameters should follow to prepare a
mucoperiosteal flap except:
A. Flap should provide adequate visibility and accessibility
B. Free margin broader than base
C. Ragged margins should be avoided
D. Flap should be a full thickness mucoperiosteal flap

71. Most common cause of post-extraction bleeding is:


A. Bleeding disorders
B. Failure of the patient to follow post extraction instructions
C. Due to the analgesics such as aspirin
D. Patient with the anticoagulant therapy

72. A deep level of general anesthesia is enhanced by a:


A. Muscle relaxant like d-tubocurarine
B. Non-irritating drug
C. Loose fitting mask
D. High alveolar concentration of anesthetic agent

70. B 71. B 72. D

170
73. "Postage stamp" method is:
A. A method of bone removal in transalveolarextraction
B. A method of extraction of maxillary canines by intra- alveolar
method extraction
C. A method of bone grafting
D. None of the above

74. "Line of withdrawal" of a tooth is mainly determined


A. The size of the crown
B. The root pattern
C. Shape of the crown
D. Size of the roots

75. "Odontectomy" is synonymous to:


A. Transalveolar extraction
B. Intra-alveolar extraction
C. Extraction under local anesthesia
D. Extraction under general anesthesia

73. A 74. B 75. A

171
76. The best example of an elevator which works on wheel and axle
principle?
A. Howartli's periosteal elevator
B. Winter cross bar elevator
C. Millers apexoelevator
D. None of the above

77. Impacted mandibular third molar classification is usually based on:


A. Relation of the tooth to the ramus of the mandible
B. The position of the tooth in relation to the long axis of the second
molar
C. The relative depth of the third molar in bone
D. All the above

78. Ten teeth have been removed for a patient who was “J”
premedicated. The proper position of the patient in Pfy the recovery
room is:
A. Head elevation 300 with patient on his back
B. Trendelenburgts position patient head is almost parallel to the floor
C. Reverse Trendelenburg's position
D. Supine position

76. B 77. D 78. A

172
79. A swelling over lies an extraction wound and it crackles on
palpation. Most likely diagnosis is:
A. Ecchymosis
B. Cellulitis
C. Emphysema
D. Empyema

80. During reflecting a palatal mucoperiosteal flap for removal of an


impacted canine structures passing through the incisive canal are
averted. The sequela to this is:
A. Of no clinical significance
B. Loss of a triangular mucosal flap behind the incisors and canine due
to loss of blood supply
C. Resorption of bone around the canal
D. Anesthesia of the anterior palate

81. Which of the following is the principle action of ammonia in


syncope?
A. Respiratory stimulant
B. Vagal stimulant
C. Vasomotor stimulant
D. Inhibitor of vasomotor tone

79. C 80. A 81. A


173
82. After the surgical removal of chronically infected teeth in a
controlled diabetic patient, which of the following is of utmost
importance?
A. Postoperative check of patient for bleeding
B. Return of patient to diet of a normal individual
C. Revaluation of insulin dosage being taken by the patient
D. Medication for marked post-operative pain and discomfort

83. The following nerves should be anesthetized for the removal of


maxillary first molar:
A. Posterior palatine, anterior palatine
B. Anterior palatine, Middle superior alveolar, posterior superior
alveolar
C. Anterior superior alveolar
D. Posterior superior alveolar

84. During the maxillary teeth extraction, a patient developed severe


pain and purulent discharge from the nose. The following conditions and
tooth are most probably involved:
A. Perforation of the maxillary sinus, first molar
B. Hematoma with in a facial plane, second molar
C. Injury to buccinators muscle, canine
D. Perforation of the maxillary sinus, central incisor

82. C 83. B 84. A

174
85. Extraction of mandibular first molars in an 8-year-old child are also
known as
A. Premature extraction
B. Serial extraction
C. Orthodontic extraction
D. Wilkinson's extraction

86. Etiology of "localized alveolitis" (dry socket) includes:


A. Trauma
B. Vascular supply to regional bone
C. Infection
D. All of the above

87. When mandibular teeth are to be extracted, the patient should be


positioned in such a way that the occlusal plane of the mandibular arch
is:
A. Parallel to the floor when the teeth are closed together
B. Parallel to the floor when the mouth is wide open
C. Perpendicular to the floor when the teeth are closed together
D. Perpendicular to the floor when the mouth is wide open

85. D 86. D 87. B

175
88. A 75-years-old woman whose weight is approximately 100 pounds
requires multiple extractions under local anesthesia. For premedication,
the fol. lowing dose of diazepam is enough to allay the apprehension?
A. 20 mg
B. 30 mg
C. 10 mg
D. 5 mg

89. The beak of an extraction forceps is designed such a way that the
extraction pressure is transmitted to the:
A. Crown of the tooth
B. Alveolar bone
C. Root of the tooth
D. Cementoenamel junction of the tooth

90. During the extraction of mandibular third molar it is noted that the
distal root is missing. The root tip is most probably in the:
A. Submental space
B. Submandibular space
C. Pterygomandibular space
D. Parapharyngeal space

88. B 89. C 90. B

176
91. A 45-year-old women insists on extraction of painful mandibular
first molar. She has not rested for 2 days and nights because of pain. Her
medical his- tory reveals she took 30 mg of prednisone daily for one
year but stopped taking the medication three months age. The dentist
should:
A. Relieve the symptoms by giving antibiotics and analgesics
B. Give steroid supplements and extract the tooth under local anesthesia
C. Avoid procaine anesthetic
D. Extract the tooth under local anesthesia in a usual manner

92. Recontouring of the residual ridge in the early part of extraction of


period occurs primarily by resorption of the:
A. Buccal-lingual cortical bone
B. Labial-buccal cortical bone
C. Lingual cortical bone
D. Crestal bone

93. Three days after extraction of teeth for an immediate denture, patient
complains of a diffuse, non-painful yellow, submandibular and supra
sternal discoloration of the skin. The dentist should:
A. Advise antibiotics
B. Apply heat and advice analgesics
C. Apply cold compressions
D. None of the above
91. B 92. B 93. D

177
94. Of the following which tooth is most difficult to anesthetize by
infiltration technique alone:
A. Maxillary canine
B. Maxillary central incisors
C. Maxillary first molar
D. Maxillary first premolar

95. To luxate a tooth with a forceps the movements should be:


A. Finn and deliberate primarily to the facial surface with secondary
movements to the lingual surface
B. Sharp and definite so periodontal ligament tears easily
C. Gentle wiggles so patient get used to the pressure
D. Figure of "eight" motion

96. When a root is accidentally displaced into the maxillary antrum, the
dentist should?
A. Enlarge the socket to gain access
B. Place a drain into the socket
C. Refer to an experienced surgeon for root removal
D. Pack the socket with gel foam to prevent fistula formation

94. C 95. A 96. C

178
97. The left hand of the operator should serve the following purpose
when extracting a mandibular tooth?
A. Support the mandible
B. Protect other teeth
C. Retract soft tissues
D. All of the above

98. Patients with end-stage renal disease are of increased risk when
undergoing extraction of teeth because they?
A. Have an increased tendency to bleed
B. Are susceptible to infections
C. Are often on steroid therapy
D. All of the above

99. During extraction of maxillary third molar the maxillary tuberosity is


fractured and remains attached to periosteum with an intact blood
supply. The treatment of choice is to:
A. Refer the case to an oral surgeon
B. Remove the fractured tuberosity
C. Reposition and Stabilize loss of the tuberosity
D. Send the patient to preventive dentistry with instructions to prevent

97. D 98. D 99. C

179
100. Among the following which is most common complication after
surgical removal of a mandibular tooth?
A. Post extraction hemorrhage
B. Paresthesia of lingual nerve
C. Loss of blood clot
D. Difficulty in swallowing

101. A severe dull ache in the jaw three or four days after an extraction
of tooth usually indicates:
A. Exposure of cementum of an adjacent tooth
B. Acute pulpitis of an adjacent tooth
C. Prescription of an antibiotic agent
D. Loss of alveolar blood clot

102. Hemorrhage after dental extraction procedure can be prevented by:


A. Applying pressure
B. Ligating bleeding major vessels
C. Properly designing and carefully reflecting mucoperiosteal flaps
D. All of the above

100. C 101. D 102. D


180
103. Trismus after extraction most likely results from:
A. Passing the needle through medial pterygoid muscle while injecting
B. Injecting the solution near a branch of facial nerve
C. Injecting the solution near a branch of trigeminal nerve
D. Systemic administration of LA

104. Local factor for postponing tooth removal is:


A. Periodontitis
B. Hemangioma in the region of the tooth
C. Hypercementosis
D. Malignancy

105. Number 16 cowhorn forceps are specifically designed to extract:


A. Maxillary first and second premolars
B. Mandibular central incisors
C. Mandibular molars
D. Maxillary molars

103. A 104. B 105. C

181
106. A small opening is made into the maxillary antrum during
extraction, immediate treatment is:
A. Pack the socket with gauge
B. Allow the clot to form No special treatment is necessary
C. Place the patient on antibiotics
D. Raise a big mucoperiosteal flap and close the antrum

107. 'Rubber band extraction' is a method of extraction in patient having:


A. Hemangioma and hemophilia
B. Pericoronitis and impacted molars
C. Super numerary teeth and ankylosed teeth
D. Myocardial infraction and angina pectoris

108. A straight elevator is properly used to advantage when the:


A. Adjacent tooth is the fulcrum
B. Tooth is isolated tooth
C. Interdental bone is fulcrum
D. Adjacent tooth is not to be extracted

106. B 107. A 108. C

182
109. Contributing factor to pericoronitis of an impacted mandibular third
molar is:
A. Trauma by opposing tooth
B. Previous radiation therapy
C. Systemic disease
D. Infected follicular cyst

110. After the posterior superior nerve block patient should experience:
A. Numbness of the lip
B. Numbness of the palate till the first premolar
C. Few symptoms of anesthesia
D. Numbness of that side efface

111. The tips of anatomic forceps should be placed:


A. On the crown portion
B. At the junction of clinical and anatomical crowns
C. Near junction of apical and middle thirds of clinical crown
D. On the root surface as far apically as possible

109. A 110. C 111. D

183
112. In designing the flap the following is correct:
A. The flap should be as small as possible
B. Free margins should be wider than the base
C. Sutured margins should be supported by a blood clot
D. It should be a mucoperiosteal flap

113. Appropriate time to administer analgesic to reduce postoperative


pain associated with removal of two mandibular molars:
A. When pain becomes severe
B. Before the effect of LA wears off
C. The morning of surgery
D. Only after the return of sensation

114. While extracting mandibular teeth the patient should be positioned


so that the occlusal plane of the mandible should be:
A. Parallel to the floor when jaws are closed
B. Form some angle to the floor when the mouth is wide
C. Parallel to the floor when the mouth is wide open
D. Perpendicular to the floor when the mouth is wide open

112. D 113. B 114. C


184
115. While extracting a multi-rooted tooth, the crown fractures, the
prudent next step is to?
A. Divide the roots with a bar
B. Reflect a flap
C. Reapply the forceps in a more apical direction
D. Leave the roots in place and observe

116. During the extraction of mandibular third molar, it is noted that


distal root of the mandibular third molar, the root is most probably in
the:
A. Submandibular space
B. Submental space
C. Inferior alveolar canal
D. Pterygomandibular space

117. A patient came for extraction of his four anterior teeth all with
grade 3 mobility. He is under prednisone 25 mg for the past three
months. The dentist should:
A. Take the patient off prednisone therapy
B. Consult the physician who will probably take the patient off
prednisone therapy for at least five days before the procedure
C. Be concerned with adrenal insufficiency
D. Proceed normally

115. A 116. A 117. C

185
118. The following maxillary third molar impaction is most likely to be
displaced in to the infratemporal fossa the antrum if improper technique
is used:
A. Mesioangular
B. Vertical
C. Distoangular
D. Mesiolingual

119. The most common complication after surgical removal of a


mandibular tooth is:
A. Paresthesia of lingual nerve
B. Paresthesia of inferior alveolar nerve
C. Postoperative hemorrhage
D. Loss of blood clot

120. Which maxillary impactions are most likely to be displaced into the
antrum or the infratemporal fossa with incorrect technique during the
extraction?
A. Distoangular
B. Mesioangular
C. Vertical
D. Faciolingual

118. C 119. D 120. A


186
121. Paresthesia of the lower lip is most commonly the result of:
A. A benign tumor of the mandible
B. Removal of mandibular third molar
C. Periapical abscess of first molar
D. Fracture of the mandible

122. Following may cause postoperative hemorrhage:


A. Liver disease
B. Aspirin and other salicylates medication
C. Prolonged use of broad spectrum antibiotics
D. All of the above

123. The difficult tooth to be anesthetized by infiltration technique is the


maxillary:
A. Central incisor
B. First molar
C. Third molar
D. Canine

121. B 122. D 123. B

187
124. Infiltration in maxillary first molar region is not effective because:
A. Of zygomatic buttress bone in the region
B. Of the presence of parotid opening in that region
C. Of three roots
D. It is supplied by two nerves

125. The following mandibular teeth can be removed without pain after
the inferior alveolar and lingual nerve block:
A. All molars and second premolars
B. First molar and premolars
C. Canine and first premolar
D. All teeth in that quadrant

126. The following history and laboratory value contraindicate


extraction of mandibular first molar:
A. Hb% is 15 gm% and hematocrit value is 42 percent
B. Five months pregnant lady without any complications
C. Rheumatic patient taking penicillin
D. Prothrombin time is 45 seconds

124. A 125. C 126. D


188
127. A 48-year-old, diabetic patient wants extraction of 2 teeth which
are in grade I mobility. He takes 40 units of NPH insulin daily. Which of
the following management procedure in indicated to remove the teeth?
A. Defer treatment, have him take breakfast, but not take insulin to avoid
hypoglycemia
B. Defer treatment, have the patient take his insulin but tell him to avoid
breakfast
C. Treat the patient in the usual manner without much alteration in the
diet and drugs
D. Admit the patient into a hospital for the procedure

128. A 20-year-old patient requires extraction of all erupted teeth


because of extensive decay. He has two maxillary third molar
impactions. X-ray film shows bony impactions. The molars should be:
A. Extracted along with other teeth
B. Retained because no changes of eruption
C. Retained under a denture until they erupt
D. Exposed surgically and can be used for prosthetic retention

127. C 128. C

189
129. Four days after multiple dental extractions, an elderly fair skinned
women complains of black and blue marks over her neck. The most
probable diagnosis is:
A. Polycythemia
B. Leukemia
C. Postoperative ecchymosis
D. Manifestation of basal cell carcinoma

130. A 45-year-old man requires a tooth extraction. His medical history


indicated myocardial infarction two years ago. This patient is receiving
anticoagulant drugs. The dentist should:
A. Never operates on this patient
B. Avoid performing surgery until the anticoagulant had been
discontinued for at least three days
C. Perform surgery if prothrombin time is 18 seconds
D. To avoid postoperative hemorrhage advice vitamin

129. C 130. C

190
131. A patient who is receiving anticoagulants requires extraction of a
tooth. Laboratory data indicating prothrombin time is 21 seconds with a
control time of 15 seconds. The treatment should consist of:
A. Regulation of the anticoagulant level with heparin
B. Injection of vitamin K and immediate extraction
C. Injection of vitamin K and extraction after half an hour
D. Extraction of the tooth and use of sutures and local hemostatic agent

132. Persistent pyrexia after full mouth extraction results from:


A. Wound infection
B. Dehydration
C. Endocarditis
D. Any of the above

133. Prophylactic antibiotic coverage before dental extraction is


indicated for patients with each of the following conditions except:
A. Kidney damage needing hemodialysis
B. Prosthetic aortic valve
C. Rheumatic heart disease
D. Coronary artery bypass

131. D 132. D 133. D

191
134. A mandibular molar was extracted because of advanced caries.
After one month patient developed an extra oral indurated swelling near
the previous extraction site. Multiple draining sinuses appeared and
drained pus with yellow granules. The most probable diagnosis is:
A. Histoplasmosis
B. Tuberculosis
C. Actinomycosis
D. Streptococcal infection

135. Most serious potential complication after extraction from areas


previously irradiated is:
A. Osteoradionecrosis
B. Alveolar osteitis
C. Prolonged healing
D. Prolonged suppression of salivary secretion

136. A patient complains of persistent bleeding five hours after the


extraction. Initial treatment is to:
A. Advise bleeding time and clotting time and other laboratory
investigations
B. Inject a local anesthetic solution into the area to provide
vasoconstriction
C. Remove clots and examine the area to locate the source of bleeding
D. Have the patient bite on a gauge socked with adrenaline
134. C 135. A 136. C

192
137. After a tooth extraction under local anesthesia, a patient with a
known cardiac problem experiences angina. This is most likely
precipitated by:
A. Allergy to the anesthetic agent
B. Allergy to preservative agent in the anesthetic solution
C. The stimulus of pain and anticipation
D. The epinephrine in the anesthetic solution

138. During the extraction of maxillary first molar, the palatal root tip of
molar slipped into the maxillary sinus. Proper way to approach to
recovery is through the:
A. Same socket by enlarging the opening through which the root entered
the sinus
B. Maxillary incisive fossa
C. Maxillary incisive fossa medial to canine
D. Lateral nasal wall in the middle meatus of the nose

139. A mandibular left second molar is un-erupted in a 14 year-old boy.


Radiological examination reveals a dentigerous cyst surrounding the un-
erupted tooth. The treatment of choice is:
A. Observe for at least 2 years
B. Aspirate and send the contents of the cyst to biochemical analysis
C. Uncover the crown and keep it exposed
D. Extract the tooth under local anesthesia
137. C 138. C 139. C

193
140. The prime purpose of Bibevel chisel is:
A. Split teeth
B. Sharpen the angles
C. Remove bone
D. Engage point establishment

141. Ammonia inhalation in the treatment for syncope acts by


producing:
A. Bronchodilatation
B. Vasoconstriction
C. Sedation
D. Mucosal irritation

142. Osteotome is:


A. Monobeveled
B. Bibeveled
C. Tetrabeveled
D. Not contain any bevels

140. A 141. D 142. B

194
143. The following nerves should be anaesthetized for extraction of a
maxillary lateral incisor:
A. Posterior superior alveolar
B. Nasopalatine
C. Anterior palatine
D. Anterior superior alveolar

144. During the administration of an inferior alveolar nerve block which


of the following muscles is usually penetrated with the needle:
A. Buccinator
B. Medial pterygoid
C. Lateral pterygoid
D. Masseter

145. For the removal of root apex from the maxillary sinus, one should
block the following nerves:
A. First division of trigeminal nerves
B. Second division of trigeminal nerve
C. Third division of trigeminal nerve
D. Infraorbital nerve

143. B 144. A 145. B

195
146. Treatment of choice for a localized osteitis or dry socket following
extraction of a tooth is:
A. Topical antibiotic application in the socket
B. Systemic antibiotic administration
C. Debridement of the wound by irrigation and placement
D. Curetting the bony walls of the socket to induce bleeding and clot
formation

147. A patient returns several days after extraction of three adjacent


mandibular molars and complains of pain similar to dry socket. The first
step is to:
A. Currete the three-socket area thoroughly in an attempt to establish
new blood clot
B. Explore the sockets gently with curet
C. Irrigate the socket and place the dressing
D. Radiographic examination of the area

148. The following muscle is most frequently encountered in an


incorrect infraorbital nerve block:
A. Orbicularis oris
B. Triangularis
C. Quadratus labi superiors
D. Risorious

146. C 147. B 148. C

196
149. A patient who is under steroid therapy and has need for extraction
of chronically infected teeth. Premedication includes:
A. Antihypertensive drugs to prevent shock
B. Atropine to prevent cardiac arrest by vagal stimulation
C. Antihistamines
D. Antibiotics

150. Forty eight hours after the extraction of impacted mandibular third
molar patient returns to your office complaining of moderate pain
radiating to the ear. His temperature is 990 F and swelling is minimal.
The most probable diagnosis is:
A. Trauma to the inferior alveolar nerve during the injection
B. Postoperative infection of parotid space
C. Postoperative infection of masticatory space
D. Post-extraction alveolitis

151. The elevator can be used to advantage when?


A. Multiple adjacent teeth are to be extracted
B. The interdental bone is used as a fulcrum
C. The tooth to be extracted is isolated
D. The adjacent tooth is not to be extracted

149. D 150. D 151. A

197
152. Post extraction localized alveolitis or "dry socket" is usually
characterized clinically by:
A. A brawny, indurated swelling with hyperpyrexia over 100" F
B. Continuous dull pain with little or no adjacent swelling
C. Attacks of sharp, lancinating pain with swelling
D. Local tenderness and ecchymosis of the skin and mucous membrane

153. Extraction of an isolated residual maxillary molar offers the


technical hazard of:
A. The development of maxillary sinusitis
B. Displacement into the infratemporal fossa
C. Fracture of the tuberosity or floor of the antrum
D. Producing a nasal oral fistula

154. Prophylactic removal of unerupted teeth is accomplished with best


results at certain stage of tooth development. The best time to remove
such a tooth
A. When two-thirds of the root is formed
B. When the root is completely formed
C. Before any of the root is formed
D. When the crown of the tooth is above the cementoenamel junction of
adjacent tooth

152. B 153. C 154. A


198
155. Following a maxillary tuberosity injection, within a few seconds,
patient face becomes extremely distended and swollen on the injected
side treatment is:
A. Incision and drainage
B. Antibiotic administration
C. Use of cold packs and pressure on the side
D. Antihistamine administration

156. The maxillary first molar is extracted by forceps method. The


healing of the socket can be described as:
A. Healing by primary intention
B. Granulomatosis
C. Healing by secondary intention
D. Epithelialization

157. The healing process that takes place following approximation of the
flaps are described as:
A. Healing by first intention
B. Healing by second intention
C. Healing by granulomatosis
D. Hematoma formation followed by organization

155. C 156. C 157. A

199
158. A tooth is resistant to luxation with forceps. Its removal is best
achieved by:
A. Fracture the crown with forceps
B. Fracture the crown with chisel and roots with forceps
C. Transalveolar method
D. Split tooth technique

159. The immediate step following the forceps extraction of a maxillary


central incisor with a periapical radiolucency is:
A. Examine the root apex for soft tissues pathology
B. Pack the socket with a medicament
C. Currets the alveolus, if tissue was not removed with extraction
D. A and C

160. "Ash forceps" is commonly used for:


A. Extraction of lower incisor
B. Extraction of upper canines
C. Holding tissues prior to biopsy
D. Holding vascular areas during surgery

158. C 159. D 160. A


200
161. If there is visible bleeding from an isolated major vessel, it is best
controlled by:
A. Systemic administrations of Vitamin K
B. Electrocoagulation
C. Clamping and ligation
D. B and C

162. Of the following most difficult impaction to extract


A. Mesioangular
B. Vertical
C. Distoangular
D. Horizontal

163. Rongeur is commonly used to:


A. Cut the bone
B. Extract third molars
C. Place the sutures in the areas where access is limited
D. None of the above

161. D 162. C 163. A

201
164. Under general anesthesia patient needs oropharyngeal:
A. Lap pad
B. 4 x 4 gauge
C. Ferguson mouth gag
D. Pick up forceps

165. One decided to use forceps for removal of a tooth.


The first direction for the force to be applied is:
A. Buccally
B. Lingually or palatally
C. Apically
D. Occlusally

166. After the extraction of maxillary first molar, a communication is


found between the palatal socket and a disease-free maxillary sinus
which measure 0.2 cm. The best treatment 'is:
A. Allow the clot to form advice proper home care
B. Primary closure and antihistamine
C. Gold foil closure
D. Caldwell-Luc operation

164. B 165. C 166. A


202
167. An approach to an impacted mandibular molar is achieved by:
A. Envelop flap
B. Bayonet flap
C. L shaped flap
D. All of the above

168. The muscles that aid in displacement of maxillary fractures are:


A. Masseter
B. Temporalis
C. Upper part of orbicularis and lower part of orbicularis occuli
D. None of the above

169. A bony opening in the canine fossa Caldwell-Luc procedure is


commonly used to:
A. Expose canine root for apicoectomy
B. Enter the maxillary sinus
C. Uncover and marsupilize a cyst
D. Establish drainage of an alveolar abscess

167. D 168. D 169. B

203
170. Ankylosis of the TMJ is best treated with:
A. Exercise and massage
B. Sclerosing solutions
C. Antibiotics
D. Condylectomy

171. Alveolectomy is one of the commonly used procedure. It is:


A. Closure of an oroantral fistula
B. Removal of alveolar bone
C. Reconstruction of mucoperiosteum and alveolar bone
D. None of the above

172. Which is best treatment for pericoronitis involving an impacted


mandibular third molar?
A. Antibiotic and analgesic therapy
B. Operculectomy
C. Extraction of the involved third molar
D. Gently irrigating under the operculum

170. D 171. B 172. C

204
173. Postoperative edema can be minimized by:
A. Careful surgical manipulation of hard and soft tissues
B. Administration of antihistamines preoperatively
C. Application of heat immediately after the procedure
D. Application of heat and cold simultaneously

174. During the dental procedure suddenly patient became "light headed,
diaphoretic" then became conscious. The most diagnosis is:
A. Shock
B. Syncope
C. Cerebrovascular accident
D. Hyperventilation

175. One day after the extraction, patient came with the complaint of a
rubbery, non-tender midfacial swelling. The most likely diagnosis:
A. Abscess
B. Fibroma
C. Hematoma
D. Inflammatory edema

173. A 174. B 175. C


205
176. Among the following which is most appropriate preoperative
instruction for a diabetic patient undergoing extraction of his teeth:
A. Maintain normal diet and normal medication
B. Increase medication preoperatively
C. Increase sugar intake prior to surgery
D. Do not eat or take any medication before the surgery

177. A patient who is a grand mal epileptic requires emergency


extraction. One should:
A. Advise the patient that procedure require admission to the hospital
B. Proceed with extraction using xylocaine without adrenalin
C. Proceed with extraction if anti-convulsive medication is employed by
patient
D. Inform the patient that in his condition no extraction should be
performed

178. Which of the following medication require preoperatively for a


patient with mitral valve replacement?
A. Ampicillin
B. Cloxacillin
C. Benzathine penicillin
D. Procaine penicillin

176. A 177. C 178. D


206
179. A 30-year-old female patient with a BP of 150/80 requires
treatment. You would:
A. Recommend general anesthesia
B. Seek medical treatment to lower BP
C. Refer the patient to an oral surgeon
D. Use sedation preoperatively

180. Among the following which condition will require specific


management prior to emergency extraction?
A. Hemophilia
B. Glaucoma
C. Carcinoma tongue
D. Mixed tumor of parotid

181. Elective dental extractions and minor oral surgical procedures on a


patient who has had a myocardial infarct two months prior are best?
A. Performed using an adrenaline free local anesthetic, because of
adrenaline action on heart muscle
B. Performed under oral diazepam
C. Postponed until at least six months have elapsed
D. Performed with normal

179. D 180. A 181. C

207
182. Among the following, which is true about mucoperiosteal flap
design?
A. Flaps should be narrower at the base
B. Flap margin should not rest on sound bone
C. Flap should be wider at the base
D. Flaps should be avoided

183. Among the following which condition is contraindicated to the use


of local anesthesia?
A. Pregnancy
B. Hypersensitivity to the drug
C. Hepatic damage
D. Parkinson's disease

184. Post extraction complications are most often caused by:


A. Inadequate diet
B. Inadequate preoperative testing
C. Inadequate patient instructions
D. Poor surgical technique

182. C 183. B 184. D


208
185. Among the following which is most important in prevention of post
extraction bleeding problems?
A. Transfusion of blood
B. Tight suturing of flaps
C. Ice to face
D. Local pressure

186. Five days after extraction of an impacted mandibular third molar,


your patient presents with hard, painful swelling of 12 hours duration
near angle of the mandible. This is most likely due to:
A. Angioneurotic edema
B. Hematoma
C. Infection
D. Surgical trauma

187. Inadvertent displacement of the teeth into inaccessible areas of head


and neck and be effectively prevented by:
A. Use of prudent surgical technique
B. Use of gauze pharyngeal curtain
C. Evaluation of radiography prior to surgery
D. All of the above

185. D 186. C 187. D

209
188. A mandibular second molar is extracted. It is found that the distal
root is fractured and the tip is missing. When the dentist attempts to
recover it and finds that his instrument will pass downward and into the
lingual soft tissue, the tip is most likely to be in the:
A. Submental space
B. Sublingual space above the mylohyoid muscle
C. Submandibular space below the mylohyoid muscle
D. Parapharyngeal space

189. 48-72 hours after extraction patient developed fever. The problem
may result from:
A. Wound infection
B. Endocarditis
C. Cellulitis
D. Any of the above are corrected

190. Wilkinson extractions are:


A. A type of extractions done with Wilkinson elevator
B. Done in order to permit eruption of 3rd molars in the proper position
C. Usually done in the patients above 45 years
D. None of the above

188. C 189. D 190. B

210
191. The best time of extraction in pregnancy:
A. First trimester
B. Second trimester
C. Third trimester
D. None of the above

192. Of the following which teeth need primarily tory movement to


extract when?
A. Maxillary first premolar
B. Mandibular second premolar
C. Mandibular central incisor
D. Mandibular lateral incisor

193. Bone removal with aeroter is usually contra indicated because of


the danger of developing:
A. Hematoma
B. Emphysema
C. Abscess
D. Edema

191. B 192. B 193. B


211
194. Which of the following nerves should be blocked to remove a root
tip from the antrum?
A. Anterior superior alveolar nerve
B. Posterior superior alveolar nerve
C. Second division of the trigeminal nerve
D. Third division of trigeminal nerve

195. Which of the following procedures eliminate hemorrhage after


dental extraction procedures?
A. Ligating bleeding vessels
B. Applying pressure and carefully reflecting mucoperiostema
C. Eliminating systemic bleeding disorders
D. All of the above

196. Cavernous sinus thrombosis following infection of maxillary and


anterior teeth most often results from spread of infected emboli along
the:
A. Pterygoid plexus
B. Ophthalmic vein
C. Facial artery
D. Angular artery

194. C 195. D 196. B

212
197. Among the following which muscle is most frequently encountered
in an incorrect infraorbital nerve block?
A. Quadratus labi superioris
B. Mentalis
C. Rizorius
D. Orbicularis oris

198. Management of dry socket (localized alveolar osteitis) include all


of the following except:
A. Curetting the socket and Promote bleeding
B. Normal routine irrigation to flush out the debris
C. Placing a sedative dressing to protect the exposed bone
D. Advising analgesics as an adjunctive treatment

197. A 198. A
213
Chapter 5: Minor Surgical Procedure
1. Repeat biopsies are indicated:
A. For blastomatoid conditions
B. When ulcers fail to heal in four days
C. In bone defect lesions that fail to heal in six weeks
D. When clinical course differs from a reported benign process

2. Hyperplastic palatal tissue known as papillomatosis is seen often


beneath dentures. This condition is best treated by:
A. Supraperiosteal dissection or electro-surgery
B. Not allowing the patient to wear his denture
C. Radical excision because it is premalignant
D. Chemical cautery

3. A soft tissue flap consisting of mucosa and periosteum can be


repositioned accurately because the:
A. Periosteum is very elastic
B. Mucosa is mostly collagen fibers
C. Periosteum sends filiform-like projections into the haversian canals
D. Periosteum is relatively inelastic

1. D 2. A 3. D

214
4. Treatment of a mucocele on the lower lip is by:
A. Incision
B. Excision
C. Electro-surgery
D. Excision, including adjacent minor salivary glands

5. An excisional biopsy is characterized by:


A. Using a trephine
B. Exfoliative cytologic techniques
C. Including normal tissues and most of the lesion
D. Including normal tissue and all of the lesion

6. In oral surgery, cold pac Epulis Fissuratum is most commonly due to:
A. Ill-fitting dentures over resorbed ridges
B. Infection by Candida albicans
C. Poorly performed 'Z' plasty
D. None of the above

4. D 5. D 6. A

215
7. The recurrent ranula is best treated by:
A. Electro-surgery
B. Cryosurgery
C. Marsupialization
D. Sub-lingual gland excision

8. Incisional biopsy of an ulcer is taken:


A. At the center of the lesion
B. Edge of the lesion
C. Edge of the lesion along with normal tissues
D. At any part of the lesion

9. Which one of the following should not be done during in incisional


biopsy of soft tissue?
A. Place suture through the intended specimens before removing it
B. Infiltrate local anesthetic solution around the intended site
C. Place the specimens in saline if 10% formalin is unavailable
D. Obtain some adjacent normal tissue if possible

7. D 8. C 9. C

216
10. The purpose of aleveoloplasty is to
A. Eliminate the undercuts that interfere with seating of the denture
B. Avoid rounding off the sharp bony projections as they will round off
in a few days during the healing process
C. Retain the width of the alveolar ridge even though undercuts exists
D. None of the above

11. Packs are applied:


A. Immediately prior to surgery
B. Immediately after surgery
C. To prevent the spread of infection
D. To produce vasodilation

12. During full mouth extraction of teeth and alveloplasties, blood loss
can be expected to be in the range of:
A. 0-100 ml
B. 200-800 ml
C. 800-1000 ml
D. 1000-1500 ml

10. A 11. B 12. B


217
13. The most frequently used approach to temporomandibular joint
surgery is:
A. Endural
B. Submandibular
C. Preauricular
D. Postauricular

14. The primary intent of an extra-oral surgical dressing is to:


A. Support the incision
B. Protect the surgical site from trauma
C. Absorb drainage
D. Keep the surgical field free from infection

15. How long should one wait before obtaining a biopsy of an oral
ulcer?
A. 4 days
B. 7 days
C. 14 days
D. 30 days

13. C 14. D 15. C


218
16. To prevent tearing of the mucosal edges during wound closure, the
following needle should be used:
A. Round bodied
B. Cutting
C. Reverse cutting
D. Five edged

17. The following principles should be adhered to when performing


alveoplasties after extractions?
A. Conservation of bone
B. Use only absorbable sutures
C. The soft tissue margins should be tightly approximated
D. All of the above

18. Damage to the following structure should be avoided while


performing an apicectomy on tooth no. 7 (12):
A. Nasal floor
B. Inferior alveolar canal
C. Anterior superior nerve
D. Incisive canal

16. A 17. A 18. A

219
19. A suitable fixative for a routine biopsy specimen is:
A. 10 % formalin
B. 40 % formalin
C. 20 % formalin
D. 10 % alcohol

20. To achieve surgical hemostasis in a patient suffering from


Hemophilia A, preoperative administration of the following blood
product should be preferred:
A. Whole blood only
B. Fresh frozen plasma
C. Factor VIII cryoprecipitate
D. Factor VIII concentrates

21. A skin graft vestibuloplasty prevents relapse by:


A. Physically maintaining the depth of the vestibule
B. Preventing the reattachment of the muscle to the periosteum
C. Causing an inhibitory effect on fibroblasts in the underlying tissue
D. All of the above

19. A 20. D 21. A


220
22. Pre-prosthetic surgery on the mandible requires the preservation of
the:
A. Labial frenum
B. Palatal tori
C. Mylohyoid ridge
D. Genial tubercles

23. Chromic catgut sutures are preferred over plain catgut as they
exhibit:
A. Greater strength
B. Less tissue inflammatory response
C. Delayed resorption
D. Ease of manipulation

24. Removal of mylohyoid ridge is planned in an edentulous patient.


Which of the following nerves should be protected?
A. Mental
B. Mylohyoid
C. Lingual
D. Inferior alveolar

22. D 23. A 24. C

221
25. Among the following which is logical explanation that explains how
an abscessed maxillary canine cause swelling beneath the eye?
A. Lymphatics drain upward in this region
B. Apex of canine lies above the attachment of the canines and levator
labi superioris muscles
C. Infection spread through the angular vein which has no valves
D. Bone is less porous above the apex of the canine which facilitates the
spread of infection

26. Most tuberosity reductions before denture construction requires:


A. Primarily soft tissue or fibrous tissue removal
B. Large amount of bone removal
C. Relief from pterygoid hamulus
D. Some bony reductions in all instances

27. Incision and drainage (I and D) in an area of acute infection should


be performed when?
A. Induration has occurred
B. Localization has occurred
C. Acute pain is present
D. There is fever which is above 1020 F

25. B 26. A 27. B

222
28. During the apicectomy on a maxillary lateral incisor, one should take
care not to perforate or damage the:
A. Maxillary sinus
B. Labial frenum
C. Floor of the nose
D. Contents of the incisive canal

29. A mucocele of the lower lip should be treated by:


A. Incision
B. Electro-cautery
C. Excision including adjacent minor salivary gland
D. 'Incision and drainage followed by antibiotics

30. Among the following which definite sign/symptom indicate infection


of masticator space
A. Difficulty in swallowing
B. Trismus
C. Decreased salivary secretion (Xerostomia)
D. Fever above 100F

28. C 29. C 30. B


223
31. Antibiotic medication for pericoronitis should be advised:
A. If extraction is delayed or postponed
B. Before surgery
C. Routinely to treat pericoffcnitis
D. If trismus and fever present

32. Among the following which stage of anesthesia describes the level
of conscious sedation?
A. stage I
B. stage II
C. stage III plane 1
D. Stage Ill plane 2

33. Alveolectomy for prosthetic reasons should be performed:


A. During the time of extraction of teeth
B. One month after the extraction
C. 2 months after the extraction
D. At the time when the complete dentures are to be constructed

31. D 32. A 33. A

224
34. Irritational fibroma which is asymptomatic is best treated by:
A. No treatment is necessary because it is asymptomatic
B. Simple excision
C. Radiation therapy
D. Wide excision followed by chemotherapy

35. Commonly employed approach for the removal of foreign body or


root tip from the maxillary antrum is:
A. Through the canine fossa
B. Through the socket by enlarging it
C. By nasal antrostomy
D. Palatal approach

36. Nasal antrostomy usually done from this:


A. Middle concha
B. Inferior concha
C. Middle meatus
D. Inferior meatus

34. B 35. A 36. D

225
37. Commonest complication with Risdon and submandibular incision
is:
A. Damage to marginal mandibular nerve
B. Damage to cervical branch of facial nerve
C. Damage to inferior alveolar nerve
D. Damage to lingual nerve

38. To remove an undercut maxillary tuberosity, the following flap


design criteria is true:
A. Incision should be made at the crest of the ridge
B. A "wedge" should be done first
C. Incision may extend to premolar canine area
D. All of the above

39. In sutured skin incisions of the face, the edges of the skin should be:
A. Everted
B. Inverted
C. Edge to Edge
D. Separated

37. A 38. D 39. A


226
40. The following methods are commonly used methods of ridge
extension procedure of the mandible:
A. Secondary epithelialization
B. Mucosal graft vestibuloplasty
C. Skin graft vestibutoplasty
D. All of the above

41. Maxillary tori are usually removed by:


A. Burs and chisels as indicated to section and remove tabulation
B. Bone forceps to hold torus while it is being sectioned
C. Rongeurs to crush bone
D. All of the above

42. The 'Z' plasty procedure is commonly used to correct:


A. Oro-antral fistula
B. Ankyloglossia
C. Interfering labial frenum
D. B and C

40. D 41. A 42. D

227
43. At the time of extraction of upper third molar, the tuberosity is
fractured, but still attached to the periosteum. Which of the following
treatment should be employed?
A. Remove the tuberosity and file the area and suture the soft tissue
B. Remove the tuberosity and fill the defect with osteogen and suture it
C. Leave the tuberosity and stabilize it
D. None of the above

44 To drain pus from an abscess, the surgeon should:


A. Cut an eclipse from the abscess surface to allow for a drain
B. Aspirate the contents
C. Penetrate into abscess cavity and probe with an artery
D. Cut only the mucosa and skin

45. Two cm highly suspicious lesion of the floor of the mouth is to be


studied microscopically. A specimen is best obtained:
A. Incisional biopsy
B. Excisional biopsy
C. FNAC
D. None of the above

43. C 44. C 45. A

228
46. The severe complication of the canine space infection is:
A. Erosion of internal carotid artery
B. Erosion of external carotid artery
C. Cavernous sinus thrombosis
D. Respiratory paralysis

47. The treatment of choice for traumatic bone cyst is:


A. Marsupialization and leaving the cyst lining
B. Opening the cavity and inducing bleeding
C. Enucleation of the cyst lining
D. Packing with BIPP and prevent bleeding and clot formation

48. Treatment of choice to a localized infection with pus


A. Antibiotic administration
B. Establish drainage
C. Apply cold to the area
D. Advise hot mouth washes

46. C 47. B 48. B

229
49. Of the following which is most frequently indicated treatment for
odontogenic cysts:
A. Fulguration
B. Enucleation
C. Marsupialization
D. Incision and drainage

50. A patient came to dental clinic with a growth on the inside of the
cheek that is approximately 0.5 cm in diameter and not painful. He
noticed the swelling 3 months back. The best treatment is to:
A. Remove all of growth and send for biopsy
B. Observe periodically
C. Start antibiotic medication
D. Remove part of the growth and send for biopsy

51. Among the following which is treatment of choice for infection with
fluctuation in an afebrile patient?
A. Administration of antibiotics
B. Application of hot packs to the area
C. Incision and drainage
D. Antibiotics administration followed by I and D

49. B 50. C 51. C

230
52. Among the following which is a problem with reconstruction of the
atrophic ridge with hydroxy apatite granules:
A. Displaced material
B. Mental nerve dysesthesia
C. Poor ridge form
D. Any of the above

53. Which muscle should be detached to lower the floor of the mouth?
A. Mylohyoid
B. Geniohyoid
C. Genioglossus
D. A and C

54. Of the following in which condition a biopsy should not be


performed on a tissue. When the tissue?
A. Responds to local treatment
B. Is associated with paresthesia
C. Suddenly enlarges
D. B and C

52. D 53. D 54. A


231
55. Marsupialization is a surgical procedure which commonly used to
treat a\an?
A. Cystic odontoma
B. Periapical cyst
C. Large cyst
D. Oro antral fistula

56. Of the following movement should be employed to reduce a


dislocation of the mandible:
A. Downward and forward
B. Downward and backward
C. Upward and backward
D. Upward and forward

57. Papillomatosis or hyperplastic palatal tissues most often seen


beneath dentures. This condition is best treated by:
A. Not allowing the patient to wear the dentures
B. Supraperiosteal dissection or electro-surgery
C. Chemical cautery
D. Radical excision because this has malignant potential

55. C 56. B 57. B

232
58. Which of the following represent excisional biopsy?
A. Exfoliative cytologic technique
B. Including normal tissue and all of the lesion
C. Including normal tissue and most of the lesion
D. Removing a representative section of the pathological tissue or lesion
and some adjacent normal tissue for comparison

59. Of the following conditions where acrylic splints are advisable


during the postoperative phase of management:
A. Torus palatinus reduction
B. Mental tubercle reduction
C. Mandibular alveoplasty
D. None of the above

60. During the removal of a maxillary torus, the mid portion of the
palatine process of the maxilla is accidentally removed. One might see:
A. Vertical fracture of the maxilla
B. An opening into the nasal cavity
C. Opening into the maxillary antrum
D. Horizontal fracture of the maxilla

58. B 59. A 60. B

233
61. In the usual approach to mandibular tori the incision is usually
placed:
A. Parallel to the internal oblique ridge
B. Over the alveolar crest
C. Over the prominence of the tori
D. Below the mylohyoid line

62. Of the following, where antrostomy is commonly used?


A. To remove high maxillary third molars
B. To expose a tooth root for apicectomy
C. To treat an alveolar abscess
D. Following closure of an oroantral fistula

63. Which of the following may cause paresthesia of the lower lip?
A. Malposed maxillary third molar
B. Removal of a mandibular third molar
C. Removal of torus mandibularis
D. None of the above

61. B 62. D 63. B

234
64. of the following which is best method to treat a large radicular cyst
of the maxilla with apical involvement
A. Marsupialization
B. Aspiration
C. Enucleation and primary closure
D. Enucleation and packing the cavity with BIPP

65. All of the following are safely excised in preparing


A. A pendulous tori
B. Genial tubercles
C. Feather edged ridges
D. Mylohyoid ridge is best to restore a missing mandibular portion.

66. _____________ is best to restore a missing mandibular portion.


A. Homogenous bone
B. Autogenous bone
C. Silastic
D. Tantalum

64. A 65. B 66. B

235
67. Which of the following criteria should be considered before a third
molar transplantation?
A. The root is at least half formed
B. The width of the crown approaches the width of the extracted tooth
C. A and B
D. None of the above

68. Most objective way to detect sharp ridges of bone while performing
alveoloplasty is:
A. Place a finger over the body ridge and palpate
B. Place a finger over the soft tissue flap and palpate
C. See the area for sharp bony projections
D. None of the above

69. Minimum volumes of local anaesthetic agent is advised for


infiltration when excising frenum because larger volumes may?
A. Cause tissue slough
B. Distort the tissues
C. Cause rebound bleeding
D. Cause toxic effects

67. C 68. B 69. B

236
70. Of the following conditions, where an incisional biopsy is indicated?
A. 4 cm hemangioma of the tongue
B. 3 cm leukoplakia of the soft palate
C. 0.5 cm papillary fibroma of the gingiva
D. 1.5 cm exostosis of jaw

71. A negative histopathology report of a highly suspicious oral lesion


suggest?
A. No malignant potential of the lesion
B. Lesion should be stained with toludine blue like stains
C. Periodical recalls are necessary to assess the nature of the lesion
D. That another biopsy is necessary in view of the clinical impression

72. Skin graft vestibuloplasty prevents relapse by:


A. Causing an inhibitory effect on fibroblasts
B. Forms a barrier to reattachment of muscle to periosteum
C. Promoting osteogenesis and bony barrier is established
D. Relapse is common in skin graft vertibuloplasty

70. B 71. D 72. B

237
73. Among the following, where excisional biopsy is indicated?
A. 2 cm hemangioma of the tongue
B. 3 cm leukoplakia of the left palate
C. 0.5 cm fibroma of the gingiva
D. 3.5 cm area of Fordyce spot of the check

74. The soft tissue incision used to remove mandibular torus should be
placed:
A. Directly over the torus
B. Inferior to the torus in the area of the floor of the mouth
C. Over the edentulous alveolar crest in the gingival crevice aroung the
dentition
D. A and B

75. After the incisional biopsy of a lesion in the floor of the mouth and
placement of sutures, the patient experiences severe bilateral swelling of
the sub mental and submandibular spaces. This is caused by:
A. Ludwig's infection
B. Too much fluid intake postoperatively
C. Injury to lingual artery with hemorrhage
D. Extravasation into these compartments

73. C 74. C 75. D

238
Chapter 6: Cyst\Tumor\Surgical Pathology
1. Marsupialization is a surgical procedure which may be used in
treating?
A. A large cyst
B. A cystic odontoma
C. A periapical cyst of 1 cm
D. On oral antral fistula

2. A painless, fluid-filled retention cyst appearing in the area of recent


dental treatment may be the result of:
A. Failure of absorption of the anesthetic agent
B. Allergic reaction to some agent employed
C. Infection incurred during treatment
D. An injury obstructing or blocking a minor salivary gland

3. A swelling on the anterior floor of the mouth is soft and painless. It


has been present for several months. The overlying mucosa has a bluish
tingue. The most likely diagnosis is:
A. A retention cyst
B. A mixed salivary gland tumor
C. An infected periodontal abscess
D. A carcinoma of the floor of the mouth

1. A 2. D 3. A

239
4 Sialoliths are most commonly found in the:
A. Parotid duct
B. parotid gland
C. Sublingual gland
D. Submandibular duct or gland

5. Bifid mandibular canal is somewhat common in the following disease:


A. Paget’s disease
B. Fibrous dysplasia
C. Neuro fibromatosis
D. A and B

6. Of the following which nevus has the maximum potential to become


melanoma?
A. Junctional nevus
B. Blue nevus
C. Hairy nevus
D. Intradermal nevus

4. D 5. C 6. A

240
7. Of the following which group of lymph nodes one most commonly
involved in tubercular lymphadenitis?
A. Axillary
B. Cervical
C. Submental
D. Submandibular

8. Which of the following is correct regarding rodent ulcer?


A. Common site of occurrence is lateral border of the tongue
occasionally seen on the ventral surface of the tongue
B. More common in blacks
C. Antibiotics particularly penicillin is effective in the treatment of the
disease
D. None of the above are correct

9. Of the following areas, where retention cysts commonly occur?


A. Floor of the mouth
B. Upper lip
C. Lower lip
D. Buccal mucosa

7. B 8. D 9. C

241
10. Marsupialization is associated with the treatment for:
A. Abscess
B. Pericoronal infection
C. Cyst
D. Cellulitis

11. Mixed tumors of major salivary glands are most frequently found in
the:
A. Tip
B. Palate
C. Tongue
D. Parotid gland

12. The swelling of bone usually seen with a cyst is due to:
A. Ballooning of cortical plates
B. New sub periosteal deposition
C. Soft tissue reactive swelling
D. Cortical plates being reabsorbed and cystic lining causes the swelling

10. C 11. D 12. B

242
13. In an otherwise asymptomatic cystic swelling there is sudden
neurapraxia in inferior alveolar nerve region it can be due to:
A. Infection of cyst
B. Expansion of periosteum due to cyst
C. Neuritis
D. Neuralgia

14. A patient presents with a non-vital and swelling in the labial sulcus.
On aspiration straw coloured fluid is present a tentative diagnosis would
be:
A. Nasopalatine cyst
B. Solitary bone cyst
C. Keratocyst
D. Periapical periodontal cyst

15. With infected large cyst the adjoining teeth give a negative vital
response, it is:
A. Non reversible
B. Reversible after decompression
C. Extraction of the teeth is a must
D. None of the above

13. A 14. D 15. D

243
16. In following situations an artefact may simulate a cystic lesion
(except in):
A. Radiolucent area in periapical region of central incisors
B. Radiolucent area apical to mandibular premolars
C. Radiolucent area apical to maxillary canines
D. Radiolucent area in ramus of the mandible below sigmoid notch

17. To obtain better informative radiographic view of a cyst which has


eroded the cortex considerably:
A. Exposure time should be reduced
B. Exposure time should be increased
C. Voltage should be increased
D. Milliampere should be increased

18. Partsch operation is same as:


A. Marsupialisation
B. Decompression with enucleation
C. Only enucleation
D. Enucleation with marsupialisation

16. D 17. A 18. A


244
19. Fissural cysts should be ideally treated by enucleation because:
A. The cysts are smaller in size
B. They never enlarge to a size where enucleation cannot be done
C. The bone does not always regenerate from margins and fissural areas
D. Marsupialisation would leave the cavities open and these areas are
more susceptible to infection

20. Bohn's nodules are:


A. Cystic swellings in neonates
B. Cysts associated with soft palate
C. Cysts of gingiva in growing children
D. Warts on the tongue

21. The choice of treatment for keratocyst should be:


A. Marsupialisation
B. Marsupialisation with secondary closure
C. Enucleation
D. Marsupialisation with primary closure

19. C 20. A 21. C

245
22. The high recurrence rate of keratocysts is incriminated to:
A. Its fragile thin lining
B. Presence of daughter cysts in the cystic lining
C. Presence of daughter cysts in the capsule of the cyst
D. All of the above

23. The protein content of periapical and dentigerous cysts is:


A. 5-11 gm/dl
B. Less than that of serum
C. More than that of serum
D. Less than 4 gm/dl

24. The window for marsupialisation shows maximum contracture


when:
A. It consists of sulcus mucosa supported by loose connective tissue
B. It consists of sulcus mucosa with firm periosteal bed loosely adherent
to bone
C. It consists of mucoperiosteum firmly adherent to underlying bone
D. Cyst is large

22. D 23. A 24. A


246
25. Complete enucleation of cyst in palatal area carries danger of:
A. Excessive bleeding from nasopalatine artery
B. Severing of nasopalatine nerve
C. Tear of nasal mucosa
D. Alteration of speech

26. In cysts of maxilla involving maxillary sinus marsupialization pack


should be changed from:
A. Nasal antrostomy
B. Caldwell-luc operation
C. Palatal opening
D. From tooth socket

27. In a 48-year-old patient the treatment of dentigerous cyst with


impacted molar lying near lower border of mandible, would be:
A. Enucleation with primary closure and IMF
B. Marsupialization with extraction of molar
C. Marsupialization with IMF
D. Enucleation with secondary closure

25. C 26. A 27. C


247
28. In which of the situation(s) the cystic lining would have become
thick and adherent?
A. Infection
B. Already decompressed earlier
C. Tooth has been extracted without treating the cyst
D. All of the above

29. During enucleation the incision should be placed on:


A. Firm bony base
B. Mucosa only
C. Cystic lining
D. Cystic capsule

30. If during enucleation of a cyst the apices of adjacent normal teeth


become exposed:
A. Retrograde filling should be done with calcium hydroxide
B. They should be treated endodontically with apicectomy
C. Dressing and secondary closure should be done
D. If vital they should be observed for three months

28. D 29. A 30. D


248
31. One of the main reasons for break-down of suture line after primary
closure is:
A. Dead space
B. Hematoma formation
C. Saliva seepage
D. Retaining adjacent teeth

32. The best material to be packed in large bony cavity after enucleation
of cyst is:
A. Allogenic bone
B. Hydroxy apatite crystals
C. Autogenic medullary bone chips
D. Autogenic cortical pieces

33. Cysts from the following teeth usually expand palatally (except):
A. Maxillary lateral incisors
B. Roots of maxillary premolars
C. Upper molars
D. Upper central incisors

31. B 32. C 33. D


249
34. An early stage cementoma may be erroneously diagnosed as a
periapical cyst but for the:
A. Position of lesion
B. Vitality of tooth
C. Radiopacity of the margin
D. Age of the patient

35. Lateral developmental periodontal cyst is typically present:


A. In relation to vital teeth
B. In relation to submental space
C. In relation to pulpless maxillary teeth only
D. Supernumerary non vital teeth

36. Eruption cysts should be treated:


A. immediately with enucleation
B. By marsupialization
C. With no active treatment
D. With antibiotics.

34. B 35. A 36. C

250
37. The protein content of keratocyst is found to be:
A. < 4 gm\dl
B. 6 gm\dl
C. Equal to serum protein
D. More than serum protein

38. Treatment of keratocyst is:


A. Marsupialisation
B. Enucleation
C. Excision
D. Enucleation with secondary closure

39. A 36-year-old patient presented with an asymptomatic swelling on


left side of body of mandible, radiograph shows small radiopaque specks
within the bone cavity and on aspiration straw colored fluid was present.
It is a typical picture of:
A. Amaeloblastic adenomatoid tumor
B. Calcifying epithelial odontogenic cyst
C. Keratocyst
D. Cystic odontoma

37. A 38. C 39. B

251
40. A 26-year-old male patient presented with multiple keratocysts,
basal cell carcinoma on right cheek and Dyskeratosis with bifid rib.
Diagnosis would constitute:
A. Gorlin's cyst
B. Gorlin's syndrome
C. Marfan's syndrome
D. Pierre Robin syndrome

41. Globulomaxillary cyst occupies bony region between:


A. Maxillary central incisor and lateral incisor
B. Maxillary lateral incisor and canine
C. Maxillary canine and premolar
D. 1st and 2nd premolar of maxilla

42. 18-year-old boy presented with swelling in labial sulcus, difficulty in


breathing, skiagram shows no radiolucent lesion of the bone and on
aspiration straw coloured fluid was present. It could be:
A. Medial palatine cyst
B. Nasoalveolar cyst
C. Nasal polyp
D. Nasopalative duct cyst

40. B 41. B 42. B

252
43. A patient 14 years of age presented with swelling on right mandible,
the adjacent teeth were vital. Radiologically there was an extensive
radiolucent lesion with scalloped margin extending between the roots
and lamina Dura was intact. On aspiration golden yellow coloured fluid
was present, a tentative diagnosis can be:
A. Aneurysmal bone cyst
B. Hemorrhagic bone cyst
C. Stafne's bone cyst
D. Static bone cyst

44. Traumatic bone cyst is also known as:


A. Solitary bone cyst
B. Hemorrhagic bone cyst
C. Static
D. A and B

45. Stafne's bone cyst is a:


A. True cyst
B. Bony depression above inferior alveolar canal
C. Bony depression below inferior alveolar canal
D. Radiolucent area below inferior alveolar canal

43. B 44. D 45. C\D

253
46. The static bone cyst should be treated by:
A. Enucleation
B. Marsupialisation
C. No active treatment
D. Exploration and closure

47. An example of retention cyst is:


A. Mucocele
B. Ranula
C. Dermoid cyst
D. Branchial cyst

48. Mucocele should be treated by:


A. Marsupialisation
B. Enucleation of cyst
C. Enucleation of the cyst and the minor salivary gland
D. Decompression only

46. C 47. B 48. C

254
49. Trauma to the excretory ducts of sublingual salivary glands causes:
A. Ranula
B. Mucocele
C. Solitary cyst
D. Sialolithiasis

50. Following cysts occur on lateral side of neck:


A. Dermoid cyst
B. Thyroglossal cyst
C. Branchial cyst
D. Epidermoid cyst

51. A patient presented with a small cystic swelling in anterior region of


neck which moved on swallowing and on protrusion of tongue, it could
be:
A. Dermoid cyst
B. Thyroglossal cyst
C. Epidermoid cyst
D. Branchial cyst

49. A 50. C 51. B


255
52. A 33-year-old patient reported with an extensive ameloblastoma of
mandible but the lower border was not involved, the treatment should
be:
A. Curettage
B. En-block resection
C. Segmental resection
D. Hemimandibulectomy

53. The recommended treatment modality for Pindborg's tumour is:


A. Curettage
B. Enucleation
C. Excision/ resection
D. Marsupialisation with secondary closure

54. Adenoameloblastoma should be managed by:


A. Enucleation
B. En-block resection
C. Segmental resection
D. Hemi/ partial mandibulectomy

52. B 53. C 54. A


256
55. When performing curettage in aggressive tumors such as
ameloblastoma or Pindborg's tumor, one
A. Not sacrifice vital structures present in the area
B. Sacrifice the vital structures present in the area
C. Both tumors are not aggressive
D. None of the above

56. The surgical management of Brown's tumor of mandible is:


A. En-block resection
B. Segmental resection
C. Parathyroidectomy
D. No treatment

57. A patient presented with a radiolucent lesion and biopsy report


shows giant cells the lesion could be:
A. Giant cell granuloma
B. Brown tumor
C. Cherubism
D. All of the above

55. B 56. C 57. D


257
58. 8-year-old boy presented with bilateral swelling of mandible which
was asymptomatic and slowly progressive in nature, radiologic picture
had extensive bilateral multilocular radiolucencies in posterior
mandibular angle and body, this is a characteristic picture of:
A. Hand-Schüller-Christian disease
B. Letterer-Siwe disease
C. Cherubism
D. Eosinophillic granuloma

59. A hypertensive patient having cafe au lait spots, hypoplastic maxilla


and signs of mental deficiency presented with a soft tissue mass on
buccal mucosa, this should insist surgeon to investigate for:
A. Neurofibromatosis
B. Neurilemoma
C. Down syndrome
D. Pierre Robin syndrome

58. C 59. A
258
60. A 35-year-old patient with history of trauma complained of
intermittent pain in mandible with areas of paresthesia. X-ray picture
revealed a radiolucent area extending into the inferior alveolar canal, on
aspiration no fluid/ gas was present, one can suspect:
A. Hemorrhagic bone cyst
B. Aneurysmal bone cyst
C. Traumatic neuroma
D. None of the above

61. Incision for removal of a palatal torus should be placed:


A. In the midline
B. Paramedian
C. From crevices of teeth
D. As an envelope flap

62. Following method/methods can be used for treating oral leukoplakia:


A. Excision
B. Cryotherapy
C. Fulguration
D. All of the above

60. C 61. A 62. D

259
63. Treatment of pyogenic granuloma consists of:
A. Antibiotics and analgesics
B. Excision with removal of teeth
C. Excision without removal of teeth
D. None of the above

64. Peripheral giant cell granuloma should be treated by:


A. En-block resection
B. Excision with removal of teeth
C. Excision without removal of teeth
D. Segmental resection

65. If after extracting a mandibular tooth, one encounters bleeding due to


an underlying hemangioma the first step in treatment would be:
A. Carotid artery ligation
B. Inferior alveolar artery ligation
C. Replacing the tooth in the socket
D. Pressure packing

63. C 64. B 65. C

260
66. Chondromas are:
A. Radiosensitive
B. Radiopaque
C. Radioresistant
D. Only present in condylar area

67. The management of ossifying fibroma consists of:


A. En-block resection
B. Curettage / enucleation
C. Segmental resection
D. Cryotherapy

68. A 14-year-old female patient presented with swelling on (right) side


of face in maxillozygomatic area. The swelling has been slowly
progressive in nature, radiograph shows diffuse radiopaque mass
involving maxillary sinus and zygoma, with a typical 'ground glass'
appearance this is characteristic picture of:
A. Osteoma
B. Ossifying fibroma
C. Fibrous dysplasia
D. Osteosarcoma

66. C 67. B 68. C

261
69. The recommended treatment for fibrous dysplasia is:
A. Curettage for contouring
B. Resection en-block
C. Radiotherapy
D. Excision

70. The tumors which are poorly differentiated are:


A. Radiosensitive
B. Radioresistant
C. Radioatropic
D. Radiorefractive

71. A protein level between 5-11 g/dl in a cystic fluid is:


A. A dentigerous cyst
B. A keratocyst
C. A radicular cyst
D. A mucocele

69. A 70. A 71. C

262
72. The Reharmann flap is used to:
A. Close an oronasal fistula
B. Gain access to the TM Joint
C. Close an oro-antral fistula
D. None of the above

73. The commonest etiological factor in the development of T M Joint


ankylosis in children is:
A. Infection
B. Tetanus
C. Trauma
D. Rheumatoid arthritis

74. The preferred treatment of a giant cell lesion 2.5 cm in diameter, in


the mandibular anterior region is:
A. Electrocauterization
B. Wide radical excision
C. Curettage
D. Block excision

72. A 73. C 74. C


263
75. The following cyst has the highest rate of recurrence:
A. Radicular cyst
B. Primordial cyst
C. Keratocyst
D. Dentigerous cyst

76. Radiographic examination is useful in detecting all the following


cysts, except the:
A. Follicular cyst
B. Nasopalatine cyst
C. Residual cyst
D. Nasoalveolar cyst

77. "Partsch No. Il operation" is the name given to:


A. Enucleation
B. Marsupialization
C. Excision of a ranula
D. Combination of (A) and (B) above

75. C 76. D 77. A

264
78. A 25-year-old lady reports to you with a diffuse, non-progressive
and a non-tender swelling of the right maxilla, approximately 2 x 1.5
cm, extending from the canine to the first molar region. The X-ray
shows a ground glass" appearance of the bone in the concerned area.
The surgical treatment will be:
A. Total excision of the lesion
B. Curettage of the area with extraction of the involved teeth
C. Surgical cosmetic recontouring only
D. Surgical excision followed by radiotherapy

79. "T3" in the TNM staging of the oral malignant lesions represents:
A. Carcinoma in situ
B. Tumour 2 cm or less in greatest diameter
C. Tumour > 2 cm but < 4 cm in greatest diameter
D. Tumour > 4 cm in greatest diameter

80. While performing a submandibular sialography the cannula is


inserted in the following direction to the Wharton's duct:
A. Upward and medical
B. Medial to lateral
C. Lateral to medial
D. Lateral and downward

78. C 79. D 80. B


265
81. Factors leading to difficulties in cannulation while performing
sialography are, except:
A. Short or blunt bevel on tubing
B. Rough bevel on tubing
C. Too large a caliber tubing
D. Excessive dilation of the duct orifice

82. In trans-oral sialolithotomy of the submandibular duct the incision


for removal of an anterior sialolith should be placed:
A. Medial to the plica
B. Lateral to the plica
C. Directly over the duct
D. Lateral to the duct

83. "Ageusia" refers to the:


A. Loss of hearing
B. Excessive sensitivity to touch
C. Lowered pain sensitivity
D, Loss of taste

81. D 82. A 83. D


266
84. A patient with an adenoid cystic carcinoma of the parotid will
present with all the following symptoms except:
A. Swelling of the parotid region
B. Fixation to the underlying tissues
C. Iritis
D. Drooping of the oral commissure

85. Steven-Jonson syndrome, a variant of erythema multiform, involves:


A. Oral lesions
B. Cuteneous genital lesions
C. Nephritis
D. Oral lesions & Iritis

86. Hormonal disturbance is the primary cause of jaw deformity in


patients suffering with:
A. Paget's disease
B. Achondroplasia
C. Acromegaly
D. None of the above

84. C 85. D 86. C


267
87. Ground glass appearance in X-rays may suggest the patient is
suffering with:
A. Hyperparathyroidism
B. Fibrous dysplasia
C. Paget's disease
D. None of the above

88. Which of the following condition associated with EB virus?


A. Common cold
B. Measles
C. Burkitt's lymphoma
D. Poliomyelitis

89. 16-year-old patient has a painful swelling which is firm in


consistency in the mandibular premolar area. Teeth near the swelling are
vital. X-ray reveals a discrete radiolucency in the premolar area. The
next diagnostic step is:
A. Aspiration
B. Biopsy
C. Complete RBC and WBC count
D. Bone scans

87. B 88. C 89. A

268
90. Bifid ribs, multiple radiolucent lesions of the jaws, multiple basal
cell nevi and falx cerebri calcification are found in:
A. Basal cell nevus syndrome
B. Sturge-Weber syndrome
C. Horners syndrome
D. Hereditary intestinal polyposis

91. Fordyces disease is due to:


A. Hyperkeratosis
B. Capillary fragility
C. Aberrant sebaceous glands
D. Cholesterol deposits

92. Rodent ulcer or basal cell carcinoma is usually present in the:


A. Intraorally lateral border of the tongue
B. Upper third of the face
C. Middle third of the face
D. Lower third of the face

90. A 91. C 92. C

269
93. Needle aspiration of a central bone lesion is useful to:
A. Rule out a vascular lesion
B. To determine thickness of buccal plate
C. To diagnose traumatic bone cyst
D. Feel for root surfaces

94. 60-year-old women with a past history of adenocarcinoma of the


breast complaining of an ulcer on the right lateral border of her tongue.
Fractured cusp on the mandibular right molar which exposes a sharp
margin of dental amalgam is found during routine clinical examination.
Treatment of choice is:
A. Immediate biopsy of the lesion
B. Restoration of the tooth with biopsy in 2 weeks if necessary
C. Restoration of the tooth
D. Restoration of the tooth with biopsy in 2 months, if the lesion does
not heal

95. Of the following regions where ameloblastoma frequently occur?


A. Mandibular molar region
B. Maxillary molar region
C. Mandibular premolar region
D. Maxillary premolar region

93. A 94. B 95. A


270
96. During routine radiographic examination one found pear shaped
radiolucency maxillary canine and lateral incisors region. The teeth are
vital most probable diagnosis is:
A. Radicular cyst
B. Incisive canal cyst
C. Globulomaxillary cyst
D. Primordial cyst

97. The best treatment for cementoma is:


A. No treatment
B. Endodontic treatment
C. Extraction of the teeth
D. Resection of the involved area

98. During the examination of radiograph of a 60-year- old male patient,


one finds periapical radiolucencies near mandibular incisors. The
diagnosis include:
A. Periapical cyst
B. Early cementomas
C. Paget's disease
D. A and B

96. C 97. A 98. D

271
99. In the clinical evaluation, the most significant finding of a parotid
mass may be accompanying:
A. Slow progressive painless enlargement
B. Nodular in consistency
C. Submental and preauricular lymphadenopathy
D. Facial paralysis

100. In a radiograph radiopaque line around the pathology:


A. Slow growing pathology
B. Rapidly growing pathology
C. No correlation to the growth
D. Malignant lesion with rapid growth

101. During the radiographic examination of maxillary central incisors


one notices a radiolucency between the two central incisors. It is most
likely:
A. Nasopalatine cyst
B. Primordial cyst
C. Globulomaxillary cyst
D. Lateral periodontal cyst

99. D 100. A 101. A

272
102. A which of the following condition one might see multiple
odontogenic cysts?
A. Cleido cranial dysostosis
B. Paget's disease
C. Craniofacial dysostosis
D. Marfan's syndrome

103. During examination of a panoramic X-ray of 65-year-old patient,


one noticed 1cm Lytic lesion in the mandible. Which of the following is
the possible diagnosis?
A. Hyperparathyroidism
B. Ca prostate
C. Aneurysmal bone cyst
D. All of the above

104. Large soft swelling noticed in the floor of the mouth.


It could be:
A. Thyroglossal duct cyst
B. Mucous retention cyst
C. Dermoid cyst
D. All of the above

102. D 103. D 104. D

273
105. Nikolsky's sign helps in the diagnosis of:
A. Herpetic stomatitis
B. Erythema multiforme
C. Pemphigus vulgaris
D. Drug allergy

106. Which of the following is most common site for sialoliths?


A. Parotid duct
B. Submandibular duct
C. Minor salivary gland
D. Parotid gland

107. Of the following where sialoliths are most common?


A. Parotid gland
B. Parotid duct
C. Submandibular duct
D. Sublingual gland

105. C 106. B 107. C

274
108. Of the following which is true? "Port wine stain" is a:
A. A type of stain occasionally used by histopathologist
B. Commonly used contrast media for better image by
C. A type of hemangioma
D. None of the above are correct

109. Which of the following is correct regarding Bowen's disease of the


skin?
A. A type of dermatitis, sometimes affects the eye
B. A pre malignant condition/ lesion
C. Is due to some abnormalities in sweat gland
D. Occur in mercury poisoning

110. Which of the following may cause a painless fluid filled retention
cyst appearing in the area of recent dental treatment?
A. Allergic reaction to some agent employed
B. Infection
C. An injury obstructing a minor salivary gland
D. Failure of absorption of the anesthetic gland

108. C 109. B 110. C


275
111. A bluish soft and non-tender swelling on the anterior floor of the
mouth is noted. It has been present for several months. Most likely
diagnosis is:
A. Mixed salivary gland tumor
B. An infected periodontal abscess
C. A retention cyst
D. Carcinoma of the floor of the mouth

112. All of the following are seen in cleidocranial dysostosis except:


A. Absence or hypoplasia of clavicles
B. Pseudo or false prognathism
C. Premature eruption of teeth
D. Bulging forehead and prominent cranial bones

113. One might see Cafe au lait spots in the following conditions except:
A. Tuberous sclerosis
B. Sjögren's syndrome
C. von Reckling Hausen's disease
D. Albright's syndrome

111. C 112. C 113. B


276
114. Most likely cause of tender swelling in the submandibular triangle
is:
A. Obstruction to stenen's duct
B. Lipoma
C. Lymphadenopathy
D. All of the above

115. Of the following which denotes the inflammation of the salivary


duct?
A. Pulmonary embolism
B. Acute myocardial infarction
C. Thrombophlebitis
D. All of the above

116. A patient is under oral anticoagulants such as warfarin therapy.


Most probably he has:
A. Pulmonary embolism
B. Acute sialadenitis
C. Chronic sialadenitis
D. Sialodochitis

114. C 115. D 116. D

277
117. Of the following which is characteristic of the traumatic
(hemorrhagic) bone cyst?
A. It causes root resorption
B. Rarely expands cortices or displaces teeth
C. Devitalized involved teeth
D. Is usually found in the maxilla

118. Ameloblastoma is commonly seen in which of the following areas?


A. Antrum and floor of the nose
B. Symphysis area of the mandible
C. Molar area of the ramus of the mandible
D. Mandibular premolar area

119. In a routine examination of an IOPA one noticed the following,


Ground glass appearance of bone, complete loss of lamina Dura, Loss of
trabecular pattern of bone
The most probable diagnosis is:
A. Hypercalcemia
B. Hyperparathyroidism
C. Fibro-osseous lesion
D. Hypocalcemia

117. B 118. C 119. B

278
120. A 60-year-old man complaining some loss of hearing, some loss of
visual acuity and enlargement of his maxilla. The above findings suggest
he is most probably suffering with:
A. Paget's disease
B. Multiple myeloma
C. Ossifying fibroma
D. Ameloblastoma

121. In routine radiological examination one notices a pear-shaped


radiolucency located between the maxillary lateral incisor and canine.
Most probably it would be:
A. Nasoalveolar cyst
B. Median palatine cyst
C. Globulo maxillary cyst
D. Naso palatine duct cyst

122. Which of the following may cause fixation of a non-tender soft


tissue mass to surrounding tissues?
A. Benign tumor
B. An inflammatory process
C. Malignancy
D. Lesion arising from bone

120. A 121. C 122. C


279
123. In routine radiological examination one found radiolucency near
angle of the mandible and below the inferior alveolar canal. Most
probably it is:
A. A cyst
B. A developmental defect
C. A cavernous hemangioma
D. Granuloma

124. Globulomaxillary cyst is a type of:


A. Dentigerous cyst
B. Idiopathic
C. Fissural cyst
D. Primordial

125. Radiographs of the maxillary central and lateral incisors disclose a


heart shaped radiolucency in the midline. Teeth next to the radiolucency
are vital, this is most probably:
A. Nasopalatine duct cyst
B. Globulomaxillary cyst
C. Nasoalveolar cyst
D. Median palatine duct cyst

123. B 124. C 125. A

280
126. Fibrous dysplasia can be treated by:
A. Irradiation of the lesion
B. Surgical excision
C. Conservative surgery
D. Removal of the adjacent teeth

127. Among the following which is most frequent complication of sub


apical orthodontic surgery?
A. Nonunion of segments
B. Devitalization of teeth
C. Avulsion effect
D. Vertical fracture of the roots

128. Ameloblastoma of the jaw can best be treated by:


A. Excision
B. Surgical resection followed by cauterization
C. Enucleation
D. Irradiation

126. C 127. B 128. B

281
129. Which of the following lesion has no epithelial lining?
A. Nasopalatine cyst
B. Nasolabial cyst
C. Aneurysmal bone cyst
D. Follicular cyst

130. Among the following which is fissural cyst and entirely located in
soft tissue?
A. Nasoalveolar cyst
B. Globulomaxillary cyst
C. Median alveolar cyst
D. Primordial cyst

131. Osteitis deformans is the condition in:


A. Which roots have hypercementosis
B. Which anodontia seen
C. Which supernumerary roots are more common?
D. None of the above

129. C 130. A 131. A

282
Chapter 7: Facial Space Infection\Osteomyelitis
1. The hard, firm condition of the tissues is termed:
A. Cellulitis
B. Purulence
C. Coagulum
D. Abscess

2. Treatment at the first appointment for patient with cellulitis should


include:
A. Antibiotics and heat only
B. Extraction of the offending tooth
C. Antibiotics, heat and fluids
D. Incision and drainage

3. The patient with cellulitis after giving heat, antibiotics, and fluids
returns 24 hours later. On palpation, the area is soft painful to touch, and
the tissue rebounds when palpated. This condition is termed:
A. Abscess
B. Cellulitis
C. Resorption
D. Induration

1. A 2. C 3. A
283
4. Treatment for the patient with abscess should include:
A. Aspiration
B. Antibiotics and heat only
C. Incision and drainage
D. Antibiotics, heat and fluids

5. If one treats an abscess with antibiotics without I and D it may cause:


A. Non-suppurative inflammatory reaction
B. Formation of antibioma
C. Healing with scar formation
D. None of the above

6. Of the following which organism is commonly associated with sub-


acute bacterial endocarditis of dental origin?
A. Staphylococcus aureus
B. Streptococcus viridans
C. Beta hemolytic streptococcus
D. Lacto bacillus acidophilus

4. C 5. B 6. B

284
7. Usually acute pyogenic bacterial infection produce
A. Neutropenia
B. Lymphocytic leukocytosis
C. Neutrophilic leukocytosis
D. Eosinophilia

8. Koplik's spots are oral manifestations of


A. Smallpox
B. Rubeola
C. Rubella
D. Scarlet fever

9. Classical Ludwig's angina has bilateral involvement of the following


spaces except:
A. Submandibular
B. Sublingual
C. Submental
D. Lateral pharyngeal

7. C 8. B 9. D

285
10. Osteoradionecrosis occurs due to damage to which of the following
structures?
A. Blood vessels
B. Salivary glands
C. Muscle
D. Nerves

11. In a 19-year-old patient with a swelling over the left angle of the
mandible, temperature 380 C and negative history of trauma, one should
suspect:
A. Spontaneous fracture of the mandible
B. Pericoronal infection
C. Parotid tumor
D. Submaxillary gland tumor

12. Infection from a maxillary first molar spreads most often to which
space:
A. Infratemporal
B. Retropharyngeal
C. Submandibular
D. Buccal

10. A 11. B 12. D

286
13. Cavernous sinus thrombosis following extraction of acutely infected
maxillary anterior teeth is most often due to spread of infection along:
A. The anterior facial, angular, ophthalmic veins
B. The pterygoid plexus and inferior ophthalmic vein
C. The pterygoid plexus and superior ophthalmic vein
D. Not likely by any of these routes

14. The greatest deterrent to normal wound healing is:


A. Trauma
B. Infection
C. Metabolic disease
D. Nutritional deficiency

15. Facial spaces are filled by:


A. Loose connective tissue
B. Elastic fibres
C. Loose adipose tissue
D. Dead space

13. A 14. B 15. A

287
16. The characteristic features of infection of masticator space is:
A. Swelling
B. Draining pus intraorally
C. Trismus
D. High grade fever

17. The infections of masticator space do not enter into neck because:
A. The fascia is tenaciously adherent to mylohyoid line
B. The fascia is firmly adherent to periosteum lower border of mandible
C. Before it reaches the neck it follows path of least resistance to open
extraorally or intraorally
D. Masticator space is not continuous with spaces in neck

18. The infection of masticator space can spread to except:


A. Temporal pouches
B. Lateral pharyngeal space
C. Sublingual space
D. Submandibular space

16. C 17. B 18. D

288
19. Swellings of masticator space and lateral pharyngeal space are
similar. The distinctive difference is that masticator space infection:
A. Is of dental origin
B. Is not pushed towards the midline
C. Is more diffuse and visible from outside
D. Has a tendency to spread to temporal pouches

20. Incision and drainage of masticator space should be attempted:


A. At region anterior to masseter muscle
B. Intraorally from buccal sulcus
C. Extraorally in subangular region
D. From pterygomandibular raphe

21. Infections from mandibular 1st molar would travel to:


A. Submandibular space
B. Sublingual space
C. Masticator space
D. Digastric space

19. B 20. C 21. B

289
22. The major structures present in the submandibular space are:
A. Deep part of submandibular gland, branches of facial artery, lingual
nerve
B. Superficial part of submandibular gland, branches of facial artery and
lingual nerve
C. Superficial part of submandibular gland, branches of facial artery,
mylohyoid nerve
D. Submandibular duct, lingual nerve and hypoglossal nerve

23. In Ludwig's angina the classical sign is:


A. Tongue is raised and falls back causing respiratory embarrassment
B. That submandibular, sublingual and submental spaces are involved
though tongue may not be raised
C. That submandibular, sublingual and sub-mental spaces are involved
bilaterally
D. Board-like brawny induration of mandible with tongue falling back
and causing respiratory embarrassment

24. In Ludwig's angina the incision should be placed deep up till:


A. Mylohyoid muscle
B. Anterior belly of digastric
C. Geniohyoid
D. Mucous membrane of floor of mouth

22. C 23. C 24. D

290
25. Infections of lateral pharyngeal space travel usually from:
A. Temporal pouches
B. Masticator space
C. Sublingual space
D. None of the above

26. Infections of lateral pharyngeal space are life-threatening because


there may be dangers of:
A. Thrombosis of IJV
B. Erosion of ICA
C. Oedema of larynx
D. All of the above

27. Infections from lateral pharyngeal space can traverse


A. Anterior mediastinum
B. Middle mediastinum
C. Posterior mediastinum
D. Only superior mediastinum

25. B 26. D 27. C

291
28. Infections from submandibular space and sub-mental space usually
traverses to:
A. Anterior mediastinum
B. Middle mediastinum
C. Posterior mediastinum
D. Only superior mediastinum

29. Infections which travel from masticator space to parotid space are
very painful because:
A. Facial nerve is irritated
B. The capsule of parotid does not give way for the developing infection
to spread
C. Auriculotemporal nerve is irritated by infection
D. None of the above

30. While giving an inferior alveolar nerve block, infection is transposed


to:
A. Pterygopalatine fossa
B. Pterygomandibular space
C. Submandibular space
D. Masticator space

28. A 29. B 30. A

292
31. While giving posterior superior alveolar nerve block, infection may
be instituted into:
A. Pterygomandibular space
B. Infratemporal fossa
C. Temporal pouches
D. Pterygopalatine fossa

32. A patient, presented with ophthalmoplegia and signs of meningitis


after extraction of upper central incisor, could be diagnosed as due to:
A. Tumor of pituitary
B. Tuberculous meningitis
C. Cavernous sinus thrombosis
D. No relation

33. The diagnostic sign/s which Eagleton characterized for cavernous


sinus thrombosis is/are:
A. Known site of infection
B. Paresis of Ill, IV, VI nerves
C. Proptosis of eye and (B)
D. All of the above

31. A 32. C 33. D

293
34. Dissecting subperiosteal abscess develops:
A. Immediately after 3rd molar extraction on lingual side
B. Several weeks later and distant to site of 3rd molar extraction
C. In association of post extraction infection in buccal area of extracted
3rd molar
D. When extensive dissection is done while extracting an impacted tooth

35. An acute alveolar abscess should be treated with:


A. First antibiotics for three days and then incision and drainage
B. Incision and drainage with broad spectrum antibiotic
C. Broad spectrum antibiotics and analgesics
D. Antibiotics and proteolytic drugs as chymotrypsin

36. Osteomyelitis begins as an inflammation of:


A. Cortical bone
B. Periosteum
C. Medullary bone
D. Periosteum and inner cortex

34. B 35. B 36. C

294
37. Which of the following conditions are susceptible to osteomyelitis?
A. Paget's disease
B. Fibrous dysplasia
C. Radiation
D. All of the above

38. Osteomyelitis is more common in:


A. Maxilla
B. Mandible
C. Zygoma
D. Nasal complex

39, Osteomyelitis is caused most commonly by:


A. Streptococcus
B. Staphylococcus
C. M. tuberculosis
D. E. coli

37. D 38. B 39. B

295
40. In osteomyelitis how much bone should have been destroyed before
it manifests radiologically:
A. 10-12%
B. 15%
C. 30-60%
D. 80%

41. Moth eaten appearance so characteristically seen in the radiographs


of osteomyelitis is due to:
A. Presence of sequestrum
B. Enlargement of medullary spaces
C. Reduced medullary spaces
D. Narrowing of Volkmann's canals

42. Treatment of chronic osteomyelitis consists of:


A. Culture sensitivity and prolonged antibiotic therapy
B. Culture sensitivity with antibiotic therapy and hyperbaric oxygen
therapy
C. Sequestrectomy, surgical exploration and prolonged antibiotic
therapy after culture sensitivity
D. Sequestrectomy, antibiotics after sensitivity and hydrocortisone
therapy

40. C 41. B 42. C


296
43. Saucerisation as a surgical treatment for osteomyelitis connotes:
A. Complete removal of decayed bone with primary closure of wound
B. Trimming or excision of margins of necrotic bone overlying focus of
osteomyelitis and allowing secondary healing
C. Creating a saucer shaped defect by excision of the defect with
primary closure
D. None of the above

44. In treating osteomyelitis with hyperbaric oxygen:


A. 30% oxygen is used at 1 atm
B. 70% oxygen is used at 2 atm
C. 100% oxygen is used at 3 atm
D. 80% oxygen is used at 2 atm

45. A40-year-old patient presented with multiple extraoral sinuses with


yellowish discharge and with history of intermittent remission after
antibiotics treatment two months ago. The disease started after
extraction of 2nd mandibular premolar. It is suggestive of:
A. Tubercular osteomyelitis
B. Actinomycosis
C. Subperiosteal Garre's osteomyelitis
D. Dissecting subperiosteal abscess

43. B 44. C 45. B


297
46. Treatment of "alveolar osteitis" consists of:
A. Curettage of the socket to induce fresh bleeding in the socket
B. Prescription of antibiotics specific for gram-negative microorganisms
C. Gentle removal of debris, irrigation of the socket and placement of an
obtundent dressing
D. Irrigating the socket with 5% povidone-iodine

47. Surgical management of Ludwig's angina includes:


A. Incision and drainage
B. Incision and decompression
C. Drainage through closed dwelling catheters
D. None of the above

48. When a canine space infection has proceeded from cellulitis to


fluctuance, which is apparent visibly and digitally, and the patient is in
great pain, the initial treatment should be:
A. Administering parentral antibiotics
B. Application of hot packs over the region
C. Incision and drainage
D. Giving an infraorbital nerve block to provide pain relief

46. C 47. B 48. C

298
49. Management of ecchymosis following oral surgery includes:
A. Application of cold packs over the area
B. Administration of steroids and enzymes
C. Aspiration with a wide bore needle
D. None of the above

50. The major local factor that predisposes mandibular bone to


osteomyelitis is:
A. Reduced cellular proliferation
B. Reduced lymphatic circulation
C. Reduced vascular supply
D. Reduced antibody formation

51. Complications of lateral pharyngeal space infection include:


A. Thrombosis of the internal jugular vein
B. Erosion of the internal carotid artery
C. Respiratory paralysis
D. All of the above

49. A 50. C 51. D

299
52. Infection from a horizontally impacted mandibular third molar will
initially involve the:
A. Submandibular space
B. Sublingual space
C. Pterygomandibular space
D. Masseteric space

53. While draining a submandibular abscess, the surgeon should:


A. Increase the overlying tissues with a no. 10 blade
B. Aspirate the contents, and put in a vacuum drain to evacuate the
remaining pus
C. Use the Hilton's method to completely evacuate the pus
D. Always perform the procedure under general Anesthesia

54. In Marx's protocol for the management of osteoradionecrosis of the


mandible using hyperbaric oxygen if tissue dehiscence occurs after the
initial 30 dives and then:
A. The mandible is resected and 30 more dives are given
B. The mandible is resected and the dives are discontinued
C. The full course of 60 dives is given followed by the resection of
mandible
D. The full course of 60 dives is given followed by additional 20 dives
after 10 weeks

52. C 53. C 54. A

300
55. According to BIRN's hypothesis the severe pain associated with dry
socket is due to:
A. Release of plasmin
B. Release of kinin from the degenerative clot
C. Thermal irritation of the exposed nerve ending of the alveolar bone
D. B and C

56. Which is the potential primary hazard to a patient under


propylthiouracil therapy and whose laboratory data indicates hematocrit
value of 45 percent, reverse neutropenia, reduced myeloblasts in bone
marrow, clotting time of 8 min?
A. Thyroiditis
B. Hemorrhage
C. Hypoxia
D. Infection

57. The following are the characteristic features of


Ludwig's angina:
A. Raised tongue
B. Bilateral submandibular swellings
C. Dysphagia
D. All of the above

55. D 56. D 57. D

301
58. A teenage girl presents with pain, trismus swelling associated with a
partially erupted lower third molar.
The most likely diagnosis is:
A. Peritonsillar abscess
B. Pericoronitis
C. Ameloblastoma
D. Fracture of the mandible near angle region

59. Which of the following may result from acute pyogenic bacterial
infections?
A. Lymphopenia
B. Leukopenia
C. Lymphocytosis
D. Leukocytosis

60. After radiotherapy bone is more prone for infection.


This is probably related to:
A. Decalcification
B. Anaemia
C. Invasion by malignant cells
D. Endarteritis of small blood vessels

58. B 59. D 60. D


302
61. Of the following which is not a component of masticator space?
A. Submandibular space
B. Sub masseteric space
C. Deep temporal pouch
D. Superficial temporal pouch

62. Dental infection from the mandibular first molar spreads to the:
A. Submental space
B. Buccal vestibular space
C. Infratemporal space
D. Pterygomandibular space

63. Specimen for bacteriologic examination and diagnosis should be


collected:
A. Prior to the development of the symptoms
B. One or two days after the antibiotic treatment
C. During the acute stage of the disease
D. During the convalescence

61. A 62. B 63. C

303
64. Local factors that predispose bone to osteomyelitis are related
mainly to:
A. Reduced antibody formation
B. Potent endotoxins
C. Reduced blood supply
D. Increased lymphatic circulation

65. Osteomyelitis of the TMJ can be treated by:


A. Condylectomy
B. Irradiation
C. Incision and drainage, if needed condylectomy
D. None of the above

66. Among the following which anaerobic organism is frequently


responsible for oral and facial infection?
A. Staphylococcus aureus
B. Staphylococcus albus
C. Bacteroids fragilis.
D. Streptococcus viridans

64. C 65. C 66. C


304
67. Among the following which variable has the greatest significance in
antibiotic management of dental infection?
A. Susceptibility of the organism
B. Route of administration
C. Antigenicity of the antibiotics
D. Specific antibody titer of the host

68. All the spaces mentioned below are involved in classic Ludwig's
angina except:
A. Submandibular
B. Space of the body of the mandible
C. Submental
D. Sub lingual

69. To drain pus from an abscess of the pterygomandibular space from


an intraoral approach, the muscle most likely to be incised is the:
A. Buccinator
B. Temporal
C. Medial pterygoid
D. Lateral pterygoid

67. A 68. B 69. A

305
70. Contamination from a patient with a recent serum hepatitis?
A. Wear gloves
B. Wear a mask
C. Sterilize all instruments and drapes after treatment
D. All of the above

71. pterygomandibular space infection may lead to involvement of the:


A. Submaxillary space
B. Parapharyngeal space
C. Infratemporal space
D. All of the above

72. Which of the following sign/symptom does not suggest


postoperative infection?
A. Firm, tender swelling
B. Localized pitting oedema
C. Temperature elevation
D. Increasing or persisting pain

70. D 71. D 72. B

306
73. After I and D of an abscess, the infectious process has failed to
regress inspite of the patient being on high doses of an antibiotic, it
would be wise to:
A. Repeat culture and sensitivity tests
B. Insert a large drain
C. Augmenting antibiotic action by administration of parenteral
proteolytic enzymes
D. Debride and irrigate the area with a fibrinolylic agent

74. Teeth in line of fire means:


A. Teeth in the area of planned therapeutic radiation
B. Teeth in the line of fracture
C. Teeth within the cancerous lesion
D. None of the above

75. Scrub technique refers to the proper method of scrubbing the:


A. Operator's hands and forearms before donning gown and gloves
B. Patients body surface before surgical incision
C. Operating room after each infected operation
D. All of the above

73. A 74. B 75. A

307
76. Acute pyogenic bacterial infections produce:
A. Leukopenia
B. Lymphopenia
C. Neutropenia
D. Leukocytosis

77. The tendency to infection of bone after radiotherapy is probably


related mainly to:
A. Anaemia
B. Decalcification
C. Invasion by malignant cells
D. Endarteritis of small blood vessels

78. A patient has received a tumoricidal course of cobalt therapy for a


'lesion of the pharynx. His mandibular first molar has developed a
periapical abscess related to caries. This tooth would be treated by:
A. Administering an antibiotic and extracting
B. Performing root canal therapy if at all possible
C. Prescribing an analgesic compound plus an antibiotic
D. Incising the patient to the radiotherapist

76. D 77. D 78. B

308
79. Forty-eight hours following the removal of a left impacted
mandibular third molar, the patient returns to your office complaining of
moderate pain radiating to the left ear. His temperature is 99F and
swelling is minimal. The most probable diagnosis is:
A. Traumatic injury to the inferior alveolar nerve during the injection
procedure
B. Postoperative infection involving the masticator facial space
C. Postoperative infection involving the parotid space
D. Post extraction alveolitis

79. D

309
Chapter 8: TMJ And Maxillary Sinus
1. Paranasal sinus view (or) Water's view is advised for sinusitis
patients. Features of sinusitis include:
A. Fluid levels.
B. Clouding of the antra
C. Clouding and fluid levels
D. Erosion of the bone

2. Of the following which method is commonly used to treat ankylosed


TMJ?
A. Steroid injection into the joint.
B. Muscle relaxants and occlusal equilibration
C. Surgical procedure involving condylectomy
D. Systemic steroids

3. Of the following which is not a structural element of the TMJ?


A. Joint cavities.
B. Condylar process
C. Sigmoid notch
D. Articular disc

1. C 2. C 3. C

310
4. The Al-kayat and Bramley approach to the TM joint is a modification
of the:
A. Hemicronal approach
B. Retroauricular approach
C. Preauricular approach
D. Risdon's approach

5. Mention the blood supply to the flap that has used to close an
oroantral fistula in the area of tooth:
A. Nasopalatine
B. Greater palatine
C. Posterior superior alveolar
D. Facial

6. The treatment of unilateral TMJ ankylosis in an 8-year-old child


would be:
A. Simple gap arthroplasty
B. Condylectomy
C. Gap arthroplasty with costochondral grafting
D. High condylotomy with costochondral grafting

4. C 5. B 6. C

311
7. The ideal surgical approach to TMJ ankylosis is:
A. Endaural
B. Submandibular
C. Postauricular
D. Preauricular

8. Interposition of temporal muscle and fascia in treatment of TMJ


ankylosis is advocated:
A. To prevent reankylosis
B. To prevent erosion of glenoid fossa due to movement of ramal end
C. To provide soft pad for easy movement of ramal end
D. None of the above

9. Which of the following is\are cause/s of TMJ ankylosis?


A. Trauma
B. Middle ear infection
C. Rheumatoid arthritis
D. All of the above

7. D 8. A 9. D

312
10. In surgical management of TMJ ankylosis, one can encounter
excessive bleeding from:
A. Inferior alveolar artery
B. Internal maxillary artery
C. Pterygoid plexus of veins
D. All of the above

11. In unilateral TMJ ankylosis the chin is deviated to:


A. The affected side
B. The contralateral side
C. No deviation seen
D. Side where growth is occurring

12. In a bilateral TMJ ankylosis case the chin would be deviated to:
A. Side of intense ankylosis
B. Side where more movement is present
C. No deviation
D. None of the above

10. D 11. A 12. C

313
13. Dautrey procedure is a treatment modality for:
A. TMJ clicking
B. TMJ dislocation
C. TMJ arthritis
D. TMJ ankylosis

14. A patient complains of pain in TMJ area on mastication, his muscles


of mastication are tender and an audible click is there, these features are
characteristic of:
A. MPDS
B. Traumatic subluxation
C. Rheumatoid arthritis
D. Rheumatic arthritis

15. The hypertonic saline or sclerosing solution is used for conservative


management of TMJ sub-luxation and dislocation. These injections are
given:
A. In superior compartment
B. In inferior compartment
C. Paracapsular
D. In the articular disc

13. B 14. A 15. C

314
16. In TMJ osteoarthritis which medicament is injected in TMJ?
A. Sodium morrhuate
B. Hydrocortisone
C. Sodium salicylate
D. Hypertonic saline

17. Myofacial pain dysfunction syndrome can be precipitated by:


A. High filling or malocclusion
B. Psychogenic factors
C. Bruxism
D. All of the above

18. Arthroscopy is a technique by which:


A. The inside of joint can be seen from outside but for treatment open
surgery is required
B. The inside of joint can be seen and operated from outside, without
any open surgery
C. Dye is injected into the joint and serial radiographs are taken to see
movement of disc in the joint
D. Dye is injected into the joint and outline of joint cavity is delineated
to see any bony erosion spur formation, etc.

16. B 17. D 18. B

315
19. A patient who reports with bilateral dislocation of TMJ should be
managed:
A. Manually without LA
B. Manually with LA
C. Under GA only
D. Surgically under GA

20. The management of recurrent TMJ dislocation is:


A. High condylotomy
B, Eminectomy
C. Dautrey procedure
D. All of the above

21. A patient of MPDS with typical psychosomatic etiology should be


prescribed:
A. Carbamethaxamol
B. Diazepam
C. Fomentation and cold compresses
D. All of the above

19. B 20. D 21. D


316
22. Berger's flap for OAF closure utilizes a:
A. Palatal flap
B. Buccal flap
C. Only a mucosal mobilization
D. None of the above

23. A palatal flap has high success rate in management of OAF because:
A. Abundance of tissue
B. Branch of palatal artery is also mobilized
C. It is resistant to infection
D. Of fatty layer there are less chances of tear

24. If nasal antrostomy is planned after OAF closure Opening should be


made in:
A. Middle meatus
B. Inferior meatus above nasal floor level
C. Inferior meatus at nasal floor level
D. Just above middle concha

22. B 23. B 24. C

317
25. OAF should never be closed if:
A. Palatal mucosa is deficient
B. Signs of infection are present
C. Opening is too large
D. Patient is to have a complete denture

26. If on removing a tooth, one realizes that a large OAF has been
formed:
A. Immediate primary closure should be done
B. Closure should be done after 7 days
C. The sinus should be irrigated, lavage for 2/3 days then closed
D. None of the above

27. Nasal decongestants are prescribed in management of OAF to:


A. Allow drainage
B. Shrink antral lining
C. Prevent infection
D. Make breathing easier

25. B 26. A 27. B

318
28. Sialoangiectasis denotes:
A. Salivary gland and duct system as vastly dilated
B. A sialolith is present
C. A stricture in duct is present
D. Chronic inflammation of salivary gland

29. The treatment of parotid abscess is:


A. Antibiotics only
B. Dilation of duct and (A)
C. Incision parallel to facial nerve branches and drainage with (A)
D. Fomentation with (A)

30. The stone in anterior submandibular salivary gland duct should be


removed by placing the incision:
A. Medial to plica sublingualis
B. Lateral to plica sublingualis
C. Never in anterior region
D. Just superficially in 2nd-3rd molar region

28. A 29. C 30. A

319
31. Once the stone in submandibular salivary gland duct has identified
the incision should be placed:
A. Longitudinally and duct sutured
B. Transversely and duct sutured.
C. Longitudinally and surgical wound closed without suturing the duct
D. Transversely and surgical wound closed without suturing the duct

32. While removing a submandibular gland one encounters:


A. Facial artery, facial vein, cervical branch of facial nerve
B. Facial artery, facial vein, cervical branch of facial nerve
C. Facial artery, facial vein, hypoglossal nerve only
D. Facial artery, facial vein, marginal mandibular branch of facial nerve
only

33. The early manifestation of sialadenitis on a sialogram


A. Terminal acini are dilated
B. The acinar system is dilated
C. The ductal system is dilated
D. Constriction of ductal and acinar system

31. C 32. A 33. A


320
34. Warthin's tumor is:
A. Malignant parotid tumor
B. Benign submandibular tumor
C. Benign parotid tumor
D. Any tumor of salivary glands which can be benign or malignant

35. Mucoepidermoid tumor is:


A. Malignant
B. Benign
C. Squamous cell tumor of salivary gland
D. Same as adenocarcinoma

36. Cylindroma:
A. Is malignant tumor
B. Is slow growing but metastases
C. Shows extensive invasion
D. All of the above

34. C 35. A 36. D

321
37. Ankylosis of the temporomandibular joint is best treated via:
A. X-ray therapy
B. Arthroplasty
C. Cortisone injection
D. Exercises

38. Early movement following surgery for temporomandibular joint


ankylosis is:
A. Harmful
B. Desirable
C. Unimportant
D. Indicated only when ankylosis is limited to one side

39. To reduce a dislocation of the mandible, the movement employed is:


A. Downward and backward
B. Downward and forward
C. Upward and backward
D: Upward and forward

37. B 38. B 39. A

322
40. The most common disorder causing pain about the masticatory
apparatus including the TMJ, is:
A. Myofacial-pain-dysfunction
B. Trigeminal neuralgia
C. Degenerative arthritis
D. Traumatic arthritis

41. Radiographic features of sinusitis include:


A. Fluid levels
B. Erosion of bone
C. Clouding of the antra
D. Clouding and fluid levels

42. Extraction of a maxillary second molar has resulted in a perforation


of the maxillary antrum 0.5 cm in diameter. An acceptable procedure
would be:
A. Caldwell-Luc procedure
B. Creation of a nasal antrostomy window for proper aeration and
drainage
C. Smoothing bone margins of the socket and placement of sutures
across the socket
D. Insertion of iodoform gauze packing into the socket to allow healing
by second intention

40. A 41. D 42. C

323
43. The most common cause of temporomandibular joint ankylosis is:
A. Infection
B. Rheumatoid arthritis
C. Trauma
D. Congenital malformations

44. The "hanging drop" appearance in the maxillary sinus radiograph


indicates:
A. A nasal polyp
B. A blow out # of the orbit
C. A radiographic artifact
D. An antrolith

45. The most critical period for a child after having a condylar fracture
would be the age group between:
A. Five to ten
B. Ten to twelve
C. One to five
D. Twelve to fifteen

43. C 44. B 45. C


324
46. The following movement is used to reduce an anterior dislocation of
the condyles:
A. Downward, forward and upward
B. Downward, backward and upward
C. Upward, forward and medial
D. Upward, backward and lateral

47. The Denker's procedure is an approach to the:


A. Pterygomandibular space
B. Ethmoid sinus
C. Maxillary sinus
D. Medial wall of the orbit

48. Hydrocortisone acetate is injected in a painful arthritic


TM Joint to:
A. Increase the blood supply
B. Lubricate the synovial lining
C. Anaesthetize the nerve supply
D. Decrease the inflammation

46. B 47. C 48. D

325
49. A fractured mandibular condyle is displaced forward and medially
by the action of the following muscle:
A. Temporalis
B. External pterygoid
C. Internal pterygoid
D. Masseter

50. Following an oral surgical procedure the surgical site is covered with
a gauze dampened with saline because the:
A. Saline promotes haemostasis at the site
B. Saline decreases the tendency of the clot to become embedded in the
gauze mesh
C. Saline has localized anti-inflammatory action
D. Saline accelerates the healing of the wound

51. "Drum splints" are used in the management of patients with:


A. Myofacial pain dysfunction syndrome
B. Non displaced mandibular angle fractures
C. Fibrous TMJ ankylosis
D. Atypical facial pain

49. B 50. B 51. A

326
52. The amount of dye that can be injected into ductal system of the
parotid glands during sialography varies between:
A. 0.5 to 0.75m1
B. to 1.5 ml
C. 1.5 to
D. to 2.5 ml

53. The following agent is now commonly used for chemical


capsulorrhaphy in patients with recurrent dislocation of the TMJ:
A. Carnoy's solution
B. Sodium psylliate
C. Sodium tetradecyl sulphate
D. Sodium mourrhate

54. According to Sawhney's classification of TM joint ankylosis, type II


cases present:
A. A bridge of bone between the Remus and the zygomatic arch
B. A joint entirely replaced by a mass of bone
C. Flattening of the condyles with no joint space seen on the radiograph
D. Bony fusion of the outer edge of the articular surface of the joint

52. B 53. C 54. D

327
55. A rib harvested for growth center transplantation following TMJ
arthoplasty, should have at least the following amount of cartilage
attached to it:
A. 5 mm
B. 10 mm
C. 15 mm
D. 25 mm

56. A reciprocal click occurring during movement of the


TM joint is indicative of:
A. Anterior disc displacement with reduction
B. Complete anterior disc dislocation of the disc
C. Disc displacement with perforation
D. Normal joint function

57. The canfiled operation to explore the maxillary sinus utilizes assess
through:
A. Inferior angle of the anterior-inferior angle of the antrum
B. The canine fossa above the premolar teeth
C. An opening created by removal of lower portion of the angle formed
by the junction of antral and nasal walls
D. An opening made below the inferior turbinate

55. B 56. A 57. A

328
58. The clinical sign of acute maxillary sinusitis are all
A. Mucopurulent exudates
B. Tenderness over the anterolateral sinus wall
C. Tenderness to percussion of maxillary molar teeth
D. Loss of vitality of maxillary molar teeth

59. Among the following which is best radiograph to examine maxillary


sinuses is:
A. Orthopantamogram
B. Intraoral periapical films
C. Occlusal view
D. Water's view

60. Commonly advised extra oral radiograph to see the maxillary sinuses
is:
A. AP skull
B. Submentovertex
C. Occipitomental
D. Lateral skull

58. D 59. D 60. C


329
61. Hypertrophy of the mandibular condyle may cause the following:
A. An anterior cross bite
B. Ipsilateral posterior open bite
C. Unilateral class Ill malocclusion
D. All of the above

62. Of the following which is most common disorder causing pain about
the masticatory apparatus including the TMJ?
A. Traumatic arthritis
B. Trigeminal neuralgia
C. Myofacial pain dysfunction syndrome
D. Degenerative arthritis

63. Early movements of TMJ following surgery for TMJ ankylosis is:
A. Desirable
B. Harmful
C. Contraindicated
D. B and C

61. D 62. C 63. A

330
64. Which of the following incisions best exposes TMJ:
A. Submandibular
B. Preauricular
C. Risdon
D. Intraoral

65. The following clinical disease process affect the TMJ directly:
A. Ankylosis
B. Arthritis
C. Dislocation
D. All of the above

66. Best treatment for a small opening of a disease-free maxillary sinus


is:
A. Not treating socket but advising the patient proper home care
B. Packing tile socket with a hemostatic agent to encourage clotting
C. Leaving the socket undisturbed but prescribing nasal vasoconstrictor
and antibiotics.
D. Employ primary closure of the socket using sutures

64. B 65. D 66. A

331
Chapter 9: Maxillofacial Injury
1. Of the following which view is best to visualize zygomatic arches?
A. Submentovertex or jug handle view
B. Occipitomental view
C. Orthopantamogrum
D. Skull PA view

2. Open reduction and internal fixation of fracture fragments in the older


patients is risky because:
A. Of their old age, they less likely tolerate the major procedure under
general anesthesia
B. Fixation is difficult because bone will become denser as age advances
C. Delayed or non-union may occur because of overall decrease in
reparative abilities of the body
D. None of the above are correct

3. Of the following which is the best method of treating a green stick


fracture of the mandible?
A. Allow normal masticatory movements
B. Bringing the teeth into occlusion with interdental wiring
C. Extra skeletal fixation
D. None of the above

1. A 2. D 3. B

332
4. Radiographic examination following chin trauma to a patient
discloses a unilateral fracture of the mandible. Clinical examination
discloses a deviation of the jaw to the right side on opening. One would
suspect fracture of the:
A. Symphysis
B. Left condyle of the mandible
C, Right condyle of the mandible
D. Left body of the mandible

5. The most common site of fracture of the mandible is the:


A. Angle
B. Symphysis
C. Coronoid process
D. Midbody

6. Fat embolism may result following:


A. Nephrosis
B. Burn
C. Crush injury
D. Diabetes

4. C 5. A 6. C

333
7. A LeFort I fracture is a:
A. Transvers fracture of the maxilla
B. Pyramidal fracture of the maxilla
C. Craniofacial dysjunction
D. Fracture of the zygomatic arch

8. A patient with maxillofacial injuries should be carried


A. Supine position
B. Lateral position
C. Prone position
D. Sitting position

9. Immediate management of nasal bleed in facial injuries is:


A. Reduction of nasal bones manually
B. Paraffin gauze packing
C. Positioning the patient in supine position
D. Positioning the patient in prone position

7. A 8. B 9. B

334
10. Placing a nasal pack during nasal bleeding and CSF leak carry the
danger of:
A. Fracture of ethmoidal plates
B. Redirecting the CSF to oropharynx
C. Meningitis
D. Redirecting CSF to orbit

11. Hypovolemic shock develops after loss of:


A. 10% blood
B. 20% blood
C. 30% blood
D. 40% blood

12. Facial wounds can be considered for primary closure when they
report within:
A. 24 hrs
B. 72 hrs
C. 36 hrs
D. 48 hrs

10. C 11. D 12. A


335
13. Failure of primary suturing occurs in facial wounds when:
A. Fine silk had not been used
B. Catgut has been used
C. Dead space develops
D. Continuous suturing is done

14. Diplopia would result if fracture line around zygomaticofrontal


suture passes:
A. Below the Whitnall's tubercle
B. Above the Whitnall's tubercle
C. Through zygomaticofrontal suture
D. Tearing the periosteum of orbital surface of zygomatic bone

15. Battle's sign is associated with:


A. Fracture zygoma
B. Fracture anterior cranial fossa
C. Fracture middle cranial fossa
D. Fracture nasoethmoid

13. C 14. B 15. C

336
16. The differentiating feature of bleeding due to black eye and that due
to fracture of orbit is\are:
A. Circumorbital ecchymosis in black eye develops rapidly
B. Posterior limit of subconjunctival haemorrhage cannot be seen in
black eye
C. Posterior limit of subconjunctival haemorrhage can be seen in black
eye
D. None of the above

17. Guerin type fracture is same as fracture:


A. Le Fort I
B. Le Fort II
C. Suprazygomatic
D. Le Fort Ill

18. 'Moon face' appearance is not present in fracture:


A. LeFort I
B. LeFort II
C. Zygomatic complex
D. Le Fort Ill

16. C 17. A 18. C

337
19. 'Dish face' deformity commonly seen with fractures of middle third
of face is because of:
A. Posterior and downward movement of maxilla
B. Anterior and forward movement of maxilla
C. Anterior and downward movement of maxilla
D. Nasal complex fracture

20. In a crown-root fracture of the tooth, if fracture is not below alveolar


bone and pulp is not exposed the tooth should be:
A. Endodontically restored
B. Extracted
C. Only jacket crown given
D. Observed for 3-6 weeks

21. In replanting an avulsed tooth:


A. It should be thoroughly made sterile
B. Root filling with apicoectomy should be done
C. There is failure due to external resorption
D. All of the above

19. A 20. C 21. C


338
22. Pathognomonic sign of fracture mandible is:
A. Deranged occlusion
B. Tenderness and swelling at site
C. Sublingual haematoma
D. Inability to open mouth

23. Fracture of coronoid process can occur due to:


A. Trauma at chin region
B. Trauma from posterior region
C. Reflex muscular contraction
D. Lateral trauma

24. The term vertical in 'vertical favorable' fractures connotes:


A. The fracture line running in vertical direction
B. The displacement of fracture is in vertical plane
C. The direction of view of the observer is in vertical direction
D. Fracture can be reduced vertically

22. C 23. C 24. C

339
25. Submentovertex view is an ideal view for diagnosing fracture of:
A. Zygoma
B. Zygomatic arch
C. Horizontal fracture of mandible
D. Nasoethmoid region

26. Gunning type splints are used when patient is:


A. Edentulous in one jaw
B. Edentulous in both jaws
C. When vertical relation is not known
D. All of the above

27. A 32-year-old female patient reported with bilateral subcondylar


fracture with anterior open bite, the treatment would constitute:
A. Intermaxillary Fixation for 6 weeks
B. Distraction with rubber stoppers and anterior traction followed by
Intermaxillary fixation for 4-6 weeks
C. Intermaxillary fixation for 4 weeks
D. Distraction with rubber stoppers and posterior traction followed by
intermaxillary fixation for 4-6 weeks

25. B 26. D 27. B

340
28. If fracture angle results following extraction of mandibular impacted
3rd molar the immediate treatment should be:
A. IMF only
B. Bone plating (under GA)
C. Superior border transosseous wiring and IMF
D. Transosseous wiring at the lower border and IMF

29. Walsham's forceps are used for:


A. Disimpaction of maxilla
B. Reduction of maxilla fractures
C. Reduction of fracture nasal bones
D. Ash septal force

30. Indirect reduction of fracture zygoma can be done by:


A. Gillies approach
B. Intraoral approach
C. Percutaneous approach
D. All of the above

28. C 29. C 30. D

341
31. The optimum length of screw, for fixation of plate in mandible is:
A. 2 mm
B. 3 mm
C. 4 mm
D. 6 mm

32. The most commonly injured tooth during the placement of miniplate
for the fracture of mandible in anterior region may be:
A. Central incisor
B. Lateral incisor
C. Canine
D. 1st premolar

33. In fractures of mandible in elderly patients, fixation of plate is:


A. Submucosal
B. Supraperiosteal
C. Subperiostal
D. None of the above

31. C 32. C 33. B

342
34. The contraindication to miniplate along the line of osteosynthesis
would be:
A. A comminuted fracture
B. An infected fracture site
C. A fracture in a 10-year-old
D. When more than one fracture site exists in mandible

35. Stress shielding effect is seen in:


A. Miniplating
B. Compression bone plating
C. Lag screw
D. Transosseous wiring

36. During compression bone plating which type of healing would be


observed:
A. Contact healing
B. Gap healing
C. Primary healing
D. Secondary healing

34. C 35. B 36. C


343
37. In Luhr system of plating, the two individual compression screws
move through:
A. 1 mm
B. 1.6 mm
C. 3.2 mm
D. 4 mm

38. The whole of middle third of face can be approached by:


A. Infraorbital incisions
B. Bicoronal flap
C. Alkayat and bramley approach
D. Transconjunctival approach

39. Acceptable treatment modality for fracture mandible in an 8-year-old


is:
A. Transosseous wiring since IMF cannot be done
B. IMF for 3 weeks
C. Circum-mandibular splinting
D. AO bone plating

37. B 38. B 39. C


344
40. A patient with maxillofacial injuries should be carried in a supine
position only when there is:
A. Spinal, cervical injury
B. Bilateral parasymphysis fracture
C. Unconsciousness
D. Excessive mobility of fractured maxilla

41. Tongue-tie is indicated in:


A. Bilateral parasymphysis fracture
B. Unconscious patient
C. Chin has been destroyed in gunshot
D. All of the above

42. Glasgow coma scale is used:


A. To ascertain motor responsiveness
B. Verbal responsiveness
C. Eye response
D. To ascertain level of consciousness

40. A 41. D 42. D

345
43. Examination of pupils is of paramount importance in maxillofacial
injuries because it indicates:
A. Trauma to brain
B. Trauma to optic tract
C. Progress of patient after trauma
D. All of the above

44. The method commonly used to differentiate nasal discharge from


CSF in fracture of middle third of face:
A. Examining level of glucose
B. Examining level of chlorides
C. Drying the discharge on a piece of cloth
D. Examining the level of proteins

45. Patient with maxillofacial injury complains of regurgitation, absence


of gag reflex and weakening of voice, he may have:
A. Laryngeal trauma
B. Injury to middle cranial fossa
C. These symptoms are due to acute pain
D. Paralysis of IX N

43. D 44. C 45. B

346
46. Treatment of choice of a linear non-displaced fracture of the body of
the mandible with a full complement of teeth is:
A. Kirchner's wire
B. Circumferential wiring
C. External pin fixation
D. Closed reduction with intermaxillary fixation

47. Following a bilateral fracture of the mandible in the canine region,


the anterior fragment of the mandible is displaced posteriorly by the
action of the:
A. Anterior belly of the digastric muscles, geniohyoid and genioglossus
muscles
B. Thyrohyoid, genioglossus and geniohyoid muscles
C. Mylohyoid, genioglossus and geniohyoid muscles
D. Mylohyoid, geniohyoid and thyrohyoid muscles

48. The best radiographic view for evaluation of fracture of the middle
face is:
A. PA skull
B. Lateral skull
C. Towne's view
D. Water's view

46. D 47. A 48. D


347
49. The most important step in suturing lacerated lip:
A. Apposition of muscular layer
B. Apposition of vermilion border
C. Apposition of mucosal layer
D. All of the above

50. A patient presents with lateral subconjunctival haemorrhage.


Infraorbital step and diplopia on right side with inability to open mouth,
he can be having:
A. Fracture subcondylar right side
B. Fracture zygoma right side
C. Fracture Le Fort Il right side
D. Fracture of floor of the orbit

51. Apatient presents with bilateral infraorbital step, paraesthesia on left


cheek region, with posterior gagging, and mobility of maxillary complex
at nasal bones, it indicates:
A. Bilateral fracture zygoma
B. Bilateral fracture Le Fort Il
C. Fracture zygoma left side with bilateral Le Fort Il
D. Fracture bilateral subcondylar and fracture zygoma left side

49. D 50. B 51. B

348
52. A patient complains of diplopia following fracture zygoma, this is
because of:
A. Fracture of orbital floor
B. Entrapment of medial rectus
C. Entrapment of superior oblique
D. All of the above

53. Traumatic telecanthus is associated with:


A. Bilateral Le Fort Il fracture
B. Nasoethmoidal injury
C. Fracture nasal bones
D. Bilateral fracture zygoma with enophthalmos

54. Guerin sign is presence of:


A. Ecchymosis at mastoid area
B. Ecchymosis at greater palatine foramen area
C. Ecchymosis in zygomatic butress area
D. Ecchymosis in sublingual area

52. A 53. B 54. B

349
55. The typical 'cracked pot' sound on percussion of upper teeth is
indicative of fracture:
A. Le Fort I
B. Le Fort II
C. Le Fort III
D. A and B

56. In Le Fort I fracture infraorbital rim is:


A. Bilaterally involved
B. Not involved
C. Involved medially
D. May or may not be involved

57. Ecchymosis at zygomatic buttress would indicate fracture:


A. LeFort I
B. Le Fort II
C. Zygoma
D. All of the above

55. D 56. B 57. D

350
58. On palpation there is a step at bilateral infraorbital margins and
mobility of midface is detectable at nasal bridge a possible diagnosis
would be fracture:
A. Le Fort I
B. Le Fort II
C. Le Fort III
D. Le Fort III and II

59. There is tenderness at zygomatico-frontal suture, with hooding of


eyes and step at zygomatic arches with disturbed occlusion, a possible
diagnosis would be:
A. Fracture zygoma and zygomatic arch
B. Fracture zygoma with paralysis of III nerve causing hooding
C. Fracture Le Fort II and fracture zygoma
D. Fracture Le Fort III

60. On moving the maxilla bimanually, movement is felt at zygomatico-


frontal suture area in a case of middle third fracture of face, it is
indicative of fracture:
A. Le Fort I
B. Le Fort II
C. Le Fort III
D. Zygoma

58. B 59. D 60. C

351
61, Step and mobility at infraorbital margin and step at
Zygomatico-frontal region would indicate fracture:
A. Le Fort 111 and Le Fort 11
B. Le Fort Il and zygoma
C. Le Fort Ill and zygoma
D. None of the above

62. Fracture Le Fort Il involves the following bones:


A. Frontal process of maxilla, nasal, lacrimal
B. Frontal process of maxilla, lacrimal, ethmoidal
C. Frontal, maxilla and nasal
D. Maxilla, frontal process of zygoma, nasal and lacrimal

63. A patient presents with open bite on left side and with tenderness at
nasal bones, it could be fracture:
A. Unilateral Le Fort I on right side
B. Subcondylar on left side and zygoma on right side
C. Le Fort Il on right side
D. Zygoma on right side and subcondylar on right side

61. B 62. A 63. C

352
64. If there is root fracture in apical third of tooth without mobility:
A. Tooth should be extracted
B. Treated endodontically
C. No treatment and periodic review
D. None of the above

65. High rate of fractures at canine region of mandible is


A. Change of direction of forces occurring here
B. Long canine root
C. Lower border is thin in this area
D. Alveolus is thin in this area

66. Respiratory embarrassment can occur in fracture:


A. Angle
B. Parasymphysis
C. Bilateral parasymphysis
D. Bilateral subcondylar

64. C 65. B 66. C

353
67. A patient reported with deviation of jaw to the right side on opening
and bleeding from the right ear, is a typical picture of:
A. Left-subcondylar fracture
B. Right-subcondylar fracture
C. Right-subcondylar with fracture of anterior cranial fossa
D. None of the above

68. A patient with bilateral subcondylar fracture presents with:


A. Inability to open mouth
B. On opening mandible moves forward
C. Anterior open bite
D. Closed bite

69. A horizontalIy unfavorable fracture of angle of mandible runs from:


A. Lingual plate anteriorly backward through buccal plate posteriorly
B. Upper border downward and forward
C. Upper border downward and backward
D. None of the above

67. B 68. C 69. C

354
70. Best radiograph for fractures of middle third of face:
A. Submentovertex
B. Reverse Towne's view
C. OPG
D. Occipitomental view

71. To find if fracture of angle mandible is vertically favourable or


unfavourable the radiograph advised:
A. PA view mandible
B. Lateral oblique 300 mandible
C. Occipitomental view
D. Lateral oblique 150 mandible

72. There is absolute indication for extraction of a tooth which is present


in the fracture line when there is:
A. Longitudinal fracture of tooth involving the root
B. Infected fracture line
C. Dislocation of tooth from its socket
D. All of the above

70. D 71. A 72. D


355
73. A 7 -year-old boy presented with fracture of left subcondylar region
with occlusion undisturbed, the treatment would be:
A. Immobilisation for 7 days
B. Immobilisation for 14 days with intermittent active opening
C. No immobilisation with restricted mouth opening for 10 days
D. No immobilisation and active movement

74. While doing circum-mandibular wiring there are chances of injuring:


A. Facial nerve
B. Facial artery, vein
C. Epiglottis
D. Lingual nerve

75. The submandibular incision for approaching angle fracture is placed


one finger breadth below the lower border of mandible:
A. To keep the incision line masked
B. To prevent injury to facial vessels
C. To prevent injury to marginal mandibular nerve
D. Access becomes easy

73. D 74. B 75. C


356
76. In old patients, open reduction and fixation should be done with
great care to:
A. Prevent iatrogenic fracture of atrophic mandible
B. Detach minimum of periosteum
C. Prevent dislocation of condyle
D. None of the above

77. Following are the examples of rigid fixation:


A. Lateral Fontal suspension
B. Extra skeletal pin fixation
C. Bone plating
D. B and C

78. To fix a zygomatic fracture by open reduction following sites have to


be approached:
A. Zygomatic, frontal and infraorbital
B. Infraorbital and zygomaticotemporal
C. A and B
D. Zygomaticofrontal, zygomatic prominence and, floor of orbit

76. B 77. C 78. C


357
79. Tetanus immunization of patients with facial injuries includes:
A. Administration of antibiotics
B. Administration of 0.5 ml of absorbed toxoid
C. Administration of 250 units of TIG
D. (B) and (C) above

80. The 'Golden Hour of Trauma' refers to:


A. The period of time between minutes and hours after the trauma
B. The period of time in seconds or minutes after the traumatic incident
C. The period of time exactly one hour after the trauma is sustained
D. The period of time during which the patient regains consciousness
after trauma

81. A true open bite is seen in which of the following fractures:


A. Horizontal fracture of the maxilla
B. Fracture of the angle of the mandible
C. Unilateral condylar fracture
D. Fracture of the zygomatic bone

79. D 80. C 81. A

358
82. Which of the following mouth guards are most recommended for
high impact sports such as rugby?
A, Type I (stock) mouth guards
B. Type II (boil and bite) mouth guards
C. Type Ill (custom) mouth guards
D. All are equally effective

83. The "Epi-Tek" catheter is used for:


A. Drainage of pus from a fascial space
B. Administration of fluids
C. Sialography
D. Control of nasal haemorrhage

84. Endotoxic shock is seen in patients of maxillofacial trauma with


related:
A. Abdominal injuries
B. Cardio-thoracic injuries
C. Crush injuries
D. Spinal cord injuries

82. C 83. D 84. A


359
85. Indications for neurosurgical referral in a patient with maxillofacial
trauma include all, except:
A. Meningitis
B. Large aerocoele
C. Excessive oral/ nasal bleeding
D. Young patient with CSF leak

86. As per the Glasgow Coma Scale, motor responsiveness scores 3


denotes:
A. Obeying
B. Localizing
C. Extending
D. Flexing

87. Acrylated arch bars are also known as:


A. Krupps arch bars
B. Winter's arch bars
C. Wipla arch bars
D. Schuchardt arch bars

85. C 86. D 87. D


360
88. Contraindications for use of gunning splints in patients with fractures
of the mandible include all, except:
A. Grossly comminuted fractures
B. Fractures of the atrophic mandible
C. Bilateral fractures of the edentulous mandible where proximal
fragments can be controlled by IMF
D. Posterior displacement of the anterior part of the mandible

89. Following intermaxillary fixation, a patient can lose weight on an


average up to:
A. 5kg
B. 5.5 kg
C. 6.5 kg
D. 7 kg

90. Log rolling a trauma patient into the recovery position minimizes the
risk of:
A. Damage to the spinal cord
B. Aspiration of blood or foreign bodies
C. Obstruction of the airway from the displaced tongue
D. None of the above

88. C 89. C 90. A

361
91. The "parade-ground" fracture refers to:
A. Bilateral fractures of the mandibular angles
B. Midline symphyseal and undisplaced bilateral condylar fracture
C. Bilateral fracture dislocation of the condyles
D. Midline symphyseal and bilateral fracture dislocation of the condyles

92. A patient with severe maxillofacial trauma has a systolic pressure


less than 100 mm Hg. The expected amount of blood loss should be in
the range of:
A. 500 ml
B. 750 ml
C. 1000 ml
D. 1250 ml

93. Replantation technique of avulse mature teeth requires:


A. Removal of any soil contamination by rinsing with water followed by
sterilization
B. Leaving the pulp tissue intact but sealing the apex with an alloy prior
to replantation
C. Removal of any blood clot in the socket by curettage
D. None of the above

91. C 92. C 93. D

362
94. "Matchbox" injuries of the maxilla-mandibular region refer to:
A. Burns of the soft tissues overlying the facial skeleton
B. Fracture of the nasal bones
C. Fractures of the middle third of the facial skeleton
D. Crush injuries of the maxillary sinus

95. Paresthesia is one of the commonest findings in which of the


following fractures?
A. Subcondylar fracture of the mandible
B. Zygomatico maxillary complex fracture
C. Fractured coronoid and displacement of the fracture
D. Symphysis fracture associated with bilateral subcondylar fracture
(parade ground fracture)

96. Clinical findings of a subcondylar fracture on the right side include:


A. Trismus and crepitus bilaterally
B. Inability to deviate the mandible to the left
C. Deviation of the mandible on protrusion towards the left side
D. Bleeding intraorally most of the times moderately sometimes
severely

94. C 95. B 96. B

363
97. Clinical findings of a subcondylar fracture on the left side include:
A. Trismus and crepitus bilaterally
B. Inability to deviate the mandible to the left
C. Deviation of the mandible on protrusion towards the left side
D. Bleeding intraorally, most of the times moderately sometimes
severely

98. After reduction of a mandibular dislocation that occurred for the first
time, treatment should be to:
A. Inject sclerosing solution into the joint, so further dislocation is less
likely
B. Inject corticosteroids into the joint which reduces the inflammation
C. Immobilize with IMF for 5-6 weeks
D. Advise the patient to limit opening of the mouth for
2-3 weeks

99. Among the following which is the least common site of mandibular
fracture?
A. Body
B. Coronoid
C. Condyle
D. Angle

97. C 98. D 99. B

364
100. CSF rhinorrhoea commonly seen in patient with:
A. Le Fort I fracture
B. Zygomatic complex fracture
C. Le Fort Ill fracture
D. Bilateral condylar fracture with symphysis fracture of mandible

101. Forward displacement of the condyle in condylar fractures is due


to:
A. Medial pterygoid
B. Lateral pterygoid
C. Masseter
D. Temporalis

102. The movement employed in the reduction of displaced mandible is:


A. Downward and backward
B. Upward and forward
C. Upward and backward
D. Downward and forward

100. C 101. B 102. A

365
103. After a blow to the chin, patient has pain and tenderness over the
right TMJ. Open bite, and deviation of the chin to the right side. This
suggests:
A. Fracture mandible right angle region
B. Fracture mandible left angle region
C. Fracture mandible right condyle
D. Fracture mandible left condyle

104. Fractures of mandible where full complement of teeth are present is


best treated by:
A. Intraoral open reduction
B. Circumferential wiring
C. Full cast splints
D. Closed reduction and intermaxillary fixation

105. In which of the following anterior open bite Occur?


A. Bilateral condylar fractures
B. Symphysis fracture on one side and angle fracture on the other side of
the mandible
C. Horizontal fracture of the maxilla
D. A and C

103. C 104. D 105. D

366
106. For an oral surgery patient undergoing closed reduction of a
fractured mandible which of the following procedures should be
performed?
A. Medical history and physical examination
B. CBC
C. Urine analysis
D. All of the above

107. Bones usually fracture at the sites of:


A. Compressive strain
B. Tensile strain
C. Rich blood supply
D. Thin periosteal covering

108. Energy range required to fracture the mandible is in the range of:
A. 44.6 - 74.4 kg/m
B. 79.2 - 98.1 kg/m
C. 10.1 - 28.5 kg/m
D. 100.2 - 150.9 kg/m

106. D 107. B 108. A


367
109. In case of subcondylar fracture, the condyle moves
A. Anterior lateral direction
B. Posterior medial direction
C. Posterior lateral direction
D. Anterior medial direction

110. Geurin's fracture is synonymus to:


A. Le Fort I level fracture
B. Le Fort Il level fracture
C. Le Fort Ill level fracture
D. Le Fort IV level fracture

111. The following fracture is usually pyramidal in shape:


A. Le Fort I fracture
B. Le Fort Il fracture
C. Le Fort III fracture
D. Mandibular symphysis fracture

109. D 110. A 111. B

368
112. Cranio facial dysjunction commonly occurs in:
A. Le Fort I fracture
B. Le Fort 111 fracture
C. Mandibular symphysis fracture
D. Mandibular condyle

113. Gilli's approach is:


A. Used to block inferior alveolar nerve
B. Used to reduce the fractured zygoma
C. Placed just anterior to the ear
D. One of the frequent approaches for condylar surgery

114. Lowering the pupillary level of eyeball occur if:


A. The orbital volume increases
B. Detachment of suspensory ligament of lockwood occurs
C. In case of blow out fracture
D. None of the above

112. B 113. B 114. B

369
115. How many weeks of fixation are required for fracture mandible?
A. 8 - 10 weeks
B. 6 - 8 weeks
C. 4 - 6 weeks
D. 2 - 4 weeks

116. During the Gillies approach, the structure of anatomic significance


is:
A. Superficial temporal artery
B. Marginal mandibular nerve
C. Internal jugular vein
D. Inferior alveolar nerve

117. An unfavorable displaced fracture of the mandibular angle is


difficult to treat because of:
A. Muscle pull causes distraction
B. Malocclusion secondary to the injury
C. Injury to nerves and vessels
D. Bone in that region is very thick

115. C 116. A 117. A


370
118. Among the following which may produce respiratory obstruction?
A. Bilateral condylar fracture
B. Symphysis fracture of mandible
C. Bilateral fracture of mandible in the second premolar area
D. Fracture of the angle of the mandible

119. In a patient with bilateral dislocated fractures of the necks of the


mandibular condyles one can expect the following clinical signs:
A. Anterior open bite
B. Inability to protrude the mandible
C. Inability to bring posterior molars into contact
D. A and B

120. Of the following which facial bone is most frequently fractured?


A. Mandible
C. Nasal
B. Maxilla
D. Zygomatic

118. C 119. D 120. D

371
121. Ecchymosis in the post auricular region over the mas-
A. Battle's sign
B. Murphy's sign
C. Guiren's sign
D. None of the above

122. "Hanging drop" sign in radiograph usually indicate:


A. Nasal bone fracture
B. Orbital floor fracture
C. Isolated coronoid fracture fragment hanging by temporalis muscle
D. Condylar fracture

123. Of the following which is almost pathognomonic of a mandibular


fracture?
A. Deep laceration near the area of trauma
B. Ecchymosis in the lingual sulcus
C. Anterior open bite
D. None of the above are correct. There is no pathogenic feature of
mandibular fracture

121. A 122. B 123. B

372
124. Among the following where one might see "Bucket handle"
displacement of fracture segments:
A. Bilateral mandibular body in a 60-year-old
B. Isolated nasal bone fracture in a 35-year-old female who is pregnant
C. Unilateral zygomatic maxillary complex in a 40-year- old patient who
is wrestler by occupation
D. Pure blow out fracture of floor of the orbit in a 20-year- old tennis
player

125. A patient came to the trauma center who had a blow over his lower
jaw. Intraoral examination reveals hematoma near lingual side of lower
second molar. The proximal fragment is medially displaced. Which of
the following might explain the above findings?
A. Vertically favorable fracture of angle of the mandible and
displacement is due to masseter action
B. Vertically unfavorable fracture of angle of the mandible and
displacement is due to internal pterygoid action
C. Horizontally favourable fracture at the angle and displacement is due
to medial pterygoid action
D. Horizontally unfavourable fracture at the angle and displacement is
due to masseter action

124. A 125. B

373
126. In case of pure symphysis fracture of mandible, fracture segments
are usually displaced:
A. Lingually and downward by the pull of genioglossus mylohyoid
muscles
B. Mainly lingually and lingual and downward movement due to the pull
of geniohyoid and mylohyoid muscle
C. Mainly upwards movement. Lingual and upward movement due to
the pull of geniohyoid and mylohyoid muscle
D. None of the above are correct. A little or no displacement occur

127. Excessive muscular contraction is one of the frequent causes of:


A. Unilateral condylar fracture
B. Cornoid fracture
C. Fracture of angle of the mandible
D. Bilateral condylar fracture

128. Which of the following is right regarding Guardsman


A. It is nasal fracture associated with unilateral zygomatic maxillary
complex fracture
B. CSF rhinorrhoea is severe in these cases
C. Commonly seen in epileptics
D. It is a type of comminuted fracture due to bullet or missile injures in
the war field

126. D 127. B 128. C

374
129. "Panda Facies" is one of the terms to describe the patient face after
mid face trauma. The appearance is due to:
A. A-gross swelling of the face
B. CSF rhinorrhoea and bleeding from the nose and laceration results in
red and white streaks on the face
C. Edema and ecchymosis around the eyes
D. Sub conjunctival hemorrhage (Bilateral)

130. Tram line pattern on the face is due to:


A. Sutures placed with tension
B. CSF rhinorrhea
C. Circumorbital ecchymosis
D. Sub conjunctival hemorrhage

131. Of the following which is weakest part of orbit:


A. A Medial wall
B. Lateral wall
C. Floor of the orbit
D. Roof of the orbit

129. C 130. B 131. C


375
132. Posterior displacement of the fractured anterior segment in the
bilateral fracture of the mandible in the canine region is due to the action
of the:
A. Thyrohyoid, genioglossus and geniohyoid
B. Mylohyoid, genioglossus and geniohyoid
C. Geniohyoid and genioglossus and anterior belly of digastric muscles
D. Mylohyoid, geniohyoid and thyrohyoid muscle

133. Which of the following is complication often open fracture?


A. Malunion
B. Nonunion
C. Infection
D. Crepitation

134. A patient with suspected cervical fracture should, be kept in:


A. Body and neck extended
B. Prone position
C. Body extended and neck flexed
D. Both body and neck flexed

132. C 133. C 134. A


376
135. Principles in treating fractures include:
A. Reduction of fracture
B. Fixation of fracture and restoration of occlusion
C. Immobilisation
D. All of the above

136. Depressed fracture of the zygomatic area may be clinically


recognized by:
A. Concavity of the overlying tissue in the zygomatic arch area
B. Interference with movements of the mandible
C. Subluxation of condyles
D. A and B

137. Which of the following is characteristic of Le fort fracture


A. CSF rhinorrhoea
B. Bleeding from the ear
C. Bleeding into the antrum
D. A and B

135. D 136. D 137. C

377
138. Most common anatomic site of fracture mandible is:
A. Angle
B. Condyle
C. Coronoid
D. Body

139. Of the following, which is the immediate treatment for a patient


with comminuted fracture and in the state of shock?
A. Ringer's lactate solution by IV route
B. Normal saline by IV route
C. Blood transfusion
D. Plasma expanders

140. A Gunning's splint is used in the treatment of:


A. Fracture of the condylar neck of a child
B. Fracture of the edentulous mandible
C. Anterior dentoalveolar fracture
D. None of the above

138. A 139. A 140. B

378
141. Presence of ecchymosis in the sulci, the floor of the mouth and hard
palate usually suggest there is a:
A. Laceration
B. Fracture
C. Abrasion
D. None of the above

142. Among the following which is not used in the fixation of bone
grafts?
A. A Bone plates
B. Titanium mesh
C. Gut
D. Tranosseous wires

143. Which is most frequently performed to correct a skeletal


mandibular retrognathia?
A. 'C' type osteotomy
B. Horizontal osteotomy or the ramus
C. Oblique sub condylar
D. Sagittal split osteotomy of the ramus

141. B 142. C 143. D

379
144. Which of the following procedure best suited to correct bi-
maxillary protrusion?
A. Extraction of four premolars and anterior alveolar segment
repositioning
B. Mandibular body osteotomy and posterior maxillary osteotomy
C. Subcondylar osteotomy
D. None of the above

145. What are the principles in the treatment of mandibular fractures to


ensure rehabilitation of jaw function?
A. Reduction of the fracture
B. Fixation of the fracture
C. Restoration of occlusion
D. All of the above

146. Among the following which muscle plays least role to displace
fractured mandibular angle?
A. Temporalis
B. Triangularis
C. Medial pterygoid
D. Lateral pterygoid

144. A 145. D 146. B

380
147. The proximal segment of mandibular angle fracture usually
displaced in which direction?
A. Anterior and superior
B. Posterior and inferior
C. Inferior only
D. Posterior and superior

148. After a depressed fracture of zygomatic arch mandibular movement


is restricted. The most probable rea-
A. Disruption of TMJ
B. Spasm of the lateral pterygoid muscle
C. Mechanical impingement of the fracture fragment on the coronoid
process
D. Splinting action of masseter and medial pterygoid muscle

149. Among the following which is compound fracture?


A. Fracture with many small fragments
B. Fracture in a "star" shaped appearance
C. Fracture with commination with the oral cavity
D. Fracture with bleeding into the masticator space

147. A 148. C 149. C

381
150. Most common disorder causing pain about the masticatory
apparatus including the TMJ is:
A. Trigeminal neuralgia
B. MPDS
C. Degenerative arthritis
D. Traumatic arthritis

151. Among the following which extra oral radiograph best


demonstrates the subcondylar fracture?
A. Towne's projection
B. AP Mandible
C. Submento vertex
D. Occipitomental

152. The mini-bone plate system is a:


A. Gompressive bone plating system
B. Monocortical system
C. Bicortical system
D. None of the above

150. B 151. A 152. B


382
153. The best and most effective position (in mandible) of miniplate as
proved by various experimental studies is:
A. Lower border of mandible
B. Buccoalveolar region
C. Linguoalveolar region
D. Ata height midway between superior alveolar region and lower
border of mandible

154. The minimum number of miniplates required in fractures anterior to


canine in mandible is:
A. No plate is required since anterior region develops less amount of
tension forces than in molar region
B. Only one plate as in molar region
C. Two plates
D. Three plates

155. Minimum number of screws required for fixation of miniplate are:


A. One screw on each side of fracture site
B. Two screws on each side of fracture site
C. Three screws on each side of fracture site
D. Two screws in smaller fragment and three screws in larger fragment

153. C 154. C 155. B

383
156. In a fracture of mandible at the angle-region the placement of
screws in proximal segment is in:
A. Sagittal plane
B. Horizontal plane
C. Such a close relation to teeth that injury to molar invariably occurs
D. No relation to teeth

157. To prevent injury to the apices of the teeth in mandible, the


placement of miniplate is:
A. At the lower border of mandible
B. At a distance; twice the height of the clinical crown below the
alveolar crest
C. Below the inferior alveolar canal
D. Not possible since alveolar bone bears the apices of the teeth

158. The spherical gliding principle is a feature of:


A. Miniplates
B. Luhr plating
C. ASIF plating
D. Lag screws

156. A 157. B 158. C

384
159. Epiphora results due to:
A. Blockage of lacrimal gland canaliculi
B. Blockage of nasolacrimal duct
C. Over activity of lacrimal glands
D. Evulsion of palpebral conjunctiva

160. The most common site of mandible, which shows non-union or


delayed union after IMF is:
A. Angle
B. Body
C. Symphysis
D. Ramus

161. Risdon wiring is indicated for:


A. Body fracture
B. Angle fracture
C. Symphysis fracture
D. Subcondylar fracture

159. B 160. C 161. C

385
162. Facial paresis following maxillofacial fractures is most common in:
A. # of the condylar neck
B. # of the nasal bones
C. # of the zygomatico-maxillary complex
D. # of the mandibular symphysis

163. The most common complication of maxillofacial injuries requiring


immediate attention is:
A. Haemorrhage
B. Airway obstruction
C. Infection
D. Shock

164. The safest initial approach to opening the airway of a patient with
maxillofacial trauma and suspected neck injury is:
A. Head tilt-chin lift
B. Jaw thrust technique
C. Head lift-neck lift
D. Heimlich procedure

162. C 163. B 164. B


386
165. The Gillies approach is used to gain access to the following bone:
A. Nasal bone
B. Zygomatic bone
C. Maxilla
D. Temporal bone

166. Ecchymosis in the mastoid region seen after a fracture of the


petrous bone is known as:
A. Chovstek's sign
B. Battle's sign
C. Guerin's sign
D. Tinel's sign

167. Direct interdental wiring is also known as:


A. Risdon's wiring
B. Gilmer's wiring
C. Eyelet wiring
D. Col. Stout's wiring

165. B 166. B 167. B

387
168. The "2.7" in the 2.7 mm plating system denotes:
A. The bone plate thickness
B. The bone screw diameter
C. The diameter of the plate hole
D. The distance between the plates

169. The proximal segment of a horizontally unfavourable mandibular


angle fracture is displaced:
A. Inferiorly
B. Posteriorly and medially
C. Anteriorly and superiorly
D. Anteriorly and laterally

170. The outermost holes of an "eccentric dynamic compression plate"


are angled at:
A. 75 Degree
B. 85 Degree
C. 60 Degree
D. 55 Degree

168. B 169. C 170. A


388
171. CSF rhinorrhea following a Le Fort Ill frontal bone fracture is
because of:
A. # of the cribriform plate of the ethmoid
B. # of the posterior wall of the frontal sinus
C. # of the roof of sphenoid air sinus
D. All of the above

172. 'Panda facies' is commonly seen after:


A. Le Fort I fractures
B. Le Fort Il fractures
C. Zygomatic arch fractures
D. Orbital blow-out fractures

173. A subconjunctival haemorrhage remains bright red in colour for a


long time because of the:
A. Permeability of the conjunctiva to oxygen
B. Natural colour of blood
C. Lack of drainage of the pooled blood
D. None of the above

171. D 172. B 173. A


389
174. The following structures are divided when the angle of the
mandible is exposed through a submandibular incision:
A. Skin and superficial fascia only
B. Skin, superficial fascia, platysma, deep cervical fascia and medial
pterygoid muscle
C. Skin, superficial fascia, deep cervical fascia and masseter muscle
D. Skin, superficial fascia, platysma, deep cervical fascia and masseter
muscle

175. The most common pathognomonic sign of a mandibular fracture is:


A. Malocclusion
B. Trismus
C. Deviation of the jaw on opening
D. Paranesthesia of the mental nerve

176. A protein supplement is provided for a 70 kg adult patient who has


been treated with intermaxillary fixation following a non-displaced
mandibular angle fracture. The amount of protein that he requires per
day is:
A. 100 mg/kg/ day
B. 100 gm/kg/day
C. 1000 mg/kg/ day
D. 500 mg/kg/ day

174. D 175. A 176. C

390
177. Clinical union of fractures occurring in the region of the middle
third of the facial skeleton takes place, on an average within:
A. 3 to 4 weeks
B. 4 to 6 weeks
C. 6 to 8 weeks
D. 2 to 4 weeks

178. Reduction of a fractured malar bone is best carried out after:


A. Teri-orbital oedema has subsided
B. Three to five days after injury
C. Chemosis has subsided
D. All of the above

179. A Class Ill fracture of the tooth is:


A. A fracture of only the enamel portion of the crown of the tooth
B. An injury extending into the dentin but with no pulpal exposure
C. An extensive injury to the coronal portion of the tooth with pulp
exposure
D. A fracture occurring at or below the cement-enamel junction of the
tooth

177. A 178. D 179. C

391
180. Elastic traction used commonly to reduce facial fractures, does so
by overcoming:
A. The active muscular pull that distracts the figments
B. The organized connective tissue at the fracture site
C. The malposition caused by the direction and force of trauma
D. All of the above

181. The gap created between the base of the skull and the ramus of the
mandible during a TMJ arthroplasty to prevent reankylosis should be at
least:
A. 05. To 1.0 cm
B. 1.0 to 1.5 cm
C. 1.5 to 2.5 cm
D. 2.5 to 3.5 cm

182. Gunshot fractures of the facial bones should not be treated via open
reduction because:
A. Infection will definitely occur
B. Closure of the wound due to soft tissue loss is difficult
C. The numerous small fragments will lose their vitality when the
periosteum is reflected
D. All of the above

180. D 181. B 182. C


392
183. While doing circumferential wiring around a mandibular gunning
splint, care must be taken not to damage the:
A. Mandibular branch of the facial nerve
B. Facial artery as it crosses the antero-inferior of the masseter
C. The lingual nerve
D. The submandibular gland and its duct

183. B

393
Chapter 10: Reconstructive and Orthognathic
Surgery
1. A transplant of bone from one human to another's is termed:
A. Autogenous
B. Homologous
C. Heterogenous
D. Alloplastic

2. Which of the following surgical procedures are commonly used for


treatment of maxillary retrognathia?
A. Le Fort I osteotomy
B. C-osteotomy
C. Inverted L osteotomy
D. Anterior maxillary osteotomy

3. Which of the following materials is best to restore a missing portion


of the mandible?
A. Silastic
B. Tantalum
C. Chrome cobalt
D. Autogenous bone

1. B 2. A 3. D

394
4. A patient with class Il div I malocclusion is operated for genioplasty
his anterior teeth after the operation would be:
A. In edge-to-edge bite
B. Without any change
C. Having normal overjet of 2 mm
D. Having no overbite

5. Jumping genioplasty is a term which connotes:


A. Movement of chin posteriorly
B. Double step genioplasty
C. Single step advancement
D. Advancement after set back of mandibular body

6. Sagittal split osteotomy is a procedure carried out for:


A. Mandibular deformities
B. Maxillary deformities
C. Deformities in which occlusion in not involved
D. Condylar repositioning

4. B 5. B 6. A

395
7. Sagittal split osteomy was first advocated by:
A. Obwegesser
B. Dai Pont
C. Wunderer
D. Moose

8. Apertognathia is a condition in which there is:


A. Retrogenia
B. Maxillary hypoplasia
C. Open bite deformity
D. Maxillary and mandibular prognathism only

9. During genioplasty there are chances of injuring:


A. Inferior alveolar nerve
B. Marginal mandibular nerve
C. Mental nerve
D. Lingual nerve

7. A 8. C 9. C

396
10. Wassmund and Wunderer procedures are:
A. Mandibular segmental osteotomies
B. Maxillary segmental osteotomies
C. Maxillary subapical osteotomies
D. Multiple subapical osteotomy procedure of maxilla and mandible
respectively

11. White graft are:


A. Nerve rejected
B. Are immunologically biocompatible
C. Are rejected without evidence of vascularization
D. Behave in same manner as autogenous grafts

12. The best bone graft which can be utilized for reconstruction of large
mandibular defect is:
A. Chostochondral graft
B. Calvarial graft
C. Iliac crest graft
D. Metatarsal bone graft

10. B 11. C 12. C


397
13. Iliac crest graft should ideally be taken from:
A. Lateral aspect
B. Medial aspect
C. Posterosuperior aspect
D. Anteroinferior aspect

14. A patient in whom iliac crest graft has been taken for mandibular
reconstruction, should be kept nil orally postoperatively:
A. For 6hr
B. Till bowel sounds appear
C. For 12 hours
D. Till patient is ambulatory

15. The graft of choice in a 30-year-old patient of ameloblastic resection


would be:
A. Free iliac crest graft
B. Free vascularized iliac crest graft
C. Medullary bone graft
D. 6th rib

13. B 14. B 15. B


398
16. Alveoplasty should be carried out:
A. When multiple extractions are done in one quadrant
B. When entire arch extraction is there
C. To remove undercuts
D. All of the above

17. The Kazanjian's technique of vestibuloplasty leaves:


A. Lip surface to re-epithelialize
B. Alveolar surface to re-epithelialize
C. Depth of sulcus periosteum to re-epithelialize
D. None of the above

18. The Lipwitch procedure is used for:


A. Ridge augmentation
B. Sulcoplasty
C. Tuberoplasty
D. Chiroplasty

16. D 17. A 18. B

399
19. When there is high crestal attachment of muscle and tissue the
indicated method of vestibuloplasty is:
A. Kazanjian's
B. Clark's
C. Obwegesser's
D. Howe's lipwitch

20. In a patient with class Ill facial profile one would think of which type
of genioplasty:
A. Reduction G
B. Advancement G
C. Straightening G
D. Rotational G

21. A patient reported with class Ill skeletal deformity the ideal choice
would be:
A. Inverted L osteotomy
B. Segmental osteotomy
C. Reverse sagittal split osteotomy
D. Sagittal split osteotomy

19. C 20. A 21. D

400
22. The basic advantage of sagittal split osteotomy is/are:
A. It is carried out intra-orally as well as extra-orally
B. No bone grafting is required when defect is less than 8 mm
C. There are no chances of paresthesia
D. All of the above

23. In a patient in whom SNA is 820 and SNB is 960 indicates he would
require:
A. Maxillary surgery with setback
B. Mandibular surgery
C. Mandibular advancement
D. Maxillary advancement

24. Allografts are grafts taken from:


A. Same species and individuals are genetically related
B. Different species
C. Same species but individuals are genetically not related
D. Same species and between genetically identical individuals

22. B 23. B 24. C

401
25. Composite grafts consist of:
A. Bone only
B. Medullary bone only
C. Bone and soft tissue
D. Particulate bone mixed with resins

26. Intercortical alveoloplasty is done by:


A. Removing margins of cortical plates
B. Removing interseptal bone entirely and collapsing labial and palatal
cortical plates
C. Removing septa till upper third of socket compressing the cortical
plates
D. None of the above

27. The principal problem with tuberosity reduction is:


A. Poor access
B. Formation of OAF
C. Infection
D. Damage to posterior superior alveolar nerve

25. C 26. B 27. B


402
28. Incision for operation of tongue-tie should be placed:
A. Transversely on lingual frenum
B. Longitudinally along lingual frenum on both sides
C. On crest of frenum longitudinally
D. None of the above

29. Surgical treatment of bimaxillary protrusion consists of:


A. Le Fort I osteotomy with sagittal split osteotomy in the mandible
B. Subcondylar osteotomy with Le Fort Il osteotomy
C. Vertical ramus osteotomy only
D. Anterior maxillary osteotomy with anterior sub apical osteotomy of
the mandible

30. The ideal time for repair of a cleft lip is:


A. Immediate after birth
B. 3 weeks to 3 months
C. 3 months to 3 years
D. After puberty

28. B 29. D 30. B

403
31. The blind sub condylar osteotomy for the correction of mandibular
prognathism
A. May damage the internal maxillary artery and cause profuse
hemorrhage
B. May damage the branches of the facial nerve
C. May be used for cases requiring less than 7 mm correction
D. All of the above

32. The extraoral subcondylar ramus osteotomy is indicated for the


following, except:
A. Major setback of the mandible 10 mm)
B. Minor setback of the mandible (K 10 mm)
C. Vertical shortening of the mandibular ramus
D. Asymmetrical mandibular setback

33. The disadvantage of a full thickness graft is:


A. Pigmentary changes
B. Lack of depth for contour
C. Decreased survival chance
D. Marked tendency to contract

31. B 32. B 33. B

404
34. The scalpel blade that should be preferred for incision and drainage
of abscess is:
A. No. 15
B. No. 12
C. No. 11
D. No. 10

35. The following are the basic indications for anterior sub apical
mandibular surgery, except:
A. To correct a mandibular dentoalveolar protrusion
B. To alter the lower third facial height and increase chin projection
C. To level an excessive curve of Spee
D. To correct mandibular dental arch asymmetry

36. The medial bony cut given while performing a modified sagittal
ramus osteotomy extends:
A. 15 to 20 mm posteriorly from the anterior border of the ramus
B. 5 mm above the inferior alveolar neurovascular bundle
C. Through the entire length of the medical ramus up to the posterior
border
D. 5 mm below the sigmoid notch and 10 mm posterior to the anterior
border

34. C 35. B 36. A


405
37. A-50-year-oId male is undergoing radiotherapy for carcinoma left
mandibular body. He requires hemi-mandibulectomy and radical neck
dissection. The ideal time for the surgery would be:
A. Six months after radiotherapy
B. Immediately upon completion of radiotherapy
C. Four to six weeks after completion of radiotherapy
D. Any time during the course of radiotherapy

38. Among the following which is advantage of sagittal split osteotomy


over transoral vertical subcondylar osteotomy?
A. Greater mandibular movement is achieved
B. Because it is intraoral procedure. No external scar is produced
C. Alone may be used to correct a mandibular retrognathism and
prognathism.
D. Less chance of injuring the inferior dental canal

39. Goals of orthognathic surgery includes:


A. Improve periodontal stability and periodontal prognosis.
B. Shorten orthodontic treatment time and improve orthodontic results.
C. To correct jaw relationships prior to major restorative procedures
D. All of the above

37. C 38. C 39. D

406
40. Generally bone marrow for grafting the defects is obtained from:
A. The iliac crest
B. The mandible
C. Maxillary tuberosity
D. Rib

41. Among the following which are characters of an ideal graft?


A. It should withstand mechanical forces.
B. It should produce no immunological response
C. It should actively assist osteogenic process of the host
D. All of the above

42. Advantage of sharp dissection over blunt dissection is:


A. Less traumatic
B. Permits muscle splitting rather than muscle tearing
C. Less likely to severe important nerves
D. A and B

40. A 41. D 42. D

407
43. Which of the following can be treated with a sagittal osteotomy of
the mandible?
A. Mandibular retrognathism
B. Mandibular prognathism
C. Open bite
D. All of the above

44. Bone transplant from one human to another is termed:


A. Allogenous
B. Autogenous
C. Homologous
D. Heterogenous

45. Which of the following bone graft has the greatest osteogenic
potential?
A. Autogenous cortical graft
B. Autogenous cancellous graft
C. A freeze-dried bone graft
D. Xenograph

43. D 44. C 45. B


408
46. To get the best long-term results, which of the following procedures
for augmentation genioplasty are recommended?
A. Injection of silastic gel
B. Onlay bone graft
C. Pedicled horizontal sliding osteotomy
D. Insertion of silastic rubber implant

46. C

409
Chapter 11: Medical Emergency
1. The surgical risk for a patient with organic heart disease depends upon
his:
A. Cardiac reserve
B. Blood pressure
C. Respiration
D. Pulse rate

2. When a localized clot forms intravascular at the site of operation or


injury, it is called:
A. Embolus
B. Thrombus
C. Coagulated hematoma
D. Platelet agglutination

3. A patient is wide-eyed, very nervous with an increased systolic


pressure (with widening of the pulse pressure), increased pulse rate, fine
skin and hair and loss of body weight. He is probably suffering from:
A. Diabetes
B. Hypothyroidism
C. Hyperthyroidism
D. Hyperpituitarism

1. A 2. B 3. C

410
4. Preoperative sedation with short-acting barbiturates
(Numbutal@, Seconal @) is valuable because it:
A. Prevents respiratory depression
B. Produces drowsiness and amnesia
C. Is easily administered and has no side effect
D. Is without hazard and is compatible with narcotics

5. A patient who is on a regimen of steroid therapy and has need for


extraction of chronically infected teeth requires premedication with:
A. Atropine to reduce the hazard of vagal stimulation and cardiac arrest
B. Antihypertensive to combat tendency toward shock
C. Antihistamines to prevent allergic reactions
D. Antibiotics

6. Differential white blood cell counts in the laboratory are useful in the
diagnosis of:
A. Anemia
B. Eosinophilia
C. Spherocytosis
D. Vitamin deficiency

4. B 5. D 6. B
411
7. Postoperatively, a patient may develop serum sickness as a reaction to
a non-protein drug or to a bio- logic product. The reaction occurs after a
sensitization period of:
A. 1 day
B. 3 days
C. 4 days
D. 1-3 weeks

8. Complete respiratory obstruction is first manifested


A. Prolonged expiration
B. No movement by the patient
C. Information given by the patient
D. Pronounced retraction of the intercostal and supraclavicular spaces

9. The immediate postoperative complication following aspiration of


liquid vomitus into trachea and bronchi is:
A. Pleurisy
B. Bronchitis
C. Atelectasis
D. Lung abscess

7. D 8. D 9. B

412
10. In administering artificial ventilation, it is recommended that the
rescuer deliver a resting tidal vol.
A. Normal
B. Twice the normal
C. Three times the normal
D. Four times the normal

11. A patient who is currently taking Coumadin may recently have had:
A. Thrombophlebitis
B. A pulmonary embolism
C. Acute myocardial infarction
D. Any of the above

12. Patients suffering from uncontrolled juvenile onset diabetes mellitus


are usually poor candidates for oral surgery because of:
A. The high incidence of wound infection and poor healing
B. Their inability to follow postoperative instruction
C. Their tendency to hemorrhage profusely
D. Abnormal protein metabolism

10. B 11. D 12. A


413
13. Which of the following always indicated obstruction to the airway?
A. Slow pounding pulse
B. Stertorous breathing
C. Increase in pulse rate
D. Decrease in blood pressure

14. Which of the following procedures is correct for treating an


unconscious patient with no pulse and dilated pupils?
A. Start closed chest massage at once
B. Start artificial ventilation at once
C. Start closed chest massage in conjunction with artificial ventilation
D. Start closed chest massage if the pupils do not constrict with light

15. Hemostasis depends upon each of the following except:


B. Prothrombin
A. Vessel wall
C. Blood platelets
D. Albumin

13. B 14. C 15. D

414
16. The earliest sign of hemorrhagic shock is
A. Dyspnea
B. Hypotension
C. Tachycardia
D. Vasoconstriction

17. In shock, vasopressor drugs are preferably given:


A. Sublingually
B. Intravenously
C. Intracardially
D. Subcutaneously

18. The prothrombin time that would cause the practitioner the greatest
concern before a surgical extraction is:
A. 20 percent of normal
B. 40 percent of normal
C. 50 percent of normal
D. 80 percent of normal

16. C 17. B 18. A

415
19. Two minutes following cessation of a two-hour treatment session
using 80 percent nitrous oxide, 20 percent oxygen and local anesthesia,
the patient becomes cyanotic and tachycardia:
A. Over-oxygenation
B. Under-oxygenation
C. Diffusion hypoxia
D. Malignant hyperthermia

20. In order to carry cardiac compression effectively, the sternum should


be depressed:
A. 2 inches every second
B. 5 inches every second
C. 2 inches every 5 second
D. 1 inch every 10 second

21. Which of the following drugs used for premedication produce a


detached serenity without clouding consciousness?
A. Ataractics
C. Analeptics
B. Antiemetics
D. Anesthetics

19. C 20. A 21. A

416
22. Which of the following laboratory studies would be expected to be
abnormal in a patient with hemophilia
A. Partial thromboplastin time
B. Prothrombin time
C. Platelet count
D. Bleeding time

23. Narcotics are used primarily:


A. To control anxiety
B. To counteract depression
C. For the treatment of pain
D. For their ataractic effect

24. A patient with a history of ankle edema, shortness of breath and


orthopnea is probably suffering from:
A. Asthma
B. Emphysema
C. Rhinophyma
D. Cardiac decompensation

22. A 23. C 24. D


417
25. Ultra short-acting barbiturates produce loss of consciousness by
depression of the:
A. Thalamus
B. Spinal cord
C. Limbic system
D. Ascending portion of the reticular activating system

26. Use of which of the following antibiotics is most restricted because


of its side effects?
A. Penicillin
B. Tetracycline
C. Cephalexin
D. Chloramphenicol

27. Abnormal coagulation time will occur in a patient with which of the
following conditions?
A. Hemangioma
B. Hemophilia
C. Thalassemia
D. Pernicious anemia

25. D 26. D 27. B

418
28. When vasoconstrictors produce constriction of arterioles, the site of
action is at which receptor?
A. Alpha
B. Beta
C. Gamma
D. Delta

29. External cold application to achieve hemostasis causes:


A. A transient/ vasoconstriction
B. Increased chemotaxis
C. Active component which assists in hemostasis
D. B and C

30. Hepatitis that commonly occur with multiple transfusion due to:
A. Hepatitis A
B. Hepatitis B
C. Non-A: Non-B
D. All of the above

28. A 29. A 30. B

419
31. Which of the following condition can be diagnosed by differential
white blood?
A. Anemia
B. Spherocytosis
C. Thrombocytopenic purpura
D. Eosinophilia

32. After intravenous administration of a short acting barbiturates which


tissue is saturated last as a result of re-distribution:
A. Liver
B. Brain
C. Muscle
D. Fat

33. After the injection of a test dose of IV sedation a patient complains


of immediate severe burning pain at the site of injection, radiating
distally in the involved arm. Simultaneously the colour of the arm
appears blackly with several blanched areas. The pulse is regular but
weak compared to the opposite side likely diagnosis is:
A. Injection into the radial nerve
B. Injection into biceps tendon
C. Injection into an artery
D. Impeding anaphylactic reaction

31. D 32. D 33. C

420
34. Which of the following is best treatment for hypoglycemia in an
unconscious diabetic patient?
A. Intravenous administration of 50% dextrose in water
B. Administration of oral carbohydrates
C. Sublingual injection of 50% dextrose
D. None of the above. No treatment is necessary

35. Which of the following may occur due to hyperventilation in an


anxious patient?
A. Acidosis
B. Elevated pC02 M
C. Carpopedal spasm
D. Cyanosis

36. Of the following which is treatment of sustained convulsive reaction


to a local anesthesia?
A. Oxygen and IV fluids
B. Diazepam and oxygen
C. Phenytoin
D. Slow acting barbiturates and oxygen

34. A 35. C 36. B

421
37. Of the following which nerve is affected in Saturday night palsy?
A. Ulnar
B. Lingual
C. Radial
D. Median cephalic

38. A patient came to the emergency ward who needs immediate


transfusion of blood. There is no time for blood grouping and cross
matching which of the following group of blood can be transfused?
A. Rh positive
B. ABRh positive
C. Rh negative
D. ABRh negative

39. Risus sardonicus is a sign of:


A. Rabies
B. Tetanus
C. Poliomyelitis
D. Diphtheria

37. C 38. C 39. B


422
40. All of the following tissues contain alkaline phosphates except:
A. Bone
B. Muscle
C. Prostate
D. Kidney

41. Which of the following indicates a localized clot formation


intravenously at the site of injury?
A. Thrombus
B. Platelet agglutination
C. Embolus
D. Organized hematoma

42. In artificial ventilation it is commonly recommended that the rescuer


delivers a resting tidal volume that is:
A. Four times the normal
B. Three times the normal
C. Twice the normal
D. Normal

40. C 41. A 42. C


423
43. In an elective tracheostomy the entry should be made:
A. Above the cricoid
B. Below the cricoid
C. Through the cricothyroid membrane
D. Laterally below the thyroid cartilage

44. Patients who are suffering with uncontrolled juvenile onset diabetes
mellitus are usually poor candidates for surgical procedure because of:
A. Their inability to follow postoperative instructions
B. The high incidence of wound infection and poor healing
C. Their tendency to bleed profusely
D. Their oral hygiene

45. All of the following play a role in hemostasis except:


A. Vessel wall
B. Blood platelets
C. Albumin
D. Prothrombin

43. B 44. B 45. C

424
46. Universal distress signal, characterizing the obstructed airway in a
conscious adult is:
A. Rapid heavy breathing
B. Victims’s hand at his throat
C. Violent choking
D. Violent thrashing of the victim's arm

47. Hemophilia-A is:


A. X-Linked recessive
B. X-Linked dominant
C. Autosomal recessive
D. Autosomal dominant

48. Normal factor VIII levels in the blood is:


A. 5.0-6.5 1.u/ml
B. 3.0-4.5 1.u/ml
C. 1.5-2.5 1.u/ml
D. 0.5-1.5 1.u/ml

46. B 47. A 48. D

425
49. Epsilon amino caproic acid (EACA) is one of the agents useful in the
treatment of hemophilia. A mechanism of action of EACA is:
A. It is rich in factor V fll, which is deficient in hemophilia
B. Antifibrinolytic activity
C. It accelerates the coagulation process which is usually delayed in
hemophilia
D. By unknown mechanism

50. Because of many side effects of (EACA) it is largely replaced by:


A. Desmopressin
B. AC A (Amino caproic acid)
C. Tranexamic acid
D. Cryoprecipitate

51. All of the following conditions characterized by neutrophil


leukocytosis (increased neutrophils) except:
A. Neonatal period
B. Corticosteroid therapy
C. Typhoid
D. Periapical abscess

49. B 50. C 51. C

426
52. Of the following which condition is characterized by decreased
eosinophils?
A. Aplastic anemia
B. Periarteritis nodosa
C. Amoebiasis
D. Psoriasis

53. Which is factor IV?


A. Thromboplastin
B. Calcium
C. Christmas factor
D. Fibrinogin

54. of the following which is present in both intrinsic and extrinsic


pathway?
A. Factor Vlll
B. Factor Xll
C. Factor V
D. None of above

52. A 53. B 54. C


427
55. of the following which is screening test for scurvy?
A. Tourniquet test
B. Immunoassay for total body Vitamin C
C. Melkerson – Rosenthal test
D. None of the above

56. Prolonged prothrombin time is seen in all of the following except:


A. Factor VII deficiency
B. In patient with coumarin therapy
C. Factor V deficiency
D. Factor XI deficiency

57. Serum Alkaline phosphatase increase in all of the following


conditions except:
A. Scurvy
B. Hyperparathyroidism
C. Paget's disease
D. None of the above

55. A 56. D 57. A

428
58. Of the following what would be the WBC range in mild infections:
A. 4000 - 8000 cells/mm3
B. 24,000 - 30,000 cells/mm3
C. 15,000 – 20,000 cells/mm3
D. None of the above

59. Of the following which is earliest sign of hemorrhagic shock?


A. Vasoconstriction
B. Hypotension
C. Tachycardia
D. Dyspnoea

60. Near accurate body temperature can be obtained if one measures:


A. Orally
B. Axillary
C. Rectally
D. Temperature over forehead

58. C 59. C 60. C

429
61. Of the following which indicates early oxygen want?
A. Cyanosis
B. Increased pulse rate
C. Bradycardia
D. A and B

62. Among the following where is tetanus likely to occur?


A. Avulsive wounds
B. Perforating wounds
C. Deep puncture wounds
D. Contaminated laceration

63. Cold packs commonly used:


A. To produce local vasodilation
B. Immediately after surgery
C. To increase permeability
D. To prevent the spread of infection

61. D 62. C 63. B

430
64. Which of the following is a constant finding in systemic infection:
A. Fever
B. Swelling
C. Bacteremia
D. Lymphadenopathy

65. Immediate post-surgical swelling can be best reduced by:


A. Gentle manipulation of soft and hard tissue and early application of
cold packs
B. Judicious administration of antihistamines
C. Judicious administration of non-steroidal anti-inflammatories
D. None of the above

66. Surgical risk for a cardiac patient depends upon his:


A. Blood pressure
B. Respiration
C. Pulse rate
D. Cardiac reserve

64. A 65. A 66. D

431
67. Hemorrhagic shock is characterized by:
A. Hypotension
B. Low blood volume
C. Increased pulse rate
D. All of the above

68. Of the following which is most useful in evaluating a patient for an


abnormal bleeding tendency?
A. History
B. Bleeding time
C. Prothrombin time
D. Rumple-Leede test

69. Treatment: Syrup Brufen 2 teaspoon TID for 2 days.


What is the volume of teaspoon and what is the meaning of TID?
A. 5 ml and twice in a day
B. 5 ml and thrice in a day
C. 15 ml and thrice in a day
D. 15 ml and twice in a day

67. D 68. A 69. B

432
70. Which of the following is first manifestation of complete respiratory
obstruction?
A. Pronounced retraction of intercostal and supraclavicular spaces
B. Prolonged expiration
C. Cyanosis
D. None of the above. No changes in the patient

71. On examination it is noted that a patient requires 5 to 6 seconds to


rapidly and completely exhale after a deep inspiration. He may be
suffering with:
A. Upper air way disease
B. Advanced pulmonary disease
C. Severe cardiovascular disease
D. Normal exhalation time

72. Which of the following may result in interruptions in cardiac


compressions?
A. Little changes in blood flow and blood pressure
B. A reduction of the blood flow and blood pressure to zero
C. Carbon dioxide buildup in the lungs
D. None of the above

70. A 71. B 72. B

433
73. Which of the following is true of cardiopulmonary resuscitation?
A. Compression to ventilation ratio in two person CPR is 5:1
B. Compression to ventilation ratio in single person CPR is 15:2
C. Compression should be 60-80/minute in adults
D. All of the above

74. In a patient presenting for dental surgery with the history of chest
pain upon exertion, which is relieved by rest and nitroglycerin, one
would suspect:
A. Pneumothorax
B. Fractured rib
C. Myocardial infarction
D. Angina pectoris

75. A patient on dicumerol therapy should have which of the following


prior to dental surgery?
A. Vitamin K
B. A sedative
C. Prothrombin time test
D. An antibiotic

73. D 74. D 75. C

434
76. In Pierre-Robin syndrome, a complete upper airway obstruction may
occur when the infant is in a supine position or during feeding. Due to
the nature of this obstruction, it may be handled by putting the patient
into a prone position. If this fails, the next procedure should be:
A. Intubation
B. Nasal airway
C. Tracheostomy
D. Displacing mandible forward

77. A patient in shock should:


A. Be placed in Trendelenburg's position
B. Be placed in reverse Trendelenburg's position
C. Have his legs elevated 20 degrees with trunk and head remaining
level
D. Have his arms elevated 20 degrees with trunk and head remaining
level

76. D 77. C
435
78. In Pierre-Robin syndrome, a complete upper airway obstruction may
also occur when the patient is in a prone position. This may be handled
by:
A. Intubating the patient
B. Performing a tracheotomy
C. Displacing the mandible forward
D. Inserting a nasal catheter with oxygen under pressure

79. Before any oral surgical procedure is carried out, prophylactic


antibiotic cover is indicated for patients with the following conditions,
except:
A. Rheumatic heart disease
B. Chronic renal failure
C. Prosthetic heart valve
D. Coronary artery bypass

78. C 79. D

436
80. A patient requiring tooth extraction is taking anti-coagulant drugs for
the past six months. His prothrombin time is 21 seconds and the control
is 15 seconds.
The treatment under these circumstances should include:
A. Preoperative administration of vitamin K
B. Postoperative administration of vitamin K
C. Discontinuation of the anticoagulant drugs for one week prior to the
extraction
D. Extraction of the tooth with proper local care to control bleeding

81. When two rescuers are performing cardiopulmonary resuscitation on


an adult, how many compressions are to be given per minute?
A. 20
B. 40
C. 60
D. 80

82. Inhaled aromatic spirits of ammonia act against syncope by:


A. Direct stimulation of the respiratory center
B. Direct stimulation of the vasomotor center
C. Irritating the sensory endings of the olfactory nerves
D. All of the above

80. D 81. C 82. C

437
83. The minimal acceptable value for a hematocrit for elective oral
surgery is:
A. 20
B. 30
C. 40
D. 50

84. For the ligation and exposure of the external carotid artery in the
carotid triangle, the incision is made from:
A. The angle of the mandible to the cricoid cartilage
B. The tip of the mastoid process to one inch below the mandibular
angle
C. The angle of the mandible to the hyoid bone
D. The level of the hyoid bone to the cricothyroid cartilage

85. Intraoperative venous haemorrhage can be controlled by the


following methods except:
A. Electrocoagulation
B. Ligation with hemostats
C. Bone wax tamponade
D. Pressure packing with gauze

83. B 84. A 85. C

438
86. In order to carry out cardiac compression effectively, the sternum
should be depressed:
A. 1 inch every second
B. 1.5 to 2 inches per second
C. 2 inches per five seconds
D. 1 inch every ten seconds

87. A pregnant woman in her third trimester loses consciousness on the


dental chair, she should immediately be:
A. Placed in the Trendelenburg's position
B. Lowered in the horizontal position and turned toward her right side
C. Placed in the supine position with the feet elevated slightly
D. Lowered in the supine position and turned toward her left side

88. The importance of chronic alcoholism prior to oral


A. Predisposition to syncope
B. Relative contraindication to local anaesthesia
C. Prolonged bleeding secondary to liver dysfunction
D. Contraindication for use of NSAIDs

86. B 87. D 88. C


439
89. An odor of acetone in one's breath should direct suspicion to:
A. Heart disease
B. Liver damage
C. Diabetes mellitus
D. Kidney disease

90. Extraction of four teeth is required in a 65-year-old diabetic female


who has had her morning insulin, pre- operative instruction should
include:
A. Do not eat or take any medication by mouth prior to surgery
B. Increase sugar intake prior to surgery
C. Maintain normal diet
D. Increased premedication

91. A patient on recent prolonged steroid therapy may have:


A. Relative adrenal insufficiency
B. Intolerance to sedatives
C. Prolonged post extraction haemorrhage
D. Relative analgesic potential

89. D 90. C 91. A


440
92. Normal bleeding time by Duke's method:
A. 1 to 5 min
B. 3 to 5 min
C. 3 to 8 min
D. 10 to 30 sec

93. Treatment required for post-operative cutaneous ecchymosis is:


A. Aspiration of blood
B. Use of topical steroids
C. Application of cold
D. None

94. The rate of infusion of intravenous diazepam is:


A. 3 ml/ min
B. 1 mg/ min
C. 1 ml/ min
D. 3 mg/ min

92. A 93. C 94. C

441
95. The following elements interfere with the activity of tetracycline:
A. Ca and Bi
B. Na and K
C. Mg and Na
D. Mg and Ca

96. Class of drugs are currently, the best oral sedative drugs for
dentistry:
A. Narcotics
B. Barbiturates
C. Benzodiazepenes
D. Ethanols

97. Among the following which is contraindicated in patients taking


sodium warfarin therapy?
A. Acetaminophen
B. Diphenhydramine
C. Ibuprofen
D. Codeine

95. D 96. C 97. C


442
98. Intravenous diazepam administration sometimes causes phlebitis.
This usually attributed to the presence of:
A. Ethyl alcohol
B. Benzoic acid
C. Propyleneglycol
D. Methyl paraben

99. Which of the following are symptoms of digitalis toxicity:


A. Resting pulse less than 60 beats/mm
B. Arrhythmias and mental disturbances
C. Nauseas
D. All of the above

100. Which drug is contraindicated in patients taking so-


A. Aspirin
B. Morphine
C. Benadryl
D. Pentobarbital

98. C 99. D 100. A

443
101. But mainly in the liver, procaine reduces the effectiveness of the
following drug group:
A. Sulfonamides
B. Penicillin
C. Aminoglycosides
D. Cephalosporin

102. One of your patients stated that he is narcotic addict. After the
extraction which of the following analgesics are contraindicated?
A. Aspirin
B. Pentazocin
C. Acetaminophen
D. None of the above

103. Prophylaxis for tetanus in a previously immunized person involves


the use of tetanus toxoid as a booster in which manner?
A. 5 ml subcutaneously
B. 5 ml orally
C. 1 ml intramuscularly
D. 3 ml intravenously

101. A 102. B 103. C

444
104. A RHD patient requires preoperative prophylaxis with penicillin.
An appropriate regimen is:
A. 2 gm penicillin V 1 hour before surgery
B. 600,000-unit procaine penicillin 1M 12 hourly days before surgery
C. 250 mg phenoxy methyl penicillin 6 hrs day before surgery
D. All of the above are correct

105. Glucocorticoids:
A. Decreases inflammation
B. Decreases pain threshold
C. Increases the rate of wound healing
D. Enhances the repair

106. Among the following which analgesic is contraindicated in patients


with G6PD deficiency?
A. Aspirin
B. Paracetamol
C. Opioid analgesic
D. None of the above

104. A 105. A 106. A

445
107. How many postoperative days one should continue antibiotic
coverage for sub-acute bacterial endocarditis?
A. One
B. Two
C. Three
D. Four

108. Among the following which is "Broad spectrum" antibiotic:


A. Sulphanamides
B. Penicillin
C. Tetracycline
D. None of the above

109. Mechanism of action of penicillin is due to:


A. Damage to the cytoplasmic membrane
B. Interference with cell wall synthesis
C. Antimetabolite action
D. None of the above

107. B 108. C 109. B

446
110. Which of the following antibiotic mainly effective against gram-
negative bacteria:
A. Lincomycin
B. Vancomycin
C. Kanamycin
D. Clindamycin

111. Among the following which drug is commonly administered to a


patient with an allergic reaction who is demonstrating urticaria and
wheezing?
A. Steroid
B. Atropine
C. Dexamethasone sodium
D. Adrenalin

112. The following drugs can be used for conscious sedation:


A. Diazepam
B. Phenobarbital
C. Chloral hydrate
D. All of the above

110. C 111. D 112. D

447
113. Among the following which drug is least rapidly acting in treatment
of anaphylaxis?
A. Adrenaline
B. Hydrocortisone
C. Diphenhydramine
D. Aminophyline

114. Among the following which is correct regarding oral route of drug
administration?
A. Its effect is most predictable
B. Quickest way to administer drug
C. It is usually a pleasant route
D. A and C

115. Of the following which group of drugs commonly cause respiratory


depression?
A. Antihistamines
B. Tranquilizers
C. Synthetic narcotics
D. Non barbiturate sedative hypnotics

113. B 114. C 115. C

448
116. In which of the following conditions corticosteroid administration
is contraindicated?
A. Peptic ulcer
B. Psychosis
C. Tuberculosis
D. All of the above conditions

117. Administration of short acting barbiturates for preoperative


sedation is valuable because it:
A. Is easily administered and has no side effect
B. Produces drowsiness and amnesia
C. Prevents respiratory depression
D. Is compatible with narcotics and has no hazards

118. Of the following which is effective agent for the treatment of


respiratory depression which is the result of over dosage of barbiturates?
A. Caffeine
B. Oxygen
C. Metronidazole
D. None of the above

116. D 117. A 118. B

449
119. Al of the following are important in the management of a patient
depressed from known over dose of morphine except:
A. Assistance of ventilation
B. Provision of a patent airway
C. Narcotic antagonist
D. Tranquilizer

120. Which of the following is the drug of choice for treatment of an


infection which is caused by a gram-positive, non-penicillin’s
staphylococci organism?
A. Clindamycin
B. Erythromycin
C. Penicillin G
D. Cephalosporin

121. Usual adult dosage of oral codeine is:


A. 0.5 - 1.0 grains
B. 0.5- 1.0 grains
C. 1.0-2.0 grains
D. 2.0 grains

119. D 120. C 121. A

450
122. Intra articular injections of steroids into painful joint such as
arthritic TMJ, have palliative effects from:
A. Providing an analgesic effect
B. Providing better lubricating effect
C. Providing an anti-inflammatory action
D. Increasing the blood supply

123. Penicillin is not effective against certain organisms because the


causative organism:
A. Produces an enzyme inhibiting the activity of the drug
B. Produces catalase
C. Is not a gram-negative organism
D. None of the above are correct. Penicillin works on almost all the
organisms

124. Which of the following drug administration produces the most


rapid relief of the symptoms of angina pectoris?
A. Pentobarbital
B. Amyl nitrate
C. Oxygen
D. Phenobarbital

122. C 123. A 124. B

451
125. History of bruising easily, nocturia, excessive thirst and low
resistance to infections indicate the patient is most likely suffering from:
A. Diabetes mellitus
B. Glomerulonephritis
C. Lupus erythematous
D. Thrombocytopenic purpura

126. Spider telangiectatic spots on the skin of a patient are noted. He


should have an examination to determine the condition of his:
A. Kidneys
B. Liver
C. Lungs
D. Gallbladder

127. A 13-year-old girl has a history of severe sore throat, migratory


type of joint pains and swollen joints of the extremities. This history is
suggestive of:
A. Gout
B. Serum sickness
C. Rheumatic fever
D. Osteoarthritis

125. A 126. B 127. C

452
128. Laboratory data of patient indicates white blood cell count more
than most likely the patient is suffering from:
A. Leukopenia
B. Polycythemia
C. Leukemia
D. Anemia

129. Clinical hemophilia is characterized by (BT - bleeding time, CT -


clotting time, PT- Prothrombin time, PTT- Partial thromboplastin time:
A. Prolonged PT and normal BT
B. Prolonged BT, PT and PTT
C. Normal BT and PT and prolonged PTT
D. Prolonged BT and normal CT; PT and PTT

130. Severe liver disease may cause the following:


A. Decreased quality of circulating platelets
B. Decreased prothrombin production
C. Increased drug detoxification and decreased drug effects
D. None of the above

128. C 129. C 130. B


453
131. Characteristic featured of hypovolemic shock include:
A. Hypotension
B. Low pulse pressure
C. Tachycardia
D. All of the above

132. "Vital signs" include which of the following?


A. TPR (temperature, pulse, respiration)
B. BP (blood pressure)
C. All of the above
D. None of the above

133. Among the following in which condition one might see


Hutchinson's incisors?
A. Acquired syphilis
B. Congenital syphilis
C. Secondary syphilis
D. Tertiary syphilis

131. C 132. C 133. B

454
134. Among the following which is absent in an asthmatic patient?
A. Hyperactive bronchi
B. Inspiratory wheezes
C. Bronchi that rapidly and spontaneously change caliber in response to
various stimuli
D. History suggestive of allergy

135. Oral examination reveals unilateral lesions along the distribution of


nerve, localized hyperemic and vesicle formation vesicles filled with
yellow fluid and when it ruptures, it leaves an eroded area surrounded by
a red areola. The lesion most likely is:
A. Lichen planus
B. Herpes zoster
C. Pemphigus
D. Fordyce diseases

136. Acetone odour in one's breath usually indicate the person is


suffering with:
A. Heart disease
B. Diabetes mellitus
C. Kidney disease
D. Liver damage

134. B 135. B 136. B

455
137. A patient came to oral surgery department who has bleeding from
the gums, acute gingival hyper trophy, along with complaining of
weakness and anorexia. His blood picture showing the:
HBO/0 14 gm, RBC count: 4.5 million/mm3, WBC 1, 10,000, DC
Neutrophils-88% Lymphocyte - 10%, Monocytes - 0%, Eosinophil’s -
2%
Likely diagnosis of the above findings
A. Infectious mononucleosis
B. purpura
C. Myelogenous leukemia
D. Marked gingivitis due to local cause

138. Which of the following preoperative hematological analysis may


reflect von Willebrand disease?
A. Bleeding time
B. Factor X level assay
C. Platelet count
D. Factor IX level assay

137. C 138. A
456
139. Among the following which is absolute contraindication to the teeth
extraction?
A. Heart murmur
B. Patient on anticoagulant therapy
C. Patient taking steroids
D. None of the above

140. One patient states he has recently become excessively thirsty


(polydipsia), hungry (polyphagia) and arises at night several times to
urinate (polyuria). The most probable diagnosis -is:
A. Acute dehydration
B. Congestive heart failure
C. Diabetes mellitus
D. Renal failure

141. Among the following in which condition eosinophilia is not seen?


A. Parasitic infection
B. Allergy
C. Hodgkin's disease
D. Cushing's syndrome

139. D 140. C 141. D

457
142. Among the following which physical sign would deter you from
proceeding with multiple extraction and alveoloplasties without further
evaluation?
A. Hepatomegaly
B. Seleral icterus
C. Ascites
D. All of the above

143. If one considers BT (bleeding time), PT (prothrombin time) PTT


(partial thromboplastin time) which of the following represent vascular
hemophilia?
A. Normal BT PTT and prolonged BT
B. Prolonged BT, PT and PTT
C. Prolonged PT, moderately prolonged PTT and normal
D. Prolonged BT, moderately prolonged PTT and normal

144. Macroglossia seen in all of the following conditions except:


A. Cretinism
B. Von Reck Linghausen's disease
C. Scurvy
D. None of the above

142. B 143. D 144. D

458
145. Patient history reveals dyspnea, orthopnoea, edema of the ankle and
palpitation most likely diagnosis is:
A. Respiratory problem
B. Hepatic failure
C. Uremia
D. Congestive heart failure

146. An anxious, nervous patient state that he has had recent weight loss
and is easily fatigued. Tremors, tachycardia and tremors, sweaty palms
are noted in physical examination most likely diagnosis is:
A. Renal disease
B. Hyperthyroidism
C. Diabetes
D. Cushing's disease

147. Of the following which represent complete respiratory obstruction?


A. Prolonged expiration
B. No movement by the patient
C. Pronounced retraction of the intercostal and supraclavicular spaces
D. Cyanosis

145. D 146. B 147. C

459
148. Of the following which is a feature of acromegaly?
A. Micrognathia
B. Hypoglycemia
C. Crowded teeth
D. Large tongue

149. In which of the following conditions one can see neck swelling?
A. Hodgkin's disease
B. Infectious mononucleosis
C. Tuberculosis
D. All of the above

150. Among the following which bone lesion is most fatal?


A. Paget's disease
B. Osteochondroma
C. Multiple myeloma
D. Odontogenic myxoma

148. D 149. D 150. C

460
151. Increased Bence-Jones proteinuria and multiple radiolucent areas in
the skull indicate which of the following conditions?
A. Hodgkin's lymphoma
B. Burkitt's lymphoma
C. Multiple myeloma
D. Adeno carcinoma

152. All of the following are signs of thyrotoxicosis except:


A. Slow pulse rate
B. Exophthalmos
C. Tremor
D. Temperature elevation

153. Cyanosis of the lip may suggest:


A. Carbon monoxide poisoning
B. Anemia
C. Congenital heart disease
D. All of the above

151. C 152. A 153. C

461
154. The asthmatic patient is characterized by:
A. Recurrent pulmonary infections
B. Multiple pulmonary emboli which may be life threatening
C. Hyperactive bronchi, that rapidly and spontaneously change calibre in
response to various stimuli
D. None of the above

155. An obese patient falls while jogging and is unconscious which of


the following should be considered to differentiate between cardiac
arrest and other causes:
A. Pulse (carotid (or) femoral)
B. Pupil constriction/ dilatation
C. Respirations present (or) absent
D. All of the above

156. Of the following which is possible sign of sub-acute bacterial


endocarditis?
A. Enlarged tongue
B. Discharge from the ear
C. Splinter hemorrhage
D. Intestinal obstruction

154. C 155. D 156. C

462
157. Which is the minimal acceptable value for a hematocrit for elective
surgery?
A. 30
B. 25
C. 15
D. 5

158. If one uses normal cuff of Sphygmomanometer on an adult obese


patient it will yield diastolic pressure readings that are:
A. Falsely high
B. Falsely low
C. Accurate but systolic pressure readings are falsely high
D. Accurate but systolic pressure readings are false

159. Among the following which is true regarding phlebitis:


A. High doses of antibiotics are necessary to accelerate resolution
B. Application of ice packs to the area during the first 2 days is indicated
C. Local application of heat and immobilisation for several days is
indicated
D. Abscess formation is quite common

157. A 158. A 159. C

463
160. The primary airway hazard for an unconscious patient in a supine
position is:
A. Tongue obstruction
B. Bronchospasm
C. Laryngospasm
D. Aspiration

161. The best blood product administered preoperatively to patients with


hemophilia A is:
A. Fresh frozen plasma
B. Factor IX concentrate
C. Whole blood
D. Factor V Ill concentrate

162. Cardinal symptom of dehydration due to disturbance of fluid and


electrolyte balance is:
A. Hypothermia
B. Nausea
C. Polydipsia
D. None of the above

160. A 161. D 162. C

464
163. Emotional stress affect the rate of absorption of oral medication.
Which of the following is correct?
A. It decreases the rate of absorption
B. It increases the rate of absorption
C. The above statement is wrong there is no effect on the rate of
absorption
D. It increases the rate of absorption of few drugs and decreases the rate
of absorption of few drugs

164. If efforts in cardiopulmonary resuscitation are effective there will


be
A. Constriction of pupils
B. Dilatation of pupils
C. Hypertension immediate
D. None of the above

165. Patient with idiopathic thrombocytopenic purpura (ITP) is most


likely to have which of the following post- operative complication?
A. Infection
B. Hemorrhage
C. Edema
D. Localized alveolar osteitis (dry socket)

163. A 164. A 165. B


465
166. Complete blood count does not include:
A. Hemoglobin
B. Hematocrit
C. Total WBC
D. ESR

167. All of the following are various methods to control intraoperative


hemorrhage except:
A. Gauze sponge pressure
B. Artery application to the open vessels
C. Infiltration with lidocaine
D. Bone compression for surface bleeders

168. Pin point hemorrhages on skin are called as:


A. Purpura
B. Hematoma
C. Petechiae
D. Ecchymosis

166. D 167. C 168. C

466
169. When one suspects tetanus organisms in a wounded patient. Anti-
tetanus prophylaxis given in the form of:
A. Tetanus antitoxin (if the patient is previously not immunized)
B. Penicillin (if there is no allergy)
C. Tetanus toxoid
D. All of the above

170. Al patients admitted to the hospital for surgery must


A. Give verbal permission
B. Sign an operative permit after receiving premedication
C. Sign an operative permit before the discharge
D. Sign an operative permit at the time or admission to the hospital

171. One suspected rheumatoid arthritis of the TMJ helpful positive


laboratory study would be:
A. Hematocrit elevation
B. GTT
C. Increased erythrocyte sedimentation rate
D. Tourniquet test or Hess test

169. D 170. D 171. C


467
172. Which of the following is early sign of want of oxygen?
A. Tachycardia
B. Cyanosis
C. Sternal relaxation
D. Constricted pupils

173. Treatment of syncope is most effectively accomplished by which of


the following?
A. Administering 100% oxygen for five minutes
B. Lowering the chair to place the patient's head lower than his feet
C. Bending the patients head forward between his knees and asking the
patient to raise his head against hand pressure
D. It requires treatment by a physician

174. Which of the following causes postoperative infection:


A. Lack of asepsis
B. Failure to remove pathology
C. Inadequate debridement
D. Any of the above

172. A 173. B 174. D


468
175. Among the following, which is best hemostatic agent for control of
local hemorrhage?
A. Gel foam
B. Surgical
C. Bone wax
D. Vit. K

176. Among the following which causes the practitioner the greatest
concern?
A. 80% of normal PTT
B. 50% of normal PTT
C. 30% of normal PTT
D. 20% of normal PTT

177. During CPR sternum should be depressed:


A. Two inches every 5 seconds
B. 2 inches every second.
C. 1 inch every 10 seconds
D. 3 inches every 5 seconds

175. B 176. D 177. B


469
178. Which of the following agent should be administered to a patient
with moderate liver disease who has an abnormal prothrombin time and
bleeds excessively after surgery?
A. Packed red cells
B. Platelet accelerator
C. Vitamin K
D. Fresh whole blood

179. After administration of inferior alveolar nerve block to an addict,


the needle accidentally pricks the dentist's finger. Subsequently the
dentist developed malaise weakness, and elevated SCOT and SGPT.
Most probably he contacted which of the following disease?
A. Serum hepatitis
B. Infectious hepatitis
C. Infectious mononucleosis
D. Primary syphilis

180. Is common to all forms of shock:


A. Hypovolemia
B. Hypertension
C. Impaired tissue perfusion
D. Vasoconstriction

178. C 179. A 180. C


470
181. Which of the following may result due to the prolonged use of
phenytoin sodium?
A. Jaundice
B. Yellow pigmentation on teeth
C. Gingival hypertrophy.
D. It is a safe drug with absolutely no side effects

182. In patients on artificial ventilators the cycle of exhale air ventilation


should be repeated every:
A. 20 seconds
B. 10 seconds
C. 5 seconds
D. 1 second

183. The following factors influence the sedation dosage of intravenous


agents:
A. Weight of the patient
B. Age of the patient
C. Type of agent used
D. All of the above

181. C 182. C 183. D

471
184. Among the following which steps, followed in completing physical
examination?
A. Inspection
B. Palpation and percussion
C. Auscultation
D. All of the above

185. Which of the following antibiotics is commonly prescribed for


penicillin allergic patients?
A. Tetracycline
B. Erythromycin
C. Chloramphenicol
D. Cephalosporin

186. Which cardiac condition require antibiotic prophylaxis before


surgery?
A. Coronary insufficiency
B. Angina pectoris
C. Rheumatic carditis
D. Congestive heart diseases

184. D 185. B 186. C

472
187. Frequent site of occurrence for ameloblastomas:
A. Mandibular premolar area
B. Maxillary molar area
C. Antrum and floor of the nose
D. Molar and ramus area of the mandible

188. Among the following which is short acting barbiturate?


A. Pentobarbital
B. Phenobarbital
C. Valium
D. Codeine

189. Site of action of valium (Diazepam) is:


A. Cerebellum
B. Limbic systems
C. Cerebral cortex
D. Pyramidal system

187. D 188. A 189. B

473
190. Among the following which drug is contraindicated in hyperthyroid
patients, because the subjects are extra-ordinarily sensitive to the drug?
A. Salicylates
B. Barbiturates
C. Adrenaline
D. Digitalis

191. Among the following which frequently causes infective


endocarditis?
A. Staphylococcus aureus
B. Streptococcus viridans
C. Staphylococcus pyogens
D. None of the above

192. The emergency, most frequently encountered during outpatient


general anesthesia is:
A. Anaphylaxis
B. Bradycardia
C. Respiratory obstruction
D. Hypotension

190. C 191. B 192. C

474
193. A patient came to dental clinic who has a pulse rate of 72, a
respiratory rate of 15, a BP of 120\80, warm pink extremities and pupils
that constrict during near accommodation. What is the most likely
diagnosis?
A. Normal patient
B. Acute anxiety syndrome
C. Coronary artery disease
D. Myopia

194. An inhaled foreign body most probably enters:


A. Left bronchus
B. Right bronchus
C. Esophagus
D. None of the above

195. Among the following which is an advantage of IV administration of


a drug?
A. Minimal skill is necessary
B. Sedative drugs are compatible with IV solution
C. It eliminates side effects
D. It allows titration of the drug

193. A 194. B 195. D

475
196. Among the following which factor is strongest stimulators to
increase the respiration?
A. Decrease in venous oxygen
B. Increase in blood pH
C. Increase in arterial carbon dioxide
D. Decrease in arterial oxygen

197. During the treatment of shock jugular venous pressure (JVP) should
be maintained in the range of:
A. 15-20 mm Hg
B. 5-10 mm Hg
C. 10-15 mm Hg
D. 0.2-5 mm Hg

198. The average daily adult fluid intake should be:


A. Approx. 4000 ml
B. Approx. 1800 ml
C. Approx. 1200 ml
D. Approx. 500 ml

196. C 197. C 198. B


476
199. The following space infection may cause severe respiratory
difficulty and tracheostomy may require:
A. Pterygopalatine space
B. Canine space
C. Temporal space
D. Parapharyngeal space

200. Of the following which is most common site of metastasis from


mandible:
A. Heart
B. Liver
C. Lung
D. Pancreas

201. After prolonged steroid therapy patient may have:


A. Prolonged post extraction bleeding
B. Analgesic potentiation
C. Relative adrenaline insufficiency
D. Not tolerable to sedatives

199. D 200. C 201. C


477
202. Prophylactic antibiotic coverage for a patient with rheumatic heart
disease is required prior to:
A. Scaling and curettage
B. Extraction of maxillary first molar (Single tooth extraction)
C. Open reduction and internal fixation (ORIF)
D. All of the above required

203. The significance of chronic alcoholism prior to oral surgical


procedure is:
A. Prolonged haemorrhage secondary to liver disease
B. Intolerance to LA
C. Predisposition to syncope
D. Contraindication for N2O analgesics

204. Which of the following is significant in a patient with renal


transplant?
A. Hematocrit 37%
B. Cholesterol -200 mg%
C. WBC -2000
D. BUN -21%

202. D 203. A 204. D

478
205. At what diastolic pressure do you consider the patient to have
significant hypertension?
A. 110
B. 90
C. 85
D. 65

206. Most important single aspect of a patient's surgical experience is:


A. Family history
B. Past medical history
C. Complete blood count
D. Suture technique

207. The hematocrit serves as an index for:


A. Monocyte volume
B. Patient red blood cell volume
C. Patient total blood cell volume
D. Patient white blood cell volume

205. A 206. B 207. B

479
208. Among the following all are important measures to prevent
emergencies except:
A. Monitor the patient intraoperatively
B. Assess your patient's medical status preoperatively
C. Weigh your patient
D. Keep emergency kit up to date

209. Treatment of choice for postoperative cutaneous ecchymosis is:


A. Aspiration of blood
B. Application of cold
C. Use of topical steroids
D. None

210. After the administration of penicillin patient reports itching in


various areas of the body. One should:
A. Ignore the itching
B. Discontinue all medication
C. Call an ambulance and meet patient in hospital
D. emergency room

208. C 209. D 210. D

480
211. Allergic reactions may be characterized by:
A. Bronchospasm
B. Cardiovascular collapse
C. Angioneurotic edema
D. All of the above

212. Continuation of antibiotic medication in gram negative soft tissue


infections is required:
A. At least 2 days after all clinical signs of infection subside
B. Until acute symptoms of infection subside
C. for a minimum of 7- 10 days
D. For a period of 21 days

213. Which of the following values should be taken if there is excessive


blood loss after a surgical procedure?
A. Hematocrit and platelet count
B. WBC and RBC count
C. Hemoglobin and Hematocrit
D. Platelet and RBC count

211. D 212. C 213. C


481
214. Among the following which patient will probably have bleeding
problems postoperatively?
A. Leukemic patient
B. Hypertensive patient
C. Thrombocytopenic patient
D. All of the above arc corrects

215. If a normal patient loss approximately 1000 cc of blood during


surgery, the necessary colloid fluid replacement should be:
A. 1000 cc
B. 2000 cc
C. 3000 cc
D. 6000 cc

216. Blood transfusions may cause complications. Which of the


following are possible complications associated with blood transfusions?
A. Circulatory overload
B. Thrombophlebitis
C. Immediate and delayed hemolytic reactions
D. All of the above

214. D 215. C 216. D


482
Chapter 12: Implant
1. Osseo integrated implants have the following features:
A. They form junctional epithelium with the surrounding tissues
B. They are anchored directly to living bone as determined by radio
graphic and light microscopic analyses
C. They have direct structural and functional connection with bone only
at the radiographic level of detection
D. They form a pseudo-periodontal ligament

2. Osseo integration implies:


A. The process of inserting implants in bone
B. Integration of the bone and implant with fibrous tissue between them
C. At least some direct contact of living bone and the surface of the
implant at the light microscopic level of magnification
D. None of the above

3. An endo-steal implant is an implant inserted in:


A. Periosteum
B. Bone
C. Root canal
D. None of the above

1. B 2. C 3. B
483
4. Endosteal implant can be:
A. Root form implant only
B. Plate form implant only
C. Can be either root form or plate form
D. Combination of both

5. The transfer coping in an implant is used:


A. To position an analog in the impression
B. To transfer the position of the implant in the mouth to the cast
C. To gain exact implant positioning
D. All of the above

6. In case of endosseous implant:


A. A seal between implant surface and soft tissue is not required
B. A permucosal seal of the soft tissue at the implant surface is very
essential
C. Whether the seal is established or not it does not really make a
difference to the success
D. None of the above

4. C 5. A 6. B
484
7. A per mucosal seal in case of a dental implant is:
A. Possible
B. Absolutely impossible as tissue will not stick to the implant surface
C. Not necessary at all
D. None of the above

8. Lack of Osseo integration can be due to:


A. Premature loading of the implant system
B. Placing the implant with too much pressure
C. Overheating the bone during preparation
D. All of the above

9. While inserting an implant a gap is observed between the implant and


the prepared site the implant will most probably:
A. Fail
B. Be a success
C. The gap does not affect the success of the implant
D. None of the above

7. A 8. D 9. A

485
10. the most common types of implants in use today are:
A. Subperiosteal implant
B. Transosteal implants
C. Endosteal implants
D. All of the above

11. Which of these is an indication for implant placement?


A. Inability to wear a removable or complete denture
B. Unfavorable number and location of natural tooth abutments
C. Single tooth loss which would necessitate preparation of undamaged
teeth for FPD
D. Any of the above

12. Ideal amount of bone under soft tissue is:


A. 8 mm vertical and 4 mm horizontal
B. 10 mm vertical and 6 mm horizontal
C. At least 12 mm vertical and 6 mm horizontal
D. Depends from patient to patient

10. C 11. D 12. B

486
13. The minimum space between implant should be:
A. 2 mm
B. 5 mm
C. 3 mm
D. 4 mm

14. This distance between the implant and the superior aspect of the
inferior alveolar canal should be:
A. 1 mm
B. 3 mm
C. 2 mm
D. 4 mm

15. The distance between the implant and the mental foramen should be:
A. 2 mm
B. 3 mm
C. 1 mm
D. 5 mm

13. C 14. C 15. B

487
16. What should the distance between the implant and the post ligament
of the adjacent teeth be?
A. 2 mm
B. 1 mm
C. 5 mm
D. 3 mm

17. Time taken for integration of implants in the maxilla


A. 4 months
B. 2 months
C. 6 months
D. 8 months

18. With respect to anatomic limitation the most straightforward area for
implant placement is:
A. Anterior mandible
B. Posterior mandible
C. Anterior maxilla
D. Posterior maxilla

16. B 17. C 18. A


488
19. A surgical guide template is used in implant dentistry for:
A. Delineate the embrasure
B. Locate the implant within the restoration contour
C. Align implants along the long axis of the completed restoration and
identify the level of the CEJ or tooth emergency profile
D. All of the above

20. The recommended time interval between surgery and placing load in
the posterior mandible is:
A. 2 months
B. 3 months
C. 4 months
D. 6 months

21. The recommended time interval between surgery and placing load in
the maxilla is:
A. 2 months
B. 3 months
C. 4 months
D. 6 months

19. A 20. C 21. D

489
22. Internal irrigation is used for implant surgery:
A. To clear the operative field
B. To cool the rotating bur so that the temperature in bone does not
increase
C. So that bone can absorb the water
D. None of the above

23. Whenever implants are placed in the posterior mandible they should:
A. Engage the superior cortical bone and medullary bone
B. Engage the superior cortical, medullary bone and inferior cortical
bone
C. Engage only the superior cortical bone
D. None of the above

24. Implants placed in the posterior maxilla should:


A. Leave 1 mm of bone between the floor of the sinus and the implant
B. Leave 2 mm of bone
C. Can go into the antrum if required as long as lining is not punctured
D. None of the above

22. B 23. B 24. A


490
25. Probing under local anesthesia is used:
A. To check if any sharp bony edge is there
B. To check for caries
C. To check the length of the implant
D. To judge soft tissue thickness at the planned implant site

26. The transfer coping in an implant is used:


A. To position an analog in the impression
B. To transfer the position of the implant in the mouth to the cast
C. To gain exact implant positioning
D. All of the above

27. The minimum safe distance between an endosteal implant and any
adjacent anatomical structure should be:
A. 2 mm
B. 2.5 mm
C. 3 mm
D. 1.5 mm

25. D 26. A 27. A

491
28. Advantage of root form implants over plate forms include:
A. Greater surface area
B. Fewer pontics
C. Greater bone density
D. All of the above

29. The decrease in bone width within the first one to three years after
tooth extraction is:
A. 25%
B. 30%
C. 35%
D. 40%

30. The following division provides bone in all dimensions for implant
placement:
A. Div. A
B. Div. B
C. Div. C
D. Div. D

28. D 29. D 30. A


492
31. Among the following which indicates successful allogeneic tooth
implant?
A. The tooth survives for 12 months
B. The periodontal ligament is re-established
C. Both the pulp and periodontal ligament survive
D. Ankylosis occur

32. Which of the following is true in regard to reconstruction of an


edentulous mandible with implants?
A. Subperiosteal implant requires only single surgery
B. Staple implant is most useful for the posterior mandible
C. Both blade and the osteointegrated cylinder are useful as posterior
abutments in patients with high mental foramen
D. Osteointegrated concept of implant. Stabilisationhas the best
documentation of long-term success

33. Angle SNA in a cephalogram indicates:


A. Mandible - cranial bone relationship
B. Mandible - maxilla relationship
C. Maxilla - cranial bone relationship
D. Maxillary teeth to cranial bone

31. D 32. D 33. C

493
Chapter 13: Miscellaneous
1. Purpose of taping the eyes shut before surgery is to:
A. Prevent lacrimal secretions which may contaminate the field
B. Prevent corneal abrasion
C. Limit ocular motility
D. None of the above

2. Of the following which is correct? Thiersch graft is in:


A. Full thickness graft
B. Pedicle graft
C. Partial thickness graft
D. None of the above

3. Of the following which is the best method of treating a green stick


fracture of the mandible?
A. Allow normal masticatory movements
B. Bringing the teeth into occlusion with interdental wiring
C. Extra skeletal fixation
D. None of the above

1. B 2. C 3. B

494
4. Which one of the following disorders responds favorably during using
an occlusal separator?
A. Capsular fibrosis
B. Muscle spasm
C. Chronic dislocation
D. Unilateral condylar hyperplasia

5. Visible bleeding of an artery is best controlled via:


A. Pressure
B. Tea bag plus pressure
C. Tourniquet
D. Clamping and ligation

6. The purpose of giving antibiotics prior to oral surgery in patients with


a history of rheumatic heart disease is to help:
A. Eliminate all bacteria from the blood stream
B. Prevent recurrence of rheumatic fever
C. Prevent rheumatoid arthritis
D. Prevent sub-acute bacterial endocarditis

4. B 5. D 6. D
495
7. A graft that has been deprived from another species
Of a different genetic disposition is also known as a:
A. Allograft
B. Isograft
C. Homograft
D. Heterograft

8. Vitamin K is used in the management of post-extraction bleeding


when the cause is determined to be due to:
A. Lowered platelet count
B. Thrombocytopenic purpura
C. Prothrombin deficiency
D. Factor VIII deficiency

9. Site selection for intravenous fluid therapy in an outpatient should be


the:
A. Dorsal vein, at the back of the hand
B. Median cubital vein
C. Femoral vein
D. Cephalic vein

7. D 8. C 9. A
496
10. 'Eagle's syndrome' is associated with the elongation of the:
A. Mastoid process
B. Odontoid process
C. Styloid process
D. Palatine process

11. Dental laser technology is now available for:


A. Soft tissue surgery
B. Treatment of hard tissues
C. Non-surgical applications
D. All of the above

12. Viral infection which can be seen in oral cavity of patients with HIV
is/are?
A. Hairy leukoplakia
B. Herpetic stomatitis
C. Papilloma warts
D. All of the above

10. C 11. D 12. D

497
13. The major criteria of WHO guide line for diagnosing
AIDS consists of all except?
A. Weight loss more than 10%
B. Prolonged fever for more than 1 month
C. Chronic diarrhea for more than 1 month
D. Generalized lymphadenopathy

14. The main target cells of HIV are:


A. Plasma cells
B. T4 cells
C. T8 cells
D. B cells

15. The cells from which HIV can be isolated:


A. Langerhans’s cells
B. Dendritic follicles
C. Alveolar macrophages
D. All of the above

13. D 14. B 15. D

498
16. Body fluids can be responsible for transmission of
A. Blood
B. Cervical secretions
C. CSF
D. Tears

17. The facial development seen in HIV positive children is


characterized by:
A. Normal facial profile
B. Microcephalic
C. Ocular hypotelorism
D. Narrow palpebral fissure

18. Detection of HIV antigen is:


A. Positive after 2 to 4 weeks of infection and then becomes negative till
the clinical disease sets in
B. Positive after 4 to 6 weeks till clinical disease sets in
C. Only positive when clinical disease sets in
D. Positive only after 1 week of infection

16. D 17. B 18. A


499
19. When blood of HIV patients is analyzed it would show:
A. Leukocytosis
B. Decreased ratio of T4/T8 cells
C. Decreased level of IgG
D. Decreased level of IgA

20. The immunodeficiency characteristic of HIV infection is due to:


A. Decreased humoral immunity
B. Decreased cellular immunity
C. Both of the above
D. None of the above

21. When T cell subset assay is done in HIV positive patients there is:
A. Increased ratio of T helper/ T suppressor cells
B. Decreased ratio of T helper/ T suppressor cells
C. Increased ratio of T4/T8 cells
D. Decreased production of T8 cells

19. B 20. B 21. B

500
22. Besides autoclaving, simple method of inactivating HIV is all
except:
A. Dry heat at 100 degree C
B. Boiling for 20 min
C. 1% sodium hypochlorite
D. 6% Hydrogen peroxide

23. WHO guide lines for suspected HIV patients suggests that they
should:
A. Be excluded from the main stream
B. Remain integrated within the society
C. Be excluded to isolation
D. None of the above

24. HIV crosses:


A. Blood-brain barrier
B. Placental and vitreous humour
C. Both of the above
D. None of the above

22. A 23. B 24. C


501
25. The impression material recommended for taking impression for
maxillofacial prosthesis in AIDS patient:
A. Alginate
B. Silicone
C. ZnOE
D. Plaster of Paris

26. Aspirator used for suction should be disinfected after using for HIV
patient by:
A. Flushing with 2% glutaraldehyde and washing with fresh water after
10 min
B. Flushing with 2% glutaraldehyde and leaving overnight
C. Flushing with savlon and washing after 12 hours
D. None of the above

27. Impressions for maxillofacial trauma should be disinfected by:


A. Thorough rinsing with water
B. Immersing in 2% glutaraldehyde for 3 hours after rinsing in water
C. Immersing in 2% glutaraldehyde for minimum of 10min after rinsing
in water
D. Immersing in 2% glutaraldehyde for 12 hours

25. B 26. B 27. C

502
28. The dental infections commonly seen in ADS patient:
A. Necrotizing ulcerative gingivitis
B. Rapidly progressive periodontitis
C. Horizontal bone loss
D. All of the above

29. Body fluid in which HIV is highly concentrated:


A. Semen, cervical fluids
B. Blood
C. Saliva
D. Tears

30. The most common pathogen isolated from pulmonary system of HIV
patient is:
A. Mycobacterium tuberculosis
B. Mycobacterium avium intracellular
C. Pneumocystis carinii
D. None of the above

28. D 29. B 30. C

503
31. HIV virus is a:
A. DNA virus
B. Retrovirus
C. DNA, RNA virus
D. None of the above

32. Diagnosis of pediatric AIDS can be done by:


A. ELISA test
B. Western blot test
C. Virus culture
D. None of the above

33. The first antibody to HIV antigen appears in blood after:


A. 1-2 weeks of infection
B. 2-4 weeks of infection
C. 4-6 weeks of infection
D. 4-6 months of infection

31. B 32. C 33. C

504
34. ELISA test demonstrates:
A. HIV antigen
B. HIV antibodies
C. HIV
D. None of the above

35. The confirmatory test used for HIV infection is:


A. ELISA
B. Immunodot test
C. RIPA test
D. Western blot test

36. HIV can be inactivated by (except):


A. Autoclaving
B. 2% glutaraldehyde
C. Boiling for 20 minutes
D. Gamma radiation

34. B 35. D 36. D

505
37. Following chemicals are used for inactivating HIV (ex.
A. 2% glutaraldehyde
B. 50% ethanol
C. 1% sodium hypochlorite
D. 2.5% cetamide

38. World's ADS Day is observed on:


A. 22nd November
B. 26th February
C. 1st December
D. 29th November

39. The recommended method for sterilizing impressions for


maxillofacial prosthesis is:
A. Impression in silicone and autoclaving
B. Impression in alginate and soaking in 2% glutaraldehyde for 10
minutes
C. Impression in silicone and soaking in 2% glutaraldehyde for 20
minutes
D. Impression in silicone and soaking in 2% glutaraldehyde for three
hours

37. D 38. C 39. D

506
40. The working surface in operation theatre should be disinfected
especially for HIV, by using:
A. Savlon
B. Gamma radiation
C. Hypochlorite solution
D. UV light

41. Best way to palpate submandibular gland is:


A. Bimanual extraoral palpation with the patient head tipped forward
and towards the same side
B. Monomanual extraoral palpation with patient's head tipped forward
and toward the same side
C. Bimanual, simultaneous intraoral and extraoral palpation
D. Intraoral palpation with the patient head tipped forward

42. In an irradiated field, the optimal time for a surgical procedure is:
A. An Immediately after completion of radiotherapy
B. Any time during radio therapy
C. 6 months to 1 year after the completion of radiotherapy
D. 4-6 weeks after radiotherapy

40. C 41. C 42. D

507
43. Commonly used suture material for closure of intraoral wound is:
A. Nylon
B. Black silk
C. Cat gut
D. Chromium catgut

44. The following is one of the advantages of chromic gut over plain gut
suture:
A. Greater ease of use
B. Non absorbability
C. Greater strength
D. Less expensive

45. Vertical mattress sutures are frequently used in closure:


A. To Evert margins
B. In one tissue layer thick flaps
C. Because of the lack of sufficient blood supply
D. None of the above

43. B 44. C 45. A

508
46. Virus which causes ADS is a:
A. Retrovirus
B. DNA virus
C. Reovirus
D. None of the above

47. The following suture material has the memory prop-


A. Silk
B. Catgut
C. Nylon
D. Linen

48. Which of the following is non resorbable suture material?


A. Dacron
B. Catgut
C. Polyglycolic acid
D. Polyglactin

46. A 47. C 48. A

509
49. Presence of suture usually increase the susceptibility to infection by
a factor of:
A. 10 times
B. 100 times
C. 1000 times
D. 10,000 times

50. Langer's lines usually run:


A. Parallel with skin creases and parallel to the action of underlying
muscles
B. Perpendicular with skin creases and parallel to the action of
underlying muscles
C. Parallel with skin creases and perpendicular to the action of
underlying muscles
D. Perpendicular with skin creases and perpendicular to the action of
underlying muscles

51. Among the following suture material which one elicit more tissue
reaction:
A. Catgut
B. Silk
C. Nylon
D. Linen

49. D 50. C 51. A

510
52. Which of the following is commonly used as preservative for Gut
sutures:
A. Ethyl alcohol
B. Isotonic saline
C. Hypertonic saline
D. Isopropyl alcohol

53. Surgical needle and sutures are usually sterilized in manufacturing


units by:
A. Gamma radiation
B. X-rays
C. Boiling
D. Dry heat sterilization

54. A patient admitted to the hospital for dental treatment by a dental


staff member is the responsibility of the:
A. Family physician
B. Admitting dentist
C. Kesident physician
D. Physician consultants

52. D 53. A 54. C

511
55. Completed patient hospital records or charts are the legal property of
the:
A. State
B. Patient
C. Hospital
D. Doctor in charge

55. C

512

You might also like